Sunteți pe pagina 1din 391

ifferent from an allopathic diagnosis of the lower extremity?

Motions
=Gross vs. fine
Relationships
=Interrelatedness of subtle dysfunction on rest of functioning organism

Diagnosis & Treatment:


Upper Half of the Body
Anterior Ribs
Thoracolumbar Junction
Cervicothoracic Junction
Thoracics / Other Ribs
Scapulothoracic Articulation
Craniocervical Junction / Cranium
Cervicals
Rest of Upper Extremity

Lower Half of the Body Diagnosis and Treatment Sequence


1. Hip restrictors (some of which cross the Lumbo-sacral junction)
2. Pubic bone dysfunction
3. Superior innominate shear ("innom. upslip")
4. Lumbar dysfunction (esp. L5 or, less often L4)
5. Sacral dysfunction
6. Innominate dysfunctions (other than upslip)
7. Iliopsoas (including thoraco-lumbar junction, approx. T11-L2)
8. Other Lower Extremity Dysfunction

Most Common SD
Fibular head (ant/post)
Tibial rotation (int/ext)
Talus (ant/post)
Calcaneal (inversion/eversion)
Dropped cuboid/navicular
Metatarsal dysfunction

Common Fibular n.
=Wraps around posterior part of fibula
=Sensitive to trauma, fracture or posterior fibular head dysfunction
=Injury at knee can produce weakness of dorsiflexors and foot drop (supplies
(motor)
==anterior--deep fibular== lateralsuperficial fibular-compartment of the leg, sensation to anterolateral
leg)

Tibial n.
Posterior compartment of the leg
Muscles of the foot

Sympathetic NN.

= Pregangionic fibers arise from T11-L2


= Shared with innervation of pelvic organs via least splanchnic n. and lumbar
splanchnic n.
= May explain Chapman's points along ITB for colon problems
= L1-3 dermatomal pattern anterior thigh

Lymphatic Drainage- Superficial


=Abdominal wall below umbilicus
=Gluteal region below iliac crest
=Perineum (part of anal canal, superficial 1/3 of penis, vulva, vagina)
=Superficial part of thighs, legs, feet
=Drainage into superficial nodes in inguinal and femoral triangle thendeep to
nodes and channels around femoral vein in upper thigh

Lymphatic Drainage- Deep


=Parallel pathways along iliac veins to cysterna chyli
= Drains to thoracic duct (left lymphatic duct)
= Receives drainage from left side of head and neck, posterior and upper lobe of left
lung.

Functional Anatomy: Knee--Articular- femur and tibia= motions and


compare
Double condylar complex synovial joint formed by femoral condyles and tibial
plateau
Medial and lateral semilunar cartilage (menisci) provide stability- resistance to
pressure
Medial condyle longer than lateral. Provides for
=posterolateral glide with internal rotation
=anteromedial glide with external rotation

femur and patella joint- patellar tendon cord level- and tendon at
patella
=Gliding joint
=Tendon of quadriceps femoris incorporates patella and attaches to tibial tuberosity
Patellar tendon reflex at L4

Articular- medial and lateral collateral ligaments


Lateral
Stabilization against lateral displacement
Not attached to lateral meniscus
Medial
Stabilization against medial displacement
Attached to medial meniscus
Makes medial cartilage more susceptible to medial displacement or twisting

Functional Anatomy: Knee--Articular cruciate ligaments


Anterior attaches to anterior tibia
Prevents excessive anterior tibial glide
Posterior to posterior tibia
Prevents excessive posterior tibial glide

Somatic dysfunction of knee


minor motions:
-anterior posterior glide
-medial/lateral glide
-anteromedial or posterolateral glide (associated with int/ext rotation)

Proximal Tibiofibular Joint- tibular position. and motions of fibula


= Separate synovial joint at the knee
= Oblique angulation- lateral anterior to medial posterior position*
=Motions at proximal and distal fibula are reciprocal*
=Dorsiflexion - foot- moves distal fibula posteriorly, proximal fibula glides anterior
=Opposite with plantar flexion (fibular head posterior)

Ankle--Articular- is there pure supination or pronation? - also what is


included in the ankle?
- Distal tibia and talus
- Ankle mortice includes distal fibula (lateral malleolus)
- Transverse axis of ankle joint not parallel to transverse axis of body, foot is
normally in slight abduction
No pure supination or pronation of the foot

ankle motions
Supination Equivalent
Inversion
Plantar Flexion
Adduction
Pronation Equivalent
Eversion
Dorsiflexion
Abduction

Lateral Ankle Ligaments**


Anterior talofibular ligment
Posterior talofibular ligament
Calcaneofibular ligament

ligaments disrupted in ankle inversion or ___________ sprain


Grade I --anterior talofibular
Grade II -add calcaneofibular
Grade III- all lateral ligments torn

Motions of the foot- what does the knee and tibia do?
Abduction
External rotation of the tibia
Anteromedial glide at the knee
Adduction
Internal rotation of the tibia
Posteorlateral glide at the knee

Functional Anatomy: Foot--Muscles and Tendons - Myofascialposterior and medial maleolus


Achilles tendon- attaches to posterior calcaneus from common origin of
gastrocnemius and soleus mm.
Posterior to Medial Malleolus
Posterior Tibial Tendon
Flexor Digitorum Longus
Flexor Hallicicus Longus
"Tom, Dick, and Harry"

Foot Anterior Tendons


Anterior Tendons
Anterior Tibialis Tendon- Supports longitudinal arch
Extensor Digitorum Longus
Extensor Hallicus Longus
"Tom, Dick, and Harry"

Tendons Posterior to Lateral Malleolus


Fibularis Brevis
Fibularis Longus- Supports transverse arch

Longitudinal Arches
Lateral
Calcaneus, cuboid, and metatarsals 4 and 5
Medial
Talus, navicular, three cuneiforms, metatarsals 1-3

Transverse Arch- whats the muscular support. what bones make up


this arch
Cuboid, navicular, three cuneiforms, and metatarsal bones
Muscular Support--Peroneus (fibularis) longus inferiorly, and tibialis anterior (first
cuneiform and first metatarsal

Plantar Ligament Support


Plantar Aponeurosis
Long plantar ligament
Short plantar ligament
Spring ligament

Motions of the foot- inversion and eversion


Forefoot inversion (supination strain)
Talus glides posterolaterally at talocalcaneal joint relative to the navicular
Forefoot eversion
Talus glides anteromedially

Scan (Knee)
Orthopedic Testing
-Medial and lateral collateral ligament
-Patellar grind
-Drawer test
-Lachman's sign

Motions: KNEE and a bunch of other

KNEE
=Lateral/Medial
=Anterior/Posterior
=Anteromedial (EXT rot)/Posterolateral (INT rot)
Fibular head motion
Lateral Malleolar Motion
Strain to Interosseous Membrane
Ankle Motion
-Talotibial motion
-Subtalar motion
-Intertarsal motion

Foot motion
Navicular bone dysfunction
Plantar glide and medial rotation (inversion)
Cuboid bone dysfunction
Plantar glide and lateral rotation (eversion)
Cuneiform dysfunction
Plantar glide without rotation
-Tarsal metatarsal joint motion
=Generally less motion at the 2nd metatarsal
=Minor motions are anterior/posterior glide, medial/lateral glide, internal/external
rotation glide

Scan: Hindfoot and Midfoot


Subtalar
Inversion and Eversion at Talar Calcaneal
Navicular
Plantar position and tenderness
Medial and inferior glide
Cuboid
Plantar position and tenderness
Lateral and inferior glide
Cuneiforms
Plantar position
Plantar/inferior glide
Metatarsals
Adduction, Abduction, Plantar, Dorsiflexion, Rotation
Motion and Position

Functional Anatomy: Subtalar Joint


Talocalcaneal Joint
Secondary glide motions are
Inversion
Eversion
Also Talonavicular glides
Anteromedial (eversion)
Posterolateral (inversion)

LE Scan Sequence**

Knee
-Internal/External Tibial torsion
-Fibular Head
Midshaft
-Interosseous
Ankle/Foot
-Talus
-Subtalar
-Tarsal
-Metatarsal
IFITSTM
I found in the stone, the mallet

Diagnosis of Extremities: General SCANComparison is made to opposite extremity


Scan is guided by:
Chief complaint or
Screening exam findings in absence of complaint
Scan checks for
Position and
Motion

Diagnosis of Extremities: General- SCAN!


Named in ease
-Medial Sternoclavicular anterior and superior
-Posterior Fibular head
Dysfunction in secondary glides of joint (mainly)

Treatment Options
Direct
Find resistance position of joint
Move through that resistance by muscle energy, springing, or thrusting
Indirect
Find position of ease in all planes and phase of respiration
Refine position as release occurs

Scan: Knee
Position: Tibial tuberosity
Motion: Tibial rotation

Scan: Ankle
Position: Talar dome
Motion: Talar glide

Foot

Epicondyles of the femur


-Palpate the patella
-Slide your thumb lateral to the outside of the knee
-Slide back to the patella and move medially

Tibial tuberosity/ Patella how to do it? WHY?

Sit, knee flexed


-Place hand on the anterior aspect of the knee ("knee cap")
-Move fingers inferiorly until encountering a boney prominence
-Used to determine medial/lateral rotation of the tibia
-Continue down the tibia until reaching the medial malleolus

Tibial Plateau
-Place thumbs on both sides of the patella
-Slide inferiorly into soft, joint space
-Continue to palpate inferiorly until you feel bone

Q Angle
Less than 15 degrees in men
Less than 20 degrees in women
-Increase in Q angle can result from excessive ankle pronation
-Increase in Q angle may result in patellofemoral syndrome

Head of the fibula- WHERE?


Locate the tibial tuberosity-Slide fingers laterally 3-4 inches- Biceps Femoris tendon
inserts at the head of the fibula
-Have your partner plantar flex the fibular head. Assess motion.
-Grasp fibular head and input anterior/posterior motion
-Trace fibula inferiorly and this becomes the lateral malleolus

Talus assessment
-Locate the medial and lateral malleoli
-Now, move medially and anteriorly to the midline of the foot.
-Palpate the rounded prominence. This is the head of the talus
-Input rotary motion between the talus and calcaneus and compare motion
bilaterally

Landmark Palpation
Cuboid
Navicular
1st metatarsal head
5th metatarsal head
Cuneiform bones
Anterior talo-fibular lig.

Calcaneo-fibular lig.
Posterior talo-fibular lig.

Calcaneus where is it
Find the medial and lateral malleoli
Drop inferiorly onto the calcaneus (heel of foot)

Navicular & Cuboid WHERE ARE THEY?


- Navicular is located just inferior and distal to the medial portion of the head of the
talus.
-Invert and evert the foot to assure contact with the navicular, in inversion the
navicular tubercle will become more prominent
-Cuboid is located just proximal to the 5th metatarsal and articulates with the
calcaneus.
-Locate the tuberosity of the 5th metatarsal and move your contact proximally until
a drop off is felt
-Note the articulation between the cuboid and navicular

Cunieforms & Metatarsals


-Begin palpation just proximal to the metatarsal phalangeal joint.
-Explore all 5 metatarsal bones-The 5th metatarsal bone has a lateral prominence
named the tuberosity of the 5th metatarsal
-Locate the 1st metatarsal and move proximally to palpate a small joint between the
medial cuneiform and the 1st metatarsal
-Move onto the cuneiform and examine the bony structures more laterally (middle
and lateral cuneiforms)

Screen/Scan for Somatic Dysfunction in the Lower Extremity


-Directed at region of interest as identified by the patient (chief complaint)
-Assessed bilaterally for comparison of motion
-Focus is usually on smaller secondary motions
-Attention must be given to joints above and below complaint area (at least)
-If dysfunction appreciated in neighboring regions must be further examined.

What is Postural Balance?


Continual adaptation of muscular tone in response to gravitational influences
(among others).

DEFINE! Optimal Postural Balance


Perfect distribution of body mass around the center of gravity, with compressive
forces on spinal disks balanced by ligamentous tension.
Osteopaths have agreed that the center of L3 is the center of average, passive,
standing adults. Like this guy.

Why is Postural Balance so Important?


-Posture is but the formal expression of the balance of power existing between the
environmental force of gravity and the strength of the individual. Thus any
deterioration of posture indicates that the individual is losing ground in her contest
with the environmental forces of gravity."

Respiratory-Circulatory Model
The artery is the river of life, health and ease."
A.T. Still, M.D.
1)for health there must be good circulation, to ensure proper nutrition and drainage
from a gross to a cellular level (and back again)
2) In order to achieve this, the respiratory processes must be working efficiently.

J Gordon Zink D.O. AND FASCIAL DYSFUNCTION- discuss his theory


-Commonly found patterns of dysfunction

-Fascia: sheets of fibroelastic connective tissue throughout the body


4 "crossover sites" where fascial tension can occur
Based on Junction/transition areas
Cranium
Thorax
Pelvis
And the diaphragms that bridge them
-Tentorium
-Thoracic inlet
-Abdominal diaphragm
-Pelvic diaphragm
-Torsions of these fascial pathways can compromise neural vascular flow.
Embryologically, transition zones are weaker, and more exposed to acute or
repetitive trauma.

compensatory and noncompensatory mechanisms


Common compensatory pattern: a series of myofascial torsions that are compatible
with physiologic function
Non-compensatory: not rotated in alternating directions
-Compromises respiratory-circulatory integrity of body
-Can be traumatically induced
-Will eventually display congestive systems

4 types of alignment- case considerations


IDEAL, KYPHOTIC, FLAT BACK, SWAY BACK
The complaint of "Neck Pain shows up in two different biomechanical environments
in these two patients. (image 1 and 2)
Which one is more likely to complain of tension headaches? #2

Which one is more likely to be able to take a deep breath? #1


Which one might be more structurally vulnerable to case of pneumonia? #2
Structure of the spine will affect the function of the thoracic viscera- heart, lungs.
Can this person take good deep breaths? Can the heart efficiently provide enough
blood to the circulatory system?

Potential Postural Influencing History


=Congenital/Birth history
Congenital scoliosis (see appendix)
Shoulder dystocia, torticollis, hip dysplasia
=Trauma/Surgeries/Injuries
Childbirth: vaginal, c-section
Knee/hip replacement
Postural imaging studies
=Medical Diagnosis
Asthma
Osteoporosis/penia
=Activities
Occupation!
Swimming
Weight lifting

Diagnosis: Physical Exam


Visual Inspection : look at your patients critically*
*Remember: words have weight; thoughts are things
Structural Exam:
-gravity line
-landmarks
Palpatory Exam
-screen and scan

Landmark review
-Base of Occiput
-Scapular Spine
-Inferior angle
-Iliac crest, PSIS
==Standing flexion
-Knees
-Arches of feet

gravitational line
Gravitational Line Should pass through:
1. External auditory meatus
2. Lateral head of the humerus
3. Center of the Body of L3
4. Anterior third of the sacral base
5. Greater trochanter
6. Lateral condyle of the knee
7. Lateral malleolus
AH BS TLM

OA junction: base of occiput- case considerations


= Biomechanics: tonic neck reflexes; cranial strain patterns
= Neural: CN exit skull in this neighborhood
= Fluid: vertebral artery to interior of cranium; sagital sinus, transverse sinus as
venous drainage
= Viscera: CNS; sinuses
= SD at this level can induce
- Hypertonicity of postural muscles
- Disturbances of facial symmetry, equilibrium and locomotor deficits.
CNs: 10,11,12

CT junction: scapulae case considerations


= "relatively mobile cervical spine meets relatively rigid thoracic spine."
=Biomechanics: UE-shoulder girdle influences
==SITS muscles
==Serratus anterior, rhomboids, pectoralis etc
=Neural: brachial plexus, phrenic, vagus; upper sympathetic chain
=Fluid: thoracic inlet
=Viscera: head/neck, heart, lungs, etc

TL junction: (12th) ribs, iliac crests case considerations


=Diaphragm!
==Biomechanics: skeletal attachments
==Fluid: Aorta, Vena Cava, Azygous; fluid pump of body

==Visceral: esophagus, lower GI, liver, kidney


==Neural: phrenic; visceral symp/parasymp considerations
=Postural imbalance of Lower Body (biomechanics)
==Tonic: Iliopsoas, QL, erector spinae
==Phasic or Inhibited: Rectus abdominus

Lumbo-sacral junction:
=Horizontal alignment of Sacral Base:
==LOTS of implications into posture, pain.
=Fluids: inguinal nodes, pelvic diaphragm
=Neural: lumbo-sacral plexus, pelvic and sacral splanchnics
=Viscera: lower GI, reproductive, urinary considerations
Biomechanics: _____ _____ ______?

Muscle phys review: alpha gamma motor neuron review


= A disturbance of MS function initiates a series of events beginning with
stimulation to mechanoreceptors and nociceptors, resulting in afferent neural
activity...
= The final common pathway is the alpha motor neuron that stimulates the muscle
fiber to contract, and through the gamma system, the muscle spindle to adapt,
resulting in alteration in muscle tone. = Chronic dysfunction feeds the afferent loop,
more nociception and abnormal mechanoreceptor information, perpetuating
ongoing aberrant muscle tone.Disturbance
Results in alteration, adaptation
Interruption and reprogramming of the vicious cycle contribute to improvement of
overall muscle tone and balance.

What this means=


JOB SECURITY

Short Leg Syndrome


Condition in which there is an anatomical or functional leg length discrepancy that
results in
1. Sacral Base unleveling
2. vertebral side bending AWAY and rotation TOWARDS low side
3. Innominate rotation and side shifting towards long leg
4. and more
Earth is mostly even- Eyes level with the horizon: pelvic imbalance can manifest
further away (headaches) because of the body's ability to compensate and adapt to
imbalance
Between earth and eyes is where the compensation and dysfunction can occur.
Pelvis dropped toward the short leg
Spine curves convexly away from the short leg- shoulders compensate opposite to
pelvis

Common Findings in Short Leg syndrome


1. Sacral Base unleveling : lower on the side of the short leg.
2. Anterior innominate rotation on the side of the short leg- Iliacus contracture can
be source of this.
3. Posterior innominate rotation on the side of the long leg.
4. Lumbar spine will sidebend away and rotate toward the side of the short leg
5. Iliolumbar ligaments may become stressed on the side of the short leg.
6. SI ligaments may become stressed on the long leg side.

What happens when muscle pairs exert uneven pull on foundation?


how to fix?
-Muscle Imbalance:
-Upper Cross Syndrome
-Lower Cross Syndrome

WAYS TO FIX? Fundamental Principles of Muscle Imbalances


Sensory motor balance training: more here in year 2!
Stretching of short, tight (tonic) muscles FIRST.
Strengthening weak, inhibited (phasic) muscles NEXT.
Aerobic conditioning
Overall Goals:
=Maintenance of enhanced MS functional capacity
=achieved by appropriate manual med intervention and a continual exercise
program

Upper and Lower Cross Syndrome- #1 postural imbalance. Upper


cross
POSTURAL IMBALANCE
- Postural muscles tend towards HYPERtonicity;
- dynamic muscles (phasic) tend towards HYPOtonic overstretch.
Tonic = Tight = Short = Strong
Phasic = weak = Long
-Asymmetry in tensions of agonist-antagonist pair
-Imbalance can be in the coronal/frontal plane, as in scoliosis... or
anterior/posterior, as in upper and lower cross.
Upper cross: anterior compartment tightens while the posterior torso weakens .

'Lower cross' follows similar general patterning


- Patterns, not absolutes.
- Postural imbalance of Lower Body (biomechanics)
= Tonic: Iliopsoas, QL, erector spinae, tensor fascia lata, adductors, piriformis,
hamstrings.
= Phasic or Inhibited: Rectus abdominus, gluts (max, med, mini) obliques, vastus
medialus, lateralis, tibialis anterior
-Stretching of short tight muscles
-Strengthening weak muscles
-Aerobic conditioning

Upper Cross Imbalance:


Upper cross: anterior compartment tightens while the posterior torso weakens .
Tonic
**Pectoralis Major
Pectoralis Minor. L Scapulae, Tere Major, upper Trapezius**
Anterior Deltoid
Subscapularis. Latissimus Dorsi, Teres Major
SCM, Scalenes, Rectus Capitus
Phasic
Rhomboids
Lower/Middle Trap, Serratus Anterior
Teres Minor, Infra Spinatus Posterior Deltoid
Longus Coli, Capitus

Upper Cross Syndrome -in light ofUpper Half of the Body Tx seq.
Anterior Ribs
=Pectoralis mm
Thoracolumbar Junction
=Diaphragm redoming
Cervicothoracic Junction
=levator scap
Thoracics / Other Ribs
=Rhomboid, lower trap strengthening exercise
Scapulothoracic Articulation
=Serratus anterior strengthening exercise
Craniocervical Junction / Cranium
Cervicals
Rest of upper Extremity

Lower cross in context of Lower Half Sequence:


Hip restrictors
Iliacus -usually tight
2. Pubic bone dysfunction
rectus abdominus -usually inhibited
adductors -usually tight
3. Superior innominate shear
QL -usually tight
4. Lumbar dysfunction
5. Sacral dysfunction
6. Innominate
7. Iliopsoas (including thoraco-lumbar junction, approx. T11-L2)
usually tight; can contribute to functional short leg, psoas syndrome
8. Other Lower Extremity Dysfunction
hamstring -can be either tonic or phasic

gastroc glut max-min usually phasic, but can be either

Summary on Posture
-Inhibited muscles are going to become weak and overstretched; facilitated muscles
will become tight and constricted.
Upper cross- kyphotic
Lower cross sway back
Great transition into tensegrity: These are common patterns of dysfunction- agonist
and antagonist asymmetry. However it's even MORE complicated!
The take home point is that the muscles and ligamentous structures such an
important influence on the way we move- and therefore the way are bodies and
structured and how they function.

Postural Balance
Continual adaptation of muscular tone in response to gravity.
Changes with:
Body habitus
Medical Status
Phases of life--> Pregnancy
Daily Activities

Watching someone breathe

Medically important in context of resp-circulatory model


Breath should move the torso from the neck, down to the pubic bone
Look for: motion from neck to pubic bone
Areas here that are Restricted to motion

#1: Anterior Ribs: Pectoralis AND ATTACHMENTS!


Treat tight muscles first!
Pecs
Pt supine
Assess anterior aspect of shoulder: is one higher off the table the other?
Can also be assessed seated/standing.
Prox attachment: clavicle, sternum, superior 6 costal cartilages
Distal attachment: intertubercular groove
Innervation: medial and lateral pec nerves (cord levels C5-T1)
Action: adducts, internally rotates the humerus; draws scap ant and inf.

Stretching Hypertonics: Pectoralis


Working with your knowledge of muscular attachments (oh immersion!)
Isolate and stretch pec major and minor
How can this be modified to a ME treatment!?

Stretches to Teach: pec minor


Doorway Stretch With back straight, align elbow with eye against frame
step forward with one foot.
Feel the stretch in the Pectoralis area.
10-15 seconds, 3-5x
Should pt stretch both sides?

Upper half #2 Thoracolumbar Junction treatment


Diaphragm redoming:
Diagnosis: using F/E, SB, R, determine motions of ease.
Treatment: Carry thorax to stack these motions.
Hold in this position
Patient can use breath to assist in "unwinding"
BIG, long, easy breaths in and out, or breath holding.
Follow diaphragm IN, with broad pressure, on exhale
Until release is felt
Return to neutral and reassess.

Diaphragm redoming: new


Pt supine, doc seated
Pass your hands around A/P diameter of thoracic cage.
Anterior hand at lower costal cage
Posterior hand at thoracolumbar fascia/upper lumbars
Engage the tissue
Treat with direct or indirect methods, stacking in F/E, SB, R.

lower half #2 2) Pubic bone dysfunction


Assess landmarks: pubic tubercles
Treat asymmetric side
If no asymmetry:
Use AB/Adduction muscle energy
2-3 times in each direction

Upper half #3
Upper picture shows combination of Levator and Pec
How can this be modified to a ME treatment!?

Stretches to Teach: Levator Scapulae


Patient seated:
Patient holds on to the side of the chair with one hand and flexes the head as if
looking and into the opposite breast pocket.
Free hand is placed on the head and a gentle stretch is placed inferior and slightly
lateral. Hold for 5-15 seconds and repeat.

Address Phasic Muscles.


Inhibited muscles second!
Usually rhomboids, mid/low trapezius, +/- serratus anterior, latissimus.

#6 in upper - Occipital triangle


Craniocervical junctions is last stop for postural asymmetry. Eyes want to be level
with the horizon, semicircular canals demand a level environment to direct the head
and body. Postural asymmetry from the base up is "corrected" as a last effort at the
base of the skull to allow the extraoccular muscles the most level approach to
environment as possible. These Occ triangle muscles have quite an intimate
relationship with the EOM, and can perpetuate asymmetry if not corrected. See
Myers' Anatomy Trains, Superficial Back Line for more information

Indirect Tx for Occipital triangle


- Pt supine.
- Find base of occiput with fingerpads
- Allow pt's head to sink your fingers into the soft tissue.
- Pause here.
- Patient gazes softly to left; neutral; softly to right.
Use extraoccular muscles to stack ease onto F/E, SB, R at occipital base.

summary 1

summary 2

Supplemental Exercises for Lower Trapezius


Patient prone, doc on side being tested
Ask patient to activate Lower Trap by pulling scapula inferiorly and medially
towards the spine
Retraining: airplane arms! (to move activation lower, can instruct pt to move into
"warrior I arms"

Lower Trapezius at home


Prone:
or
Standing, pull scapula together along spine (retraction)
Bring elbows flexed or extended along towards sides to approximately 45
Repeat 5-7 x
(Also addresses rhomboids)

Serratus Anterior: innervation and function


Prox attachment: Anterior surface of medial border of scapula
Distal attachment: ribs 1-8
Innervation: Long thoracic nerve!
Action: protracts and rotates scapula.- ABDUCTION

Supplemental Exercises for Serratus


Serratus Anterior -With patient laying on their side. The elbow bent, while flexing at
the shoulder.
Stretch for lower fibers: inferior angle moves medially and superiorly*
Upper fibers: move sup angle medially and inferiorly

serratus strengthening
Strengthen: Keep the arm parallel to the floor. May add resistance when the patient
is able. (Protracts Scapula)
Semi-truck Honk Motion.

Supplemental Exercises for Rhomboids


Patient prone, doc on side being tested
Ask patient to activate rhomboids, by pulling scapula inferiorly and medially
towards the spine
Monitor along superior medial border of scap
Retraining: Jet fighter plane arms!

Techniques/Skills
Anterior/Posterior Fibular Head
-ME
-Direct/Indirect
Anterior Posterior Glide Dysfunction, Knee
-Direct/Indirect
Interosseus Membrane
-Indirect
Anterior/Posterior Talus
-ME
-Direct/Indirect

Subtalar Inversion/Eversion
-Indirect
Plantar Navicular/Cuboid/Cuneiform
-Direct/Indirect
Dorsal/Plantar/Medial/Lateral Metatarsal
-Direct/Indirect

Dysfunction: Knee
Anterior tibial glide
Posterior tibial glide
Medial tibial glide
Lateral tibial glide
Anteromedial tibial glide
Posterolateral tibial glide
Most common: anteromedial glide, medial glide and posterior glide

Treatment: Knee
Direct
Find resistance position of joint
Use muscle energy or springing to activate
Indirect
Find position of ease in all planes
Use breath as activating force

Dysfunction: Anterior Fibular Head


Treatment: Direct Articular (Supine)
Flex knee slightly
Internal rotation of tibia
Thumb over fibular head, opposite hand stabilizes lateral malleolus
Rapid extension of knee
Encourage posterior movement of fibula
Treatment: Direct Muscle Energy
Seated position
Thumb over fibular head directing posterior
Plantar flexion of ankle to resistance (encourages inversion, supination, internal
tibial rotation, posterior proximal fibula)
Isometric contraction dorsiflexion
Repeat

Dysfunction: Posterior Fibular Head


Treatment: Direct Articular
Flex knee
External rotation of tibia
MCP or thumb posterior to fibular head

Flex to resistance
Spring flexion of knee to encourage fibular head anterior
Treatment: Direct Muscle Energy
Seated position
Thumb and finger grasp fibular head
Dorsiflexion of ankle to resistance (encourages eversion, pronation, external tibial
rotation, anterior proximal fibula)
Isometric contraction plantarflexion
Repeat

Dysfunction: Interosseous
-Proximal or distal restriction identified by testing motion while palpating both
proximal fibular head and lateral malleolus
Treatment Indirect
-while holding proximal and distal fibula, balance ease within interosseus
membrane and soft tissue, use breath as activating force

Dysfunction: Ankle
-Anterior/posterior talus
Most common: anterior/plantar flexed talus
Treatment Direct
-Seated or supine-- dorsiflex foot until talus reaches edge of resistance, have patient
plantarflex against resistance
Treatment Indirect
Seated or supine plantarflex foot to ease, balance all motions possible, use breath as
activating force

Dysfunction: Subtalus + motion


Inversion/eversion of calcaneous
With associated posterolateral/anteromedial talus
Treatment Indirect
Seated or supine
Stabilize talus, find balanced ease in calcaneous, use breath and reposition as
spontaneous release

Dysfunction: Navicular/Cuboid/Cuneiform +motion


Plantar and medial rotationnavicular
Plantar and lateral rotationcuboid
Plantar cuneiform
Treatment Direct ("Hiss Whip")
Patient prone, thumb over plantar dysfunction, plantar flex to resistance, move foot
"to and fro," then rapid short thrust through resistance

Treatment Indirect
Balance ease in all planes, use breath, re-establish position in release

Dysfunction: Metatarsal + motion


Dorsal/plantar glide
Medial/lateral glide
Treatment Direct
Seated or supine. Stabilize metatarsal in question. Articulate MT through range of
motion.
Treatment Indirect
Seated or supine. Balance metatarsal into ease, use breath to facilitate release,
reposition accordingly

Upper Extremity:
Screen
Screening:
Two Tissue Textures
Two Motion Tests
At each joint
Evaluate Joint Position
Palpate minor motion of joints
Specific Joint will be compared bilaterally
Note: Unlike the axial spine, comparison of joints not above and below

Upper Extremity:
Areas to Scan
Shoulder Complex (4)
Sternoclavicular,
Acromioclavicular,
Glenohumeral,
Scapulothoracic
Elbow Complex (3)
Radial Head,
Ulnar-Humeral Joint
Interosseous Membrane
Wrist (3)
Carpals, Metacarpals, Phalanges

Upper Extremity:
Segmental Definition
-Determined during scan
-Joint named for position of ease or motion of ease
-Finding is relative to contralateral UE
Ex. Sternoclavicular Joint
Superior Clavicle at Sternoclavicular Joint
Superior Glide at Sternoclavicular Joint
Note: Segmental definition in upper extremity is similar to segmental definition in
the axial spine

Shoulder Complex
Normal motion at shoulder involves motion at multiple joints
Sternoclavicular
Acromioclavicular
Scapulothoracic
Glenohumeral
All 4 joints are assessed during upper extremity Scan

Sternoclavicular Joint - position and motion


Position:
Superior/Inferior
Anterior/Posterior
Superior/Inferior Glide
Anterior/Posterior Glide
Rotation
This is the only bony attachment of your arm to the body! Eek
Important for proper shoulder motion
Only bony articulation to attach UE!

testing sternoclavicular joint


To Palpate Proximal Clavicular Position:
Physician Behind Patient
Thumbs on superior portion of proximal clavicle bilaterally

Assess for superior/inferior position


Fingers placed on anterior surface of clavicle bilaterally
Assess anterior/posterior position
Note: can be performed seated or supine

Shoulder Complex:
Sternoclavicular Joint Motion To Test Superior/Inferior Glide:
Shrug Test
To Test Superior/Inferior Glide:
Palpate superior aspect of clavicle
Patient "shrugs" shoulders, then returns to neutral
Normal Motion
Proximal clavicle glides inferior with shoulder shrug
Proximal clavicle glides superior with return to neutral
Objective: Right Clavicle depressed at the SC joint with Inferior Glide
Assessment: Somatic Dysfunction of the Upper Extremity

Shoulder Complex:
Sternoclavicular Joint Motion To Test Anterior/Posterior Glide:
To Test Anterior/Posterior Glide:
Palpate anterior aspect of clavicle
Patient adducts and flexes shoulder to 90
Patient reaches forward, then returns to neutral
Normal Motion:
As patient reaches forward, clavicle glides posterior
Return to neutral, clavicle glides anterior

Ex:
Objective findings: Right clavicle anterior at SC joint with Anterior Glide
Assessment: Somatic Dysfunction of the Upper Extremity

Diagnosis Review & Treatment


-Sternoclavicular Jt
Direct Springing-Anterior & Superior Glide
-Doc places caudal hand on the table and cephalad hand on inferior portion of the
proximal clavicle
-Patient uses opposite hand to stabilize the arm
-Spring posteriorly and inferiorly-lateral
-Recheck

Acromioclavicular Joint position and motion


Position:
"Stair-step" - drop off from clavicle
Motion:
Abduction vs Adduction
Internal Rotation vs External Rotation
Follow clavicle distally to acromion
Place index finger on distal clavicle
Place middle finger on acromion

Palpate for "stair-step"


Compare bilaterally

motion testing for AC joint- To Test for Abduction vs Adduction:


To Test for Abduction vs Adduction:
Midde finger on distal clavicle
Index finger on acromion
Passively abduct arm while palpating AC joint
Palpate joint for restriction of motion and compare bilaterally

treatment of AC joint- adduction ease treatment- ME!


Muscle energy - Adduction ease
Palpate AC joint
Place the pt's AC joint into restriction, abduction
Have patient contract against your forearm 3-5s
Post isometric relaxation
Reposition joint into further restriction, repeat 3-5 times
Recheck

motion testing for AC joint=To Palpate Internal/External Rotation:


To Palpate Internal/External Rotation:
=Palpate AC joint same as previous slide
=Bring upper extremity to 90
=Grasp wrist and externally rotate, while palpating for restriction at AC joint
Wrap arm under patient's arm and grasp wrist, internally rotate while palpating for
restriction at AC joint
Example:
Objective: Right AC Joint positive stair-step, abducted and externally rotated
Assessment: Somatic Dysfunction of the Upper Extremity

treatment of AC joint- internal rotation ease treatment- ME!


Muscle energy - Internally rotated AC
-Place the pt's AC joint into restriction, external rotation
-Have patient contract against your forearm 3-5s
Post isometric relaxation
-Reposition joint into further restriction, repeat 3-5 times
Recheck

Scapulothoracic Joint- position and motion


Position
Protracted vs Retracted
Superior vs Inferior
Rotated Upward vs Rotated Downward
Motion:
SAME
Palpate landmarks and compare bilaterally

Shoulder Complex:
Scapulothoracic Joint Position Supine
Place palm of hand on each anterior aspect of each shoulder
Gently compress both shoulders and assess for resistance
Side of resistance = side of shoulder protraction

Shoulder Complex:
Scapulothoracic Joint Protraction: Muscular Involvement in
protraction:
Anterior Element: Pectoralis Minor
Lateral Element: Serratus Anterior

Scapulothoracic Joint Protraction- Assess Pectoralis Minor AND


TREATMENT
To Assess Pectoralis Minor:
-Contact insertion of pectoralis minor at ribs 3-5
-Abduct ipsalateral upper extremity while palpating insertion of pectoralis minor
-Compare ROM of pectoralis minor bilaterally
Muscle Energy- Hypertonic Pec Minor
Abduct extremity until restriction (~120-130) is palpated in the pec minor
Have the patient adduct for 3-5s
Reposition into restriction and repeat 3-5 times
Recheck
Note: Can be performed seated or supine

To Assess Serratus Anterior:


To Assess Serratus Anterior:
-Patient is lateral recumbant
-Contact lateral edge of scapula with thumb and thenar eminence of caudad hand
-Cephaland hand grasps superior portion of scapula
-Test motion medially and superiorly

Test motions for: Scapulothrocic Joint Motions


Same contact as for testing serratus
Test motions for:
Protraction vs retraction
Superior glide vs inferior glide
Upward rotation vs downward rotation
EX:
Objective Findings: Hypertonic rhomboids with upwardly rotated scapula
Assessment: Somatic Dysfunction of the Upper Extremity

Diagnosis Review & Treatment


-Scapulothoracic Jt
Stretching/ME- Hypertonic serratus anterior
-Move scapula superior and slightly medial into restriction
-Have pt pull scapula inferior and hold for 3-5s
-Post isometric contraction reposition and repeat 3-5 times
-Recheck
Stretching/ME hypertonic levator scapulae or trapezius or rhomboids
Place restricted muscle into restriction and have the patient move towards ease,
hold 3-5s
Post isometric contraction reposition and repeat 3-5 times
Recheck

Glenohumeral Joint Position: and Motion


Position:
Anterior vs Posterior
Motion:
Voluntary:
Flexion vs Extension
Abduction vs Adduction
Internal vs External Rotation
Involuntary Motion: Slide/Glide with voluntary motion directions

Shoulder Complex:
Glenohumeral Joint Position
Position
-Patient seated with physician behind
-Thumb on acromion
-Middle finger on anterior aspect of humeral head
-Compare distance between thumb and middle finger bilaterally
Motion
Thumb on acromion
Index and middle finger on anterior aspect of humeral head

Squeeze thumb and fingers together


Compare resistance to glide bilaterally
Ex:
Objective Findings: Right Anterior humeral head preferring anterior glide
Assessment: Somatic Dysfunction of Upper Extremity

Elbow Complex: components


Humeroradial Joint
Humeroulnar Joint
Interosseous Membrane

Humeroradial Joint Position: and motion


Position:
Radial Head Anterior/Posterior
Motion
Radial Head Rotation on Humerus
Anterior with Supination
Posterior with Pronation

how to test position and motion


POSITION
1. Thumb anterior to proximal radial head

2. Index and middle finger on posterior aspect of proximal radial head


3. Compare Bilaterally
MOTION
Monitor at proximal radial head (as in previous slide)
With other hand, grasp patient's hand in the handshake position
Monitor proximal radial head while supinating and pronating the hand
Example:
Objective Findings: Proximal Radial Head Anterior
Assessment: Somatic Dysfunction of the Upper Extremity

Remember! with motion! radial head - supination and pronation


MOTION
Monitor at proximal radial head (as in previous slide)
With other hand, grasp patient's hand in the handshake position
Monitor proximal radial head while supinating and pronating the hand
Radial Head Posterior with Pronation
Radial Head Anterior with Supination

Review & Treatment


-Radial Head POST and ANTERIOR

Muscle Energy - Anterior Radial Head


Pronate the forearm to place radial head into restriction while palpating the radial
head
Have your patient supinate the forearm against your force
Post isometric contraction reposition and repeat 3-5 times
Recheck
Muscle Energy - Posterior Radial Head
Supinate the forearm to place radial head into restriction while palpating the radial
head
Have your patient pronate the forearm against your force
Post isometric contraction reposition and repeat 3-5 times
Recheck

Elbow Complex:
Humeroulnar Joint- position and motion- carrying angle
Position:
Abduction vs Adduction
Motion:
Major Motion: Flexion vs Extension
Minor Motion: Abduction vs Adduction

Humeroulnar Joint Position and motion

POSITION
Grasp Elbow with both hands
Evaluate for increased/decreased carrying angle and compare bilaterally
MOTION
Using both hands, grasp the proximal ulna
Place patient's wrist between physician's elbow and trunk
Add translatory force medially and laterally (adduction/abduction)
Compare Bilaterally
Example:
Objective Findings: Right Elbow increased carrying angle with right humeroulnar
joint abducted
Assessment: Somatic Dysfunction of Upper Extremity

Diagnosis: Posterior radial head, posterior translation radius,


increased carrying angle
Stabilize the humerus - done by patient position.
Make contact with the proximal radius - cephalad hand; caudad hand - hand shake.
Use the motions available across the joint to enhance ease.
Ant/post translation, supination/pronation, compression/traction.
'Stacking' sequentially; Use respiratory ease; Eventual 'Smooth Torsion Arc'

Interosseus Membrane

Important for supination/pronation at forearm


Acts like a shock absorber in trauma
To Test for Interosseous Membrane:
Palpate for tension bands in forearm proximally -->distally
Ex: Objective Findings: Forearm Interosseous Membrane - Increased Tension 2/3
distally
Assessment: Somatic Dysfunction of the Upper Extremity

Diagnosis: Fascial restriction of interosseous membrane


1. Stabilize the humerus - done by patient position.
2. Make contact with the proximal radius - prox. hand; distal hand - hand shake.
3. Use the motions available across the joint to enhance ease.
--Supination/pronation, Med/lat & Ant/post translation, flexion/extension,
compression/traction.
4. 'Stacking' sequentially; Use respiratory ease; Eventual 'Smooth Torsion Arc'

Wrist:
Carpal Bones and Metacarpals: Position and Motion
Position:
Superior vs Inferior
Internally vs Externally Rotated
Motion:
Evaluate for superior vs inferior glide
Evaluate for internal vs external rotation
Evaluate each carpal bone and metacarpal and compare bilaterally
Example:
Objective: Right Lunate Ventral
Assessment: Somatic Dysfunction of the Upper Extremity

Carpal Bones and Metacarpals treatment


Stabilize the proximal bone with proximal hand.
Make contact with the distal bone with opposite hand
Use the motions available across the joint to enhance ease.
Med/lat & Ant/post translation, flexion/extension, compression/traction.
'Stacking' sequentially; Use respiratory ease; Eventual 'Smooth Torsion Arc'

Diagnosis Review & Treatment


-Metacarpals/Phalanges
Balanced ligamentous tension(BLT), aka ligamentous articular release (LAR) - Any
dysfunction
Stabilize proximal joint
Disengage: compression or traction
Exaggerate ease
Balance (follow) ease
Can also use the phase of respiratory ease to adjust motions
Recheck
Naming?

Diagnosis Review & Treatment


-Carpal bones
-Articulatory with traction -Any carpal dysfunction
-Place hands perpendicular to pts, one on each side of their wrist
-Squeeze palms together and use clockwise and couterclockwise motion, or figureof-eight carrying dysfunction gently through the barrier(s)
-Recheck

Upper Extremity Diagnostic Sequencing:


Beginner's Approach SUMMARY
Start proximally and work distally:
Cervical, Thoracic, ribs (Dx & Tx as previously taught)
SC
AC
Scapulothoracic joint
GH
Elbow
Wrist / Hand
Depending upon the chief complaint, this can be a good clinical approach.
For beginning students, it is a good idea to use this as a protocol to gather all the
relevant diagnostic information before deciding on where to start the treatment
You rarely find only one dysfunction when scanning the extremity.
How do you decide where to start when there are two or more dysfunctions in the
upper extremity?
Severity?
What if one location is too 'hot' to move?

Screen the upper extremities 2 Tissue Texture


2 Motion Tests
Scan using Position and Motion comparing bilaterally:
1. Sternoclavicular 4. Elbow

2. Acromioclavicular 5. Wrist
3. Glenohumeral 6. Hand
Segmentally Definition
Based on motions at each unique joint
Name in ease of position and motion

golfer's elbow. tennis elbow


Medial epicondylitis
lateral epicondylitis

2 types of cervical articulations


Superior (suboccipital) segment
=C1(atlas) & C2(axis)
=Integrated unit
=Connected to each other and the occiput
="Atypical"
Inferior segment (C2-7)
=Inferior surface of C2(axis) to superior surface of T1
=Articulate at vertebral bodies with intervertebral disc
="Typical"

vertebral body shapes


Atlas
=No body, ring shape
Axis
=Odontoid process (dens)
C3-7
=Small
=Saddle Shaped

Spinous Processes:
Transverse Processes
Spinous Processes:
-Bifid
-C1-none
-C4- shortest
-C7- longest, nonbifid, moves with flex/ext
-T1 fixed
Transverse processes contain Foramen Transversarium
-Vertebral Vessels
-Groove for spinal nerves

Transverse Processes:
Anterior Tubercles sharp
Posterior Tubercles blunt
Ridge palpated along lateral neck

Articular pillars:
-Deep in facial groove
-Between semispinalis medially & cervical longissimus laterally
Size of examiners finger pad
Same level as spinous process for C2-C7

Intervertebral disks
=OA & AA
==No discs
==Weight- OA joint- axis- thru pedicles & laminae
=C3-7
==Wedge shape
==Thicker anterior
==Bodies articulate through intervertebral disc & 2 small ==uncovertebral joints
(Lushka) laterally-not synovial

Finger width Rules


=Spinous Processes
==Palpate C2
==With finger tips together rest in midline gap C3,4,5
=Transverse Processes
==Top finger on tip of mastoid process
==4 fingers cover C 2,3,4,5
==C6 wider/palpable
=Articular Pillars
==One finger width posterior to transverse process

Facet Joints C2-7


-Facet joint surfaces - approximately flat /oval planes
Superior facets- superior/medial
Inferior facets- inferior/lateral
-Facet joints support 1/2-2/3 weight of head
-F/E slides joints in sagittal plane- zygopophyzeal
-Greatest load bearing through posterior facets instead of vertebral bodies

Facet Joints C2-7


-Planes of facets not parallel
--Face backward and upward at 45 degree angle
--Meet near tip of SP of C7
-Angles of planes increase upward
--10-60 degrees-avg. incline of 45 degrees

C3-7 Uncovertebral Joints


-Uncinate process- lateral ridge (lip) along superior surface of bodies of C3-7
-Posterolateral corner vertebral bodies
-Function in gliding movements
==F/E-guide vertebral body motion
==R and SB-Limit lateral translation
-Protect disc from posterolateral herniation*
-Osteoarthritis

C3-7 Uncovertebral Joints


Uncinate process- lateral ridge (lip) along superior surface of bodies of C3-7
Posterolateral corner vertebral bodies
Function in gliding movements
-F/E-guide vertebral body motion
-R and SB-Limit lateral translation
Protect disc from posterolateral herniation

Osteoarthritis
side: Osteoarthritis-lipping- enchroaches on anterior aspect of lateral intervertebral
canal and affect spinal nerves

Atlanto-Axial Joint--Three Joints


One medial
Anterior arch of C1
Dens of C2
Two Lateral
Inferior articular facets of C1 -convex
Superior articular facets of C2-convex

A-A Motion!! motions*


-NO JOINT
- 1/2 of cervical range of motion
-Total rotation ROM- 70 degrees- 35 each side
==Hyperflexion locks out facets of C2-7 & OA- only AA rotates*
==Can get 45 degrees if other cervicals not fully locked out
-Some F/E- 17 degrees
==F=forward & down E= backward & down
==Small amount side bending-involuntary wobble effect
==Anterior arch slides superior/inferior on dens
==Facets always engaged-No physiologic neutral

AA Joint**
Odontoid process (dens)
-Articulates on small synovial joint- post. surface of ant. arch of atlas- behind ant.
tubercle
-Transverse odontoid ligament QUIZ ??*
-Post. surface - kyphotic convexity- allows ligament to slide- permits forward
translation of atlas

Ligaments
-Anterior longitudinal- anterior body -->limits extension
-Posterior longitudinal- posterior body-->limits flexion
-Ligamentum flavum- posterior wall vertebral canal
connects laminae
-Ligamentum nuchae- posterior neck

Ligaments of A-A joint + function


Cruciform Ligament
=prevent anterior displacement C1 on C2
=Transverse-Lateral masses atlas over dens
=Longitudinal-Dens to ant. foramen magnum & axis body
Apical Ligament-Apex dens to ant. Foramen magnum
Alar Ligament-Apex dens to tubercle occipital condyle
=Extension-relaxed
=Flexion-taut

Vertebral Artery
=Enters C-spine between TP of C6-7
=Exits superior to TP of C1
=Turns posterior over posterior arch of atlas C1
=Enters foramen magnum
=Forms Basilar Artery
=Bony protection of Artery
=Extension can kink artery
-Foraminal passage for vertebral artery decreased by as much as 35% when cervical
column is extended
-Can narrow 90% on contralateral side with rotation
-Combining hyperextension with rotation can occlude the vertebral artery where it
passes through the OA membrane

***Caveat on overextension
Avoid overextension when palpating, diagnosing and manipulating the cervical
spine
Caution - Down Syndrome, RA, agenesis odontoid process, fracture of odontoid

Autonomics 3!!
Cervical Chain Ganglia
=Sympathetic control to the head and neck
C2-Small branch connects to Vagus
=Internal visceral disease
C3-5-Phrenic- can treat hic-ups with this (inhibition + =pressure= 80% hiccup
survival rate lol)
Diaphragm Dysfunction

Fluid
Lymphatics
=Deep Cervical Nodes
=Extensive lymphatic drainage in the cervical region
Venous drainage
=Jugular Vein
Fascial constrictions influence drainage
=Thoracic aperture
=Abdominal diaphragm

Mobilization with Impulse


-Make an accurate structural diagnosis
-Engage resistance in as many planes of motion as necessary for appropriate
localization
-The impulse force is vectoral.
-Direction - into at least one restriction (Fryette's 3rd Principle),
-Duration & amplitude- split second, less than 1/8 - 1/4 inch of motion
-Patient relaxation is key
-Deliver the vectoral impulse as the patient exhales. This allows for a minimum of
force.
-Recheck

Contraindications
Absolute
=Osteoporosis/Osteomyelitis/Fracture
=Rheumatoid Arthritis/Downs
=Weakness transverse ligament
Relative
=Acute Whiplash
=Pregnancy
=Post OP
=Herniated Disk
=Anticoagulants
=Vertebral Artery Ischemia
Rare occurrence!
=Neck strain
=Cervical disc herniation
=Vertebrobasilar accident
===Spontaneous occurrence has nearly twice the risk when compared to cervical
manipulation**

=Neurovascular accidents
=Aggravation of disc problem
=Fractures
=Vertigo

BENEFITS!
Is safe and effective
Is comfortable and relieving for the patient
Is simple and easy to do
Is quick
Is taught safely with supervision
Is encouraged
Acknowledged Benefits:
Relief of acute neck pain
Reduction subacute or chronic neck pain
Short-term relief of tension headache
Relief of cervicogenic headache
Relief of acute migraine symptoms
Potential Harm:
Vertebrobasilar accident
Natural spontaneous occurrence rate nearly twice that associated with cervical
HVLA

Avoiding Complications
Avoid overextension
Diagnose accurately
Don't force beyond tolerance
"Feather-Edge"
Re-evaluate diagnosis and treatment method

Determinates of Motion- fryette's 3rd


Fryette's Third Principle
Initiation of motion of a vertebral segment in any plane of motion will modify the
movement of that segment in other planes of motion

C2-7 Motion**** dysfunction


Facet surfaces always in contact unless traction applied
No physiologic neutral
Facets always engaged
Only non-neutral somatic dysfunction
=Always "Type II like"
=No group somatic dysfunction
Sidebending and rotation to same side

99% of the time no NEUTRAL SEGMENTS


- Always flexed or extended and FRS (R and S in same direction)

C2-7 Motion
C2-7 Terminology
Treating C6 is actually C6 motion on C7 for HVLA
Flexion
-Inferior facets slide superior/anterior on superior facets of vertebra below
-Anterior translation of body
-Intervertebral space compressed anterior
-Gapping-tips of SPs
Extension
-Inferior facets slide inferior/posterior on superior facets below
-Posterior translation of body
-Intervertebral space compressed posterior
-Closing-tips of SPs

Restricted Facets
=Vertebral motion restriction relates to restriction at one of the facet joints of the
vertebral segments
=Restriction in motion is due to a disruption in the capacity of the facet joints to
glide freely

Normal Facet Motion


Normally in Flexion (forward bending), facets glide anterior and superior = OPEN
Normally in Extension (backward bending), facets glide posterior and inferior =
CLOSED

Acronym: FOES**
Flexed-Opposite-Extended-Same
If the diagnosis is Flexed, the restricted facet (open) is on the side Opposite the
sidebending/rotation
If the diagnosis is Extended, the restricted facet (closed) is on the Same side as the
sidebending/rotation

Lateral Translation
Test for SB mobility
Passively move superior vertebrae
Translation = Opposite SB

Lateral Translation
Translate L and R
-Monitor distance- compare sides
-Repeat in flexion and extension
-Repeat at each cervical level

Rotation Vs. Sidebending


Both extremely useful
Some people prefer one over the other
Usually combination of both
Rotation:
Place segment into ROTATIONAL restriction
Sidebending:
Place segment into SIDEBENDING restriction

C2-3 HVLA-Rotation
-Stand at head of table
-Support head with hands
-Index fingers on B/L articular pillars
-Slightly sidebend segment towards EASE
-Rotate to RESTRICTION
-Flex to segment *
-Slightly extend at & above segment*
-Quick thrust thru both hands with rotational movement, support hand does not

thrust
-Recheck

C2-7 HVLA-Side Bending


-Pt supine on table with doc standing at head of table
Support head on side of ease with palm of hand (broad contact)
-Contact articular pillar of segment on side of SB restriction with MCP joint of index
finger
-SB to restriction over MCP and translate to restriction
-Flex or extend to segment
-Rotate head to side of ease to lock out segment above
-Adjust F/E, SB, and rotation to localize
-Translational thrust through articular pillar with index finger
-Recheck

Alternate HAnd Hold for side-bending


Alternative Hand Hold
-Support head on side of ease with forearm-support chin in hand
-Do NOT thrust through chin. This is just to add extra support
Other Tips
Thrust should come from pectoral muscles (Robot Arm!)
Remember Fryette's Principles! Don't use more than 45 degrees of rotation

A-A Rotation Test C1 is pure rotation


-Rotation with Maximum flexion!!
-Pt. supine-stand at head of table
-Cradle occiput in both hands
-Flex to lock out C3-7
-Rotate to both sides-compare
Possible flexion and extension

AA HVLA-Rotation
-Pt. supine-doc standing at head of table
-Cradle head in hands
-Contact right posterior arch of atlas with MCP joint of index finger on side of
rotation
-Flex head to about 30-45 degrees to take out motion of the other vertebrae
-Rotate to restriction
-Slightly adjust SB & F/E as needed for localization
-Thrust-rotational
==forearm aligned with vector of force
==generated through forearm into wrist and MCP
-The support hand does not participate in force generation during the thrust
-Recheck

O-A Lateral Translation- with testing


-Passive movement
-Lateral translation of head while remaining parallel to sagittal plane of trunk
-Quality of motion
-Observe distance from midline in flex, ext, and neutral
-Normal translation of the head = 3-4" per side
1" trans.=arc of 1-2 degrees of opp. SB

-Patient supine-head at end of table


-Doc stand at head of table- feet apart, knees bent, elbows level with pt. head, hands
close to body
-Support head with finger pads of both hands, under center of gravity
-Translate head keeping it parallel to trunk- move head by shifting body side to side
-Estimate amount of translation=Translation = opposite side bending
-Test in neutral and varied degrees of flexion and extension
-Describe findings in restriction then name diagnosis by position (ease).
O-A Lateral Translation Test: Translation Resistance Left or Right with the Head in
Flexion or Extension => Diagnosis (FSleftRright, ESleftRright, etc.)
Example: ESRRL (ESRleft)

Cervical Stretching
Linear Traction/Stretch
-Lower cervical muscles
-Posterior cervical muscles
Perpendicular Stretch
Bilateral Stretch

OA: Flexed side bent right rotated left


Treatment:
Left Hand contacts left occiput at AO articulation
Right Hand cradles the head and neck.
Goal: Carry the left occiput forward on the atlas - toward the opposite orbit
Patient Relaxation:
'Take a deep breath in, then let it out slowly.'
'Let your head drop toward the table.'
Dx: FSrRl

OA: Flexed side bent right rotated left


Goal: Carry the left occiput forward on the atlas
Force must be directed:
-Superior to disengage the facet joint
-Medial & Anterior to follow the plane of the facets
The Resulting Vector is Toward the Opposite Orbit
Dx: FSrRl

OA: Extended side bent right rotated left


Left Hand contacts left occiput at the AO articulation
Right Hand cradles the head and neck.
Goal: Carry the right occiput backward on the atlas - toward the opposite orbit
-Sidebend Left Rotate Right into resistance
Patient Relaxation:
='Take a deep breath in, then let it out slowly.'
='Let your head drop toward the table.'
Dx: ESrRl

OA: Extended side bent right rotated left


Goal: Carry the right occiput backward on the atlas (stuck facet)
Force must be directed:
-Superior to disengage the facet joint
-Medial & Anterior to follow the plane of the facets

-Toward the Opposite Orbit


Notice the Subtle Change in Vector & Localization toward the right facet joint
Dx: ESrRl

Practice HVLA- cervical, thoracic, lumbar


Important points1. motion of C2 is primarily rotation-HVLA here is rotation into resistance
2. cervicals C2-7 behave Fryette like- Type 2 mechanics- rotation and side bending
opposite directions (NO WRONG SAME)
3. correct upper extremity position for thoracic HVLA
4. Flexed and extended thrust direction
5. Practice ! Contact your friendly local OMM fellow for assistance

Cool Down: Post-Treatment


Soft Tissue
Linear Traction/Stretch
Lower cervical muscles
Posterior cervical muscles
Perpendicular Stretch
Bilateral Stretch
OR
Other treatment modalities
Functional Methods
FPR
Muscle Energy

Contraindications hvla
Mobilization with Impulse
Absolute
Osteoporosis/Osteomyelitis/Fracture
Rheumatoid Arthritis/Downs
Weakness transverse ligament
Down syndrome, RA, agenesis odontoid process, fx odontoid,
Relative
Acute Whiplash
Pregnancy
Post OP
Herniated Nucleus Propulsus
Anticoagulants
Vertebral Artery Ischemia
congenital asymmetry, atresia

Autonomics
Superior cervical ganglion
Sympathetic control cervical blood flow

Bound by deep connective fascia


Receives preganglionic fibers of T1-4
Vagus
Parasympathetic innervation to most internal organs
Jugular foramen: exit of CN 9, 10, and 11

Lymphatics
-Extensive lymphatic drainage in the cervical region - Deep Cervical Nodes in
relationship to OA & AA
Fascial constrictions influence drainage
Thoracic aperture
Abdominal diaphragm

Spinal Dura Mater - Attached to:


Circumference of Foramen Magnum
Posterior Surfaces of the Bodies of:
C2
C3

OA Joint
Cranium sits on atlas like a sphere
Main motion is Flexion/Extention
15 degrees
Sidebending and rotation to opposite sides
Lateral atlanto-occipital ligament

OA Motion Testing
Nodding test
-Pt. moves head into F/E

-Observe for lateral deviation of chin


Patient supine (Active Motion Test)
-Doc standing at head of table
-Patient nods head without lifting it off table
-Flex- tuck chin- condyles slide back
-Ext- backward nod-chin anterior- condyles slide forward
-Observe for lateral deviation of chin in flexion or extension
Lateral translation
-Doc at head of table, hands on patient's head, elbows level with patient's head
-Translate head by shifting body side to side
==Remember sidebending and translation are opposite
-Test in neutral and with small degrees of F/E

Determining the "Stuck" Facet


As the occiput flexes, the condyles move posteriorly
As the occiput extends, the condyles move anteriorly
Prefers flexion-resists extension
-condyle cannot move back
-occiput rotates away from stuck facet
Prefers extension-resists flexion
-condyle cannot move forward
-occiput rotates toward stuck facet

Fascia
-Prevertebral fascia contacts the cranial base
-Investing layer of cervical fascia is continuous with periosteum of hyoid & sternum
-Pretracheal fascia fuses with fibrous pericardium

OA joint
YES Joint- **OA: Sidebending & Rotation occur to opposite sides
-Flexion/Extension- Transverse axis
-Principle motion
-15 degrees motion between extreme flex and extreme ext- AA contributes
additional 17 degrees
-Side Bending- AP axis
-Limited to 6 degrees side to side
-Rotation-Vertical axis
-Pure rotation nearly impossible
-Sidebending & Rotation are Coupled Motions

motions OA joint
OA extension requires that the occipital condyles move anterior on the atlas.
OA flexion requires that the occipital condyles move posterior on the atlas.

O-A Supine Nodding Test*


Chin deviated to 1 side
-Occiput rotated to that side
-Sidebent to opposite side
Unilateral flexion
-Rotation to same side
-Sidebent opposite
Unilateral extension
-Rotation opposite
-Sidebent to same side

OA Alternative Tests
Intersegmental Motion testing
Hands supporting occiput with index fingers in occipital sulcus lateral to midline.
Motion is induced while fingers are monitoring response.
One hand supporting occiput with index finger and thumb of other hand stabilizing
and monitoring motion at occipital sulcus lateral to midline. Monitor response
while motion testing
*Side of shallow sulcus is the side of sidebending preference
*Flexed or Extended position: Whichever the sulci are most even is the direction of
ease

OA Incremental Mobilization I
-Use the cephalad hand to gently cup the occiput
-Use the thumb and index (or middle finger) to contact the lateral masses of C1; Add
flexion or extension according to your diagnosis.
-Cephalad Hand: add cephalad traction down into the OA facet articulations (to
disengage the facets)
-Caudad Hand: introduce rhythmic translatory motion against the resistance
previously diagnosed
=Visualize the angle of the facets at that level
=Visualize the vector of force going to the restricted facet

OA Incremental Mobilization II
Same Hand Contact for Both Hands
Cephalad Hand: add cephalad traction down into the OA facet articulations (to
disengage the facets)
Caudad Hand: holds the atlas motionless
=Visualize the angle of the facets at that level
Cephalad Hand: Add Translatory Motion rhythmically down to the restricted facet
=Visualize the vector of force going to the restricted facet

which condyle is stuck? FLEXION


rotation is to the side of the freely moving condyle

which condyle stuck EXTENSION?


With restriction in extension, rotation is to the side of the stuck condyle

Anatomy POSTERIOR
Posterior
Occiputal Protuberance
Posterior Arch C1
Spinous Processes C2-7

Anatomy: LATERAL
Lateral
Mastoid Process
Transverse Process
Articular Pillar

Vertebral distance
Finger width
Finger width

Translating rule + testing


Translation Testing
Translation towards the R = Sidebending L
Palpate for restriction
Resists translation to the L -->
Sidebent R reistance -->
Sidebent L ease
Place index fingers on articular pillars
Translate L and R - feel for restriction
Repeat in flexion and extension
Remember, lesions will translate more symmetrically in their (flexion/extension)
position of ease
Practice this while standing too!

Rotation Testing
Patient supine
-Doc seated at head of table
-Cup head in hands, index fingers monitor articular pillars
-Rotate head to determine the rotation ease
-Flex head to level of dysfunction
=If the rotation continues to be asymmetrical then flexion is the restricted motion
-Extend to level of dysfunction
=If the rotation is more symmetrical, there is ease with extension

HVLA prep cervical


Linear Traction/Stretch
Lower cervical muscles
Posterior cervical muscles

Perpendicular Stretch- HVLA Preparation


- Cervical Stretching

HVLA Preparation
- Cervical Stretching-Bilateral Stretch

C2-7 Muscle Energy


Patient supine/Doc at head of table
Support occiput in palm
Index finger over articular pillar of segment, on side of SB resistance
Flex or extend to restriction at segment using opposite hand
Introduce side bending to restriction by translating the segment to restriction
Rotate to restriction
Instruct patient to move head into one position of ease (sidebending) against
counterforce
Hold 3-5 sec.
Relax, Reset, Repeat, Recheck

Incremental Mobilization

Use one hand to gently cup the occiput


Hold cervical spine in neutral
Use the thumb and index (or middle finger) of other hand to contact the articular
pillars; Add flexion or extension to resistance
Add cephalad traction (to disengage the facets)
For C2-7: Introduce rhythmic translatory motion in the direction of resistance
For AA: Introduce rhythmic rotatory motion left & right

Examples:
With restriction in flexion, rotation is to the side of the freely moving condyle
With restriction in extension, rotation is to the side of the stuck condyle
Flexing- condyles backward
Chin left
Left rotation
Right sidebending
L condyle moves back
R condyle stuck
ESrightRleft
Chin right
Right rotation
Left sidebending
R condyle moves back
L condyle stuck
ESleftRright
Cannot translate L
Cannot sidebend R
Cannot rotate L
L condyle stuck anterior
R condyle moves posterior
Free movements
-Extension, Left sidebending, Right rotation
Diagnosis- ESleftRright
Cannot translate R
Cannot sidebend L
Cannot rotate R
R condyle stuck anterior
L condyle moves posterior
Free movements
-Extension, Right sidebending, Left rotation
Diagnosis- ESrightRleft
Extending- condyles forward
Chin left
Left rotation

Right sidebending
Right condyle moves forward
Left condyle stuck
FSrightRleft
Chin right
Right rotation
Left sidebending
Left condyle moves forward
Right condyle stuck
FSleftRright
Cannot translate L
Cannot sidebend R
Cannot rotate L
R condyle stuck posterior
L condyle moves anterior
Cannot translate R
Cannot sidebend L
Cannot rotate R
L condyle stuck posterior
R condyle moves anterior
Free movements
Flexion, Right sidebending, Left rotation
Diagnosis- FSrightRleft
Free movements
Flexion, Left sidebending, Right rotation
Diagnosis- FSleftRright

OA ME
Introduce side bending to restriction by translating the segment to restriction
Rotate to restriction
Instruct patient to move head into one position of ease ( F/E, R, or SB) against
counterforce
Hold 3-5 sec.
Relax, Reset, Repeat, Recheck

OA: Left Condyle Resists


Occipital Extension
Dx: FSRleft or F S right R left
Treatment:
Left Hand contacts occiput near left condyle
Right Hand cradles the head and neck.
Goal: Carry the left occipital condyle forward on the atlas
Patient Relaxation:
'Take a deep breath in, then let it out slowly.'
'Let your head drop toward the table.'

OA: Right Condyle Resists


Occipital Flexion
Dx: ESRleft or E S right R left
Treatment:
Left Hand contacts occiput near left condyle
Right Hand cradles the head and neck.
Goal: Carry the right occipital condyle backward on the atlas
Sidebend Left Rotate Right into resistance
Patient Relaxation: breath and relax
Force must be directed:
-Superior to disengage the facet joint
-Medial & Anterior to follow the plane of the facets
-Toward the Opposite Orbit
Notice the Subtle Change in Vector & Localization toward the right facet joint

Post-Treatment Patient Assistance


Soft Tissue
Linear Traction/Stretch
Lower cervical muscles
Posterior cervical muscles
Perpendicular Stretch
Bilateral Stretch
OR

Indirect - Helps integrate changes


Functional Methods
Ligamentous Articular Release

OA JUNK test and treat


1. O-A Lateral Translation Test: Translation Resistance Left or Right with the Head
in Flexion or Extension => Diagnosis
Resists Translation from the left toward the right while the head is flexed.Therefore
it resists Left sidebending
Sidebent Right at the OA requires Rotation Left.
Therefore the diagnosis is: ESRRL (ESRleft)
Step 2: Direction of Condylar Motion with Head in Position of Translation
Resistance
-Flexion is the Head Position in which Translation Resistance Was Encountered
-Condyles must be able to move posterior for Head Flexion to occur
Step 3: Direction of Condylar Motion with Head in Position of Translation
Resistance
In Head Flexed Position the condyles are moving posterior.
The Left Condyle must move posterior for Left Rotation to occur.
Therefore, the Right joint is the restricted joint. It is stuck anterior.
Treatment:
Goal: Carry the left occipital condyle forward on the atlas
Force must be directed:
-Superior to disengage the facet joint
-Medial & Anterior to follow the plane of the facets
The Resulting Vector is Toward the Opposite Orbit**
Treatment Option II:
Goal: Carry the right facet of the atlas forward under the occiput
Caudad Hand Thrusts: on Right Lateral Mass of the Atlas
Treatment Option II:
Occipital Hand Disengages & Localizes:
Superior to disengage the facet joint
Flex to localize
Sidebend Right to stabilize occiput (Fryette's 3rd Law)
Force must be directed:
Atlas Hand: Medial & Anterior to follow the plane of the facets and rotate the atlas
left

Stretches summary
TIPS!
If the neck is hypertonic do something else first
Finding the Barrier
Don't lose barrier when pt takes a breath
Patient relaxation!
Engage resistance slowly and actually feel what you need to
Locking out the segment
Thrust

...
Neural
Sympathetic
Parasympathetic
Fluids
Lymphatic
Venous
Biomechanical
Visceral
Pain

...
Sympathetic
Rib Raising

...
Neural Treatments
Parasympathetic
Suboccipital
Cranial
Sacral

Fluid Treatments
Thoracic Inlet
Anterior cervical fascia
1st rib
Abdominal diaphragm
Pelvic diaphragm
Soft tissue effleurage

biomechanic treatment
Treatment of associated regional somatic dysfunction
Treatment of distal but associated somatic dysfunction

Pain Treatments
Treatment of areas directly involved with pain may be indirectly involved in any of
the other case considerations

Visceral treatments
Treatment of affected viscera
abdomen

Three regions
three treatments
three minutes

Bronze level treatment


Sympathetic
Parasympathetic
Venous/Lymphatic

From a chief complaint


- case considerations
Findings
Systems

chief complaint web

pneumonia web
Rib 3: Biomechanical influence on the function of the pulmonary tree
May impact sympathetic chain at region related to lung

Case Consideration in the UG system


Sympathetic T10-L2
T10-11 Kidney, Adrenal, Gonads
T12-L2 Bladder, Prostate
Parasympathetic
Vagus to Kidneys
Sacral Plexus to Bladder, Prostate
Fluids
Pelvic diaphragm and abdominal diaphragm
Vena cava
Cisterna chyli
Thoracic inlet
Biomechanical
Rectus abdominus tender points
Innominate dysfunction
Sacral Dysfunction
Pelvic diaphragm
Pain
Iliolumbar ligament
Trigger points
Genitofemoral nerve
Psychosocial
Incontinence
BPH
Chronic UTI

Sympathetic
T10-11 Kidney, Adrenal, Gonads
T12-L2 Bladder, Prostate
Remember diaphragm and its role as thoracolumbar modulator
History of LE injury, potentially activating the LE sympathetic levels
Low back strains, especially long-standing

Parasympathetic
Vagus to Kidneys
Sacral Plexus to Bladder, Prostate
Background cervical dysfunction
Background low back pain, sacral dysfunction

Fluid
Pelvic diaphragm and abdominal diaphragm
Vena cava
Cisterna chyli
Thoracic inlet
Diaphragmatic dysfunction
Underlying innominate dysfunction
First rib
Thoracic inlet

chapman's reflexes
Predictable anterior and posterior fascial tissue texture abnormalities assumed to be
reflections of visceral disease*
Viscero-somatic reflex

chapman's reflexes
Predictable anterior and posterior fascial tissue texture abnormalities assumed to be
reflections of visceral disease*
Viscero-somatic reflex

BiomechanicalRectus abdominus tender points


Innominate dysfunction
Sacral Dysfunction
Pelvic diaphragm
Psoas Tension
Diaphragm tension
Lumbar dysfunction
Sacral and pelvic influence on pelvic diaphragm
Lower extremity influence on sacrum/pelvis

Pain- trigger points


Iliolumbar ligament
Trigger points
Genitofemoral nerve
Consider referred pain from myofascial tissues
Consider tender points
Maybe chief complaint related

referred iliolumbar pain

Visceral
Kidney ptosis
Dysfunction of ureter
Dysfunction of bladder

Real Patient Lab SOAP


Subjective: OLD CART organization with Unique historical elements related to Msk
system
Objective: succinct MSK findings
For this lab:
Screen, Scan, Segmental Definition Findings for the region you evaluate and treat
Assessment: 1) Medical Diagnosis, 2) Regions of Somatic Dysfunction
Plan: 1) Diagnostics, 2) OMT, 3)Medications, 4) Pt. Ed., 5) etc.

SOAP overview
S: Complaint: Cough/Cold with rib pain
Onset? Duration? Local or radiating? Sore Throat?
Mucous production? [OLD CART]
O: Postnasal drainage, oropharynx red, tonsils sl. enlarged;Lungs CTAB; Heart
RRRwithout murmur
Screen positive for thoracic region;
Scan and Seg. Def. T4 FRSr
A: 1) Upper Respiratory Infection , 2) Rib Pain, 3)Somatic Dysfunction of the
thoracic region
P: 1) OMT: FPR to thoracic dysfunction; improved symmetry of motion and patient
comfort after the treatment, 2) Medication: ..., 3) Pt. Education

case considerations
OMM Case Considerations [1 point extra credit]:
Biomechanical: thoracic dysfunction affects rib function and rib pain
Nervous:
Sympathetic: T4 is at the level for innervation to the lungs

Parasympathetic: No related [Since you only have one objective finding that does
not relate to this; But if cervical were your regions this would be filled out instead of
Sympathetic
Pain: T4 related afferent fibers may be mediating part of the pain
Fluid: the biomechanical influence on ribs could decrease respiratory effectiveness
and thus fluid drainage.
Visceral: Next year

Overview
Subjective: History taking
10 minutes:
Chief Complaint
History of Present Illness
Pertinent Review of Systems (if unsure, ask faculty/fellows/TAs
Major Diagnoses - Medical/Surgical
Decide which sequence is applicable to this person's complaint
Objective - Physical exam
Screen (10 minutes): Do a focused screen. Think of regions anatomically related to
the chief complaint.
Screen the regions that are part of the sequence you chose:
Upper half of the body
Lower half of the body
Scan: Each student scans region screened
Mark identified segments with 'red dots'
Look for segmental tissue texture changes: bogginess, fibrotic, ropey, cool/hot, etc.
Record and communicate scan results in the objective section
For Extremities - Use what you have learned so far
UE: scapulothoracic eval. & tx
LE: hip restrictor eval. & tx
Segmental Definition:
Define Rotational aspects: Rotation Sidebending Flexion/Extension- Define
Translational aspects:
A-P, Left-Right, Ceph-Caud - Respiratory preference (ribs): Inhalation/Exhalation
Relate the findings on your patient to comorbidities - CV, Resp, GI/GU, etc.
Fluid congestion
Sympathetic facilitation
Parasympathetic imbalance
Pain

Assessment
Assessment:
1. Medical Diagnosis(es)
2. Somatic Dysfunction
Designate Region that you diagnosed that demonstrated evidence of somatic

dysfunction as indicated by the 'objective findings'


Example:
S: low back pain
O: Lower lumbar soft tissue resistance increased and resists rotation right;
A: 1)low back pain (or L-S Strain)
2) Somatic dysfunction of the lumbar region

Plan:
1. Diagnostics: Lab, Imaging, etc.
2. OMT:
Consider sequencing of treatment Which Somatic Dysfunction should be treated
first and why?
1. Designate & record method(s) used in each region. It is not necessary to repeat
the information already recorded in the Objective section!
2.Note patient response to overall treatment
3.Medication, if indicated
4. Pt. Education- Things to avoid- Things to do
5. Referral back to their physician for further Evaluation & Treatment?
Etc.
6. etc.

Types of Somatic Dysfunction


remember yellow?

Lymphatics summary
Lymphatics are the 'Overflow system' of the body.
Local drainage or lack thereof is only one component in the equation of fluid build
up.
Key Principle: Any blockage Proximal to the excess fluid will inhibit its drainage.

How is an osteopathic diagnosis different from an allopathic diagnosis


of the lower extremity?
Motions
=Gross vs. fine
Relationships
=Interrelatedness of subtle dysfunction on rest of functioning organism

Diagnosis & Treatment:


Upper Half of the Body
Anterior Ribs
Thoracolumbar Junction
Cervicothoracic Junction
Thoracics / Other Ribs
Scapulothoracic Articulation
Craniocervical Junction / Cranium
Cervicals
Rest of Upper Extremity

Lower Half of the Body Diagnosis and Treatment Sequence


1. Hip restrictors (some of which cross the Lumbo-sacral junction)
2. Pubic bone dysfunction
3. Superior innominate shear ("innom. upslip")
4. Lumbar dysfunction (esp. L5 or, less often L4)
5. Sacral dysfunction
6. Innominate dysfunctions (other than upslip)
7. Iliopsoas (including thoraco-lumbar junction, approx. T11-L2)
8. Other Lower Extremity Dysfunction

Most Common SD
Fibular head (ant/post)
Tibial rotation (int/ext)
Talus (ant/post)
Calcaneal (inversion/eversion)
Dropped cuboid/navicular
Metatarsal dysfunction

Common Fibular n.
=Wraps around posterior part of fibula
=Sensitive to trauma, fracture or posterior fibular head dysfunction
=Injury at knee can produce weakness of dorsiflexors and foot drop (supplies
(motor)
==anterior--deep fibular== lateralsuperficial fibular-compartment of the leg, sensation to anterolateral
leg)

Tibial n.
Posterior compartment of the leg
Muscles of the foot

Sympathetic NN.

= Pregangionic fibers arise from T11-L2


= Shared with innervation of pelvic organs via least splanchnic n. and lumbar
splanchnic n.
= May explain Chapman's points along ITB for colon problems
= L1-3 dermatomal pattern anterior thigh

Lymphatic Drainage- Superficial


=Abdominal wall below umbilicus
=Gluteal region below iliac crest
=Perineum (part of anal canal, superficial 1/3 of penis, vulva, vagina)
=Superficial part of thighs, legs, feet
=Drainage into superficial nodes in inguinal and femoral triangle thendeep to
nodes and channels around femoral vein in upper thigh

Lymphatic Drainage- Deep


=Parallel pathways along iliac veins to cysterna chyli
= Drains to thoracic duct (left lymphatic duct)
= Receives drainage from left side of head and neck, posterior and upper lobe of left
lung.

Functional Anatomy: Knee--Articular- femur and tibia


Double condylar complex synovial joint formed by femoral condyles and tibial
plateau
Medial and lateral semilunar cartilage (menisci) provide stability, smoothness and
resistance to pressure
Medial condyle longer than lateral. Provides for
=posterolateral glide with internal rotation
=anteromedial glide with external rotation

femur and patella joint


=Gliding joint
=Tendon of quadriceps femoris incorporates patella and attaches to tibial tuberosity
Patellar tendon reflex at L4

Articular- medial and lateral collateral ligaments


Lateral
Stabilization against lateral displacement
Not attached to lateral meniscus
Medial
Stabilization against medial displacement
Attached to medial meniscus
Makes medial cartilage more susceptible to medial displacement or twisting

Functional Anatomy: Knee--Articular cruciate ligaments


Anterior attaches to anterior tibia
Prevents excessive anterior tibial glide
Posterior to posterior tibia
Prevents excessive posterior tibial glide

Somatic dysfunction of knee


minor motions:
-anterior posterior glide
-medial/lateral glide
-anteromedial or posterolateral glide (associated with int/ext rotation)

Proximal Tibiofibular Joint


= Separate synovial joint at the knee
= Oblique angulation- lateral anterior to medial posterior position*
=Motions at proximal and distal fibula are reciprocal*
=Dorsiflexion - foot- moves distal fibula posteriorly, proximal fibula glides anterior
=Opposite with plantar flexion (fibular head posterior)

Ankle--Articular
- Distal tibia and talus
- Ankle mortice includes distal fibula (lateral malleolus)
- Transverse axis of ankle joint not parallel to transverse axis of body, foot is
normally in slight abduction
No pure supination or pronation of the foot

ankle motions
Supination Equivalent
Inversion
Plantar Flexion
Adduction
Pronation Equivalent
Eversion
Dorsiflexion
Abduction

Lateral Ankle Ligaments**


Anterior talofibular ligment
Posterior talofibular ligament
Calcaneofibular ligament

Generally disrupted in ankle inversion or supination sprain


Grade I --anterior talofibular
Grade II -add calcaneofibular
Grade III- all lateral ligments torn

Motions of the foot


Abduction
External rotation of the tibia
Anteromedial glide at the knee
Adduction
Internal rotation of the tibia
Posteorlateral glide at the knee

Functional Anatomy: Foot--Muscles and Tendons - Myofascialposterior and lateral maleolus


Achilles tendon- attaches to posterior calcaneus from common origin of
gastrocnemius and soleus mm.
Posterior to Medial Malleolus
Posterior Tibial Tendon
Flexor Digitorum Longus
Flexor Hallicicus Longus
"Tom, Dick, and Harry"

Foot Anterior Tendons


Anterior Tendons
Anterior Tibialis Tendon- Supports longitudinal arch
Extensor Digitorum Longus
Extensor Hallicus Longus
"Tom, Dick, and Harry"

Tendons Posterior to Lateral Malleolus


Fibularis Brevis
Fibularis Longus- Supports transverse arch

Longitudinal Arches
Lateral
Calcaneus, cuboid, and metatarsals 4 and 5
Medial
Talus, navicular, three cuneiforms, metatarsals 1-3

Transverse Arch- whats the muscular support. what bones make up


this arch
Cuboid, navicular, three cuneiforms, and metatarsal bones
Muscular Support--Peroneus (fibularis) longus inferiorly, and tibialis anterior (first
cuneiform and first metatarsal

Plantar Ligament Support


Plantar Aponeurosis
Long plantar ligament
Short plantar ligament
Spring ligament

Motions of the foot


Forefoot inversion (supination strain)
Talus glides posterolaterally at talocalcaneal joint relative to the navicular
Forefoot eversion
Talus glides anteromedially

Scan (Knee)
Orthopedic Testing
-Medial and lateral collateral ligament
-Patellar grind
-Drawer test
-Lachman's sign

Motions: KNEE and a bunch of other

KNEE
=Lateral/Medial
=Anterior/Posterior
=Anteromedial (EXT rot)/Posterolateral (INT rot)
Fibular head motion
Lateral Malleolar Motion
Strain to Interosseous Membrane
Ankle Motion
-Talotibial motion
-Subtalar motion
-Intertarsal motion

Foot motion
Navicular bone dysfunction
Plantar glide and medial rotation (inversion)
Cuboid bone dysfunction
Plantar glide and lateral rotation (eversion)
Cuneiform dysfunction
Plantar glide without rotation
-Tarsal metatarsal joint motion
=Generally less motion at the 2nd metatarsal
=Minor motions are anterior/posterior glide, medial/lateral glide, internal/external
rotation glide

Scan: Hindfoot and Midfoot


Subtalar
Inversion and Eversion at Talar Calcaneal
Navicular
Plantar position and tenderness
Medial and inferior glide
Cuboid
Plantar position and tenderness
Lateral and inferior glide
Cuneiforms
Plantar position
Plantar/inferior glide
Metatarsals
Adduction, Abduction, Plantar, Dorsiflexion, Rotation
Motion and Position

Functional Anatomy: Subtalar Joint


Talocalcaneal Joint
Secondary glide motions are
Inversion
Eversion
Also Talonavicular glides
Anteromedial (eversion)
Posterolateral (inversion)

LE Scan Sequence**

Knee
-Internal/External Tibial torsion
-Fibular Head
Midshaft
-Interosseous
Ankle/Foot
-Talus
-Subtalar
-Tarsal
-Metatarsal
IFITSTM
I found in the stone, the mallet

Diagnosis of Extremities: General SCANComparison is made to opposite extremity


Scan is guided by:
Chief complaint or
Screening exam findings in absence of complaint
Scan checks for
Position and
Motion

Diagnosis of Extremities: General- SCAN!


Named in ease
-Medial Sternoclavicular anterior and superior
-Posterior Fibular head
Dysfunction in secondary glides of joint (mainly)

Treatment Options
Direct
Find resistance position of joint
Move through that resistance by muscle energy, springing, or thrusting
Indirect
Find position of ease in all planes and phase of respiration
Refine position as release occurs

Scan: Knee
Position: Tibial tuberosity
Motion: Tibial rotation

Scan: Ankle
Position: Talar dome
Motion: Talar glide

Foot

Epicondyles of the femur


-Palpate the patella
-Slide your thumb lateral to the outside of the knee
-Slide back to the patella and move medially

Tibial tuberosity/ Patella how to do it? WHY?

Sit, knee flexed


-Place hand on the anterior aspect of the knee ("knee cap")
-Move fingers inferiorly until encountering a boney prominence
-Used to determine medial/lateral rotation of the tibia
-Continue down the tibia until reaching the medial malleolus

Tibial Plateau
-Place thumbs on both sides of the patella
-Slide inferiorly into soft, joint space
-Continue to palpate inferiorly until you feel bone

Q Angle
Less than 15 degrees in men
Less than 20 degrees in women
-Increase in Q angle can result from excessive ankle pronation
-Increase in Q angle may result in patellofemoral syndrome

Head of the fibula- WHERE?


Locate the tibial tuberosity-Slide fingers laterally 3-4 inches- Biceps Femoris tendon
inserts at the head of the fibula
-Have your partner plantar flex the fibular head. Assess motion.
-Grasp fibular head and input anterior/posterior motion
-Trace fibula inferiorly and this becomes the lateral malleolus

Talus assessment
-Locate the medial and lateral malleoli
-Now, move medially and anteriorly to the midline of the foot.
-Palpate the rounded prominence. This is the head of the talus
-Input rotary motion between the talus and calcaneus and compare motion
bilaterally

Landmark Palpation
Cuboid
Navicular
1st metatarsal head
5th metatarsal head
Cuneiform bones
Anterior talo-fibular lig.

Calcaneo-fibular lig.
Posterior talo-fibular lig.

Calcaneus where is it
Find the medial and lateral malleoli
Drop inferiorly onto the calcaneus (heel of foot)

Navicular & Cuboid WHERE ARE THEY?


- Navicular is located just inferior and distal to the medial portion of the head of the
talus.
-Invert and evert the foot to assure contact with the navicular, in inversion the
navicular tubercle will become more prominent
-Cuboid is located just proximal to the 5th metatarsal and articulates with the
calcaneus.
-Locate the tuberosity of the 5th metatarsal and move your contact proximally until
a drop off is felt
-Note the articulation between the cuboid and navicular

Cunieforms & Metatarsals


-Begin palpation just proximal to the metatarsal phalangeal joint.
-Explore all 5 metatarsal bones-The 5th metatarsal bone has a lateral prominence
named the tuberosity of the 5th metatarsal
-Locate the 1st metatarsal and move proximally to palpate a small joint between the
medial cuneiform and the 1st metatarsal
-Move onto the cuneiform and examine the bony structures more laterally (middle
and lateral cuneiforms)

Screen/Scan for Somatic Dysfunction in the Lower Extremity


-Directed at region of interest as identified by the patient (chief complaint)
-Assessed bilaterally for comparison of motion
-Focus is usually on smaller secondary motions
-Attention must be given to joints above and below complaint area (at least)
-If dysfunction appreciated in neighboring regions must be further examined.

What is Postural Balance?


Continual adaptation of muscular tone in response to gravitational influences
(among others).

DEFINE! Optimal Postural Balance


Perfect distribution of body mass around the center of gravity, with compressive
forces on spinal disks balanced by ligamentous tension.
Osteopaths have agreed that the center of L3 is the center of average, passive,
standing adults. Like this guy.

Why is Postural Balance so Important?


-Posture is but the formal expression of the balance of power existing between the
environmental force of gravity and the strength of the individual. Thus any
deterioration of posture indicates that the individual is losing ground in her contest
with the environmental forces of gravity."

Respiratory-Circulatory Model
The artery is the river of life, health and ease."
A.T. Still, M.D.
1)for health there must be good circulation, to ensure proper nutrition and drainage
from a gross to a cellular level (and back again)
2) In order to achieve this, the respiratory processes must be working efficiently.

J Gordon Zink D.O. AND FASCIAL DYSFUNCTION- discuss his theory


-Commonly found patterns of dysfunction

-Fascia: sheets of fibroelastic connective tissue throughout the body


4 "crossover sites" where fascial tension can occur
Based on Junction/transition areas
Cranium
Thorax
Pelvis
And the diaphragms that bridge them
-Tentorium
-Thoracic inlet
-Abdominal diaphragm
-Pelvic diaphragm
-Torsions of these fascial pathways can compromise neural vascular flow.
Embryologically, transition zones are weaker, and more exposed to acute or
repetitive trauma.

compensatory and noncompensatory mechanisms


Common compensatory pattern: a series of myofascial torsions that are compatible
with physiologic function
Non-compensatory: not rotated in alternating directions
-Compromises respiratory-circulatory integrity of body
-Can be traumatically induced
-Will eventually display congestive systems

4 types of alignment- case considerations


IDEAL, KYPHOTIC, FLAT BACK, SWAY BACK
The complaint of "Neck Pain shows up in two different biomechanical environments
in these two patients. (image 1 and 2)
Which one is more likely to complain of tension headaches? #2

Which one is more likely to be able to take a deep breath? #1


Which one might be more structurally vulnerable to case of pneumonia? #2
Structure of the spine will affect the function of the thoracic viscera- heart, lungs.
Can this person take good deep breaths? Can the heart efficiently provide enough
blood to the circulatory system?

Potential Postural Influencing History


=Congenital/Birth history
Congenital scoliosis (see appendix)
Shoulder dystocia, torticollis, hip dysplasia
=Trauma/Surgeries/Injuries
Childbirth: vaginal, c-section
Knee/hip replacement
Postural imaging studies
=Medical Diagnosis
Asthma
Osteoporosis/penia
=Activities
Occupation!
Swimming
Weight lifting

Diagnosis: Physical Exam


Visual Inspection : look at your patients critically*
*Remember: words have weight; thoughts are things
Structural Exam:
-gravity line
-landmarks
Palpatory Exam
-screen and scan

Landmark review
-Base of Occiput
-Scapular Spine
-Inferior angle
-Iliac crest, PSIS
==Standing flexion
-Knees
-Arches of feet

gravitational line
Gravitational Line Should pass through:
1. External auditory meatus
2. Lateral head of the humerus
3. Center of the Body of L3
4. Anterior third of the sacral base
5. Greater trochanter
6. Lateral condyle of the knee
7. Lateral malleolus
AH BS TLM

OA junction: base of occiput- case considerations


= Biomechanics: tonic neck reflexes; cranial strain patterns
= Neural: CN exit skull in this neighborhood
= Fluid: vertebral artery to interior of cranium; sagital sinus, transverse sinus as
venous drainage
= Viscera: CNS; sinuses
= SD at this level can induce
- Hypertonicity of postural muscles
- Disturbances of facial symmetry, equilibrium and locomotor deficits.
CNs: 10,11,12

CT junction: scapulae case considerations


= "relatively mobile cervical spine meets relatively rigid thoracic spine."
=Biomechanics: UE-shoulder girdle influences
==SITS muscles
==Serratus anterior, rhomboids, pectoralis etc
=Neural: brachial plexus, phrenic, vagus; upper sympathetic chain
=Fluid: thoracic inlet
=Viscera: head/neck, heart, lungs, etc

TL junction: (12th) ribs, iliac crests case considerations


=Diaphragm!
==Biomechanics: skeletal attachments
==Fluid: Aorta, Vena Cava, Azygous; fluid pump of body

==Visceral: esophagus, lower GI, liver, kidney


==Neural: phrenic; visceral symp/parasymp considerations
=Postural imbalance of Lower Body (biomechanics)
==Tonic: Iliopsoas, QL, erector spinae
==Phasic or Inhibited: Rectus abdominus

Lumbo-sacral junction:
=Horizontal alignment of Sacral Base:
==LOTS of implications into posture, pain.
=Fluids: inguinal nodes, pelvic diaphragm
=Neural: lumbo-sacral plexus, pelvic and sacral splanchnics
=Viscera: lower GI, reproductive, urinary considerations
Biomechanics: _____ _____ ______?

Muscle phys review: alpha gamma motor neuron review


= A disturbance of MS function initiates a series of events beginning with
stimulation to mechanoreceptors and nociceptors, resulting in afferent neural
activity...
= The final common pathway is the alpha motor neuron that stimulates the muscle
fiber to contract, and through the gamma system, the muscle spindle to adapt,
resulting in alteration in muscle tone. = Chronic dysfunction feeds the afferent loop,
more nociception and abnormal mechanoreceptor information, perpetuating
ongoing aberrant muscle tone.Disturbance
Results in alteration, adaptation
Interruption and reprogramming of the vicious cycle contribute to improvement of
overall muscle tone and balance.

What this means=


JOB SECURITY

Short Leg Syndrome


Condition in which there is an anatomical or functional leg length discrepancy that
results in
1. Sacral Base unleveling
2. vertebral side bending AWAY and rotation TOWARDS low side
3. Innominate rotation and side shifting towards long leg
4. and more
Earth is mostly even- Eyes level with the horizon: pelvic imbalance can manifest
further away (headaches) because of the body's ability to compensate and adapt to
imbalance
Between earth and eyes is where the compensation and dysfunction can occur.
Pelvis dropped toward the short leg
Spine curves convexly away from the short leg- shoulders compensate opposite to
pelvis

Common Findings in Short Leg syndrome


1. Sacral Base unleveling : lower on the side of the short leg.
2. Anterior innominate rotation on the side of the short leg- Iliacus contracture can
be source of this.
3. Posterior innominate rotation on the side of the long leg.
4. Lumbar spine will sidebend away and rotate toward the side of the short leg
5. Iliolumbar ligaments may become stressed on the side of the short leg.
6. SI ligaments may become stressed on the long leg side.

What happens when muscle pairs exert uneven pull on foundation?


how to fix?
-Muscle Imbalance:
-Upper Cross Syndrome
-Lower Cross Syndrome

WAYS TO FIX? Fundamental Principles of Muscle Imbalances


Sensory motor balance training: more here in year 2!
Stretching of short, tight (tonic) muscles FIRST.
Strengthening weak, inhibited (phasic) muscles NEXT.
Aerobic conditioning
Overall Goals:
=Maintenance of enhanced MS functional capacity
=achieved by appropriate manual med intervention and a continual exercise
program

Upper and Lower Cross Syndrome- #1 postural imbalance. Upper


cross
POSTURAL IMBALANCE
- Postural muscles tend towards HYPERtonicity;
- dynamic muscles (phasic) tend towards HYPOtonic overstretch.
Tonic = Tight = Short = Strong
Phasic = weak = Long
-Asymmetry in tensions of agonist-antagonist pair
-Imbalance can be in the coronal/frontal plane, as in scoliosis... or
anterior/posterior, as in upper and lower cross.
Upper cross: anterior compartment tightens while the posterior torso weakens .

'Lower cross' follows similar general patterning


- Patterns, not absolutes.
- Postural imbalance of Lower Body (biomechanics)
= Tonic: Iliopsoas, QL, erector spinae, tensor fascia lata, adductors, piriformis,
hamstrings.
= Phasic or Inhibited: Rectus abdominus, gluts (max, med, mini) obliques, vastus
medialus, lateralis, tibialis anterior
-Stretching of short tight muscles
-Strengthening weak muscles
-Aerobic conditioning

Upper Cross Imbalance:


Upper cross: anterior compartment tightens while the posterior torso weakens .
Tonic
**Pectoralis Major
Pectoralis Minor. L Scapulae, Tere Major, upper Trapezius**
Anterior Deltoid
Subscapularis. Latissimus Dorsi, Teres Major
SCM, Scalenes, Rectus Capitus
Phasic
Rhomboids
Lower/Middle Trap, Serratus Anterior
Teres Minor, Infra Spinatus Posterior Deltoid
Longus Coli, Capitus

Upper Cross Syndrome -in light ofUpper Half of the Body Tx seq.
Anterior Ribs
=Pectoralis mm
Thoracolumbar Junction
=Diaphragm redoming
Cervicothoracic Junction
=levator scap
Thoracics / Other Ribs
=Rhomboid, lower trap strengthening exercise
Scapulothoracic Articulation
=Serratus anterior strengthening exercise
Craniocervical Junction / Cranium
Cervicals
Rest of lower Extremity

Lower cross in context of Lower Half Sequence:


Hip restrictors
Iliacus -usually tight
2. Pubic bone dysfunction
rectus abdominus -usually inhibited
adductors -usually tight
3. Superior innominate shear
QL -usually tight
4. Lumbar dysfunction
5. Sacral dysfunction
6. Innominate
7. Iliopsoas (including thoraco-lumbar junction, approx. T11-L2)
usually tight; can contribute to functional short leg, psoas syndrome
8. Other Lower Extremity Dysfunction
hamstring -can be either tonic or phasic

gastroc glut max-min usually phasic, but can be either

Summary on Posture
-Inhibited muscles are going to become weak and overstretched; facilitated muscles
will become tight and constricted.
Upper cross- kyphotic
Lower cross sway back
Great transition into tensegrity: These are common patterns of dysfunction- agonist
and antagonist asymmetry. However it's even MORE complicated!
The take home point is that the muscles and ligamentous structures such an
important influence on the way we move- and therefore the way are bodies and
structured and how they function.

Postural Balance
Continual adaptation of muscular tone in response to gravity.
Changes with:
Body habitus
Medical Status
Phases of life--> Pregnancy
Daily Activities

Watching someone breathe

Medically important in context of resp-circulatory model


Breath should move the torso from the neck, down to the pubic bone
Look for: motion from neck to pubic bone
Areas here that are Restricted to motion

#1: Anterior Ribs: Pectoralis AND ATTACHMENTS!


Treat tight muscles first!
Pecs
Pt supine
Assess anterior aspect of shoulder: is one higher off the table the other?
Can also be assessed seated/standing.
Prox attachment: clavicle, sternum, superior 6 costal cartilages
Distal attachment: intertubercular groove
Innervation: medial and lateral pec nerves (cord levels C5-T1)
Action: adducts, internally rotates the humerus; draws scap ant and inf.

Stretching Hypertonics: Pectoralis


Working with your knowledge of muscular attachments (oh immersion!)
Isolate and stretch pec major and minor
How can this be modified to a ME treatment!?

Stretches to Teach: pec minor


Doorway Stretch With back straight, align elbow with eye against frame
step forward with one foot.
Feel the stretch in the Pectoralis area.
10-15 seconds, 3-5x
Should pt stretch both sides?

Upper half #2 Thoracolumbar Junction treatment


Diaphragm redoming:
Diagnosis: using F/E, SB, R, determine motions of ease.
Treatment: Carry thorax to stack these motions.
Hold in this position
Patient can use breath to assist in "unwinding"
BIG, long, easy breaths in and out, or breath holding.
Follow diaphragm IN, with broad pressure, on exhale
Until release is felt
Return to neutral and reassess.

Diaphragm redoming: new


Pt supine, doc seated
Pass your hands around A/P diameter of thoracic cage.
Anterior hand at lower costal cage
Posterior hand at thoracolumbar fascia/upper lumbars
Engage the tissue
Treat with direct or indirect methods, stacking in F/E, SB, R.

lower half #2 2) Pubic bone dysfunction


Assess landmarks: pubic tubercles
Treat asymmetric side
If no asymmetry:
Use AB/Adduction muscle energy
2-3 times in each direction

Upper half #3
Upper picture shows combination of Levator and Pec
How can this be modified to a ME treatment!?

Stretches to Teach: Levator Scapulae


Patient seated:
Patient holds on to the side of the chair with one hand and flexes the head as if
looking and into the opposite breast pocket.
Free hand is placed on the head and a gentle stretch is placed inferior and slightly
lateral. Hold for 5-15 seconds and repeat.

Address Phasic Muscles.


Inhibited muscles second!
Usually rhomboids, mid/low trapezius, +/- serratus anterior, latissimus.

#6 in upper - Occipital triangle


Craniocervical junctions is last stop for postural asymmetry. Eyes want to be level
with the horizon, semicircular canals demand a level environment to direct the head
and body. Postural asymmetry from the base up is "corrected" as a last effort at the
base of the skull to allow the extraoccular muscles the most level approach to
environment as possible. These Occ triangle muscles have quite an intimate
relationship with the EOM, and can perpetuate asymmetry if not corrected. See
Myers' Anatomy Trains, Superficial Back Line for more information

Indirect Tx for Occipital triangle


- Pt supine.
- Find base of occiput with fingerpads
- Allow pt's head to sink your fingers into the soft tissue.
- Pause here.
- Patient gazes softly to left; neutral; softly to right.
Use extraoccular muscles to stack ease onto F/E, SB, R at occipital base.

summary 1

summary 2

Supplemental Exercises for Lower Trapezius


Patient prone, doc on side being tested
Ask patient to activate Lower Trap by pulling scapula inferiorly and medially
towards the spine
Retraining: airplane arms! (to move activation lower, can instruct pt to move into
"warrior I arms"

Lower Trapezius at home


Prone:
or
Standing, pull scapula together along spine (retraction)
Bring elbows flexed or extended along towards sides to approximately 45
Repeat 5-7 x
(Also addresses rhomboids)

Serratus Anterior: innervation and function


Prox attachment: Anterior surface of medial border of scapula
Distal attachment: ribs 1-8
Innervation: Long thoracic nerve!
Action: protracts and rotates scapula.

Supplemental Exercises for Serratus


Serratus Anterior -With patient laying on their side. The elbow bent, while flexing at
the shoulder.
Stretch for lower fibers: inferior angle moves medially and superiorly*
Upper fibers: move sup angle medially and inferiorly

serratus strengthening
Strengthen: Keep the arm parallel to the floor. May add resistance when the patient
is able. (Protracts Scapula)
Semi-truck Honk Motion.

Supplemental Exercises for Rhomboids


Patient prone, doc on side being tested
Ask patient to activate rhomboids, by pulling scapula inferiorly and medially
towards the spine
Monitor along superior medial border of scap
Retraining: Jet fighter plane arms!

Techniques/Skills
Anterior/Posterior Fibular Head
-ME
-Direct/Indirect
Anterior Posterior Glide Dysfunction, Knee
-Direct/Indirect
Interosseus Membrane
-Indirect
Anterior/Posterior Talus
-ME
-Direct/Indirect

Subtalar Inversion/Eversion
-Indirect
Plantar Navicular/Cuboid/Cuneiform
-Direct/Indirect
Dorsal/Plantar/Medial/Lateral Metatarsal
-Direct/Indirect

Dysfunction: Knee
Anterior tibial glide
Posterior tibial glide
Medial tibial glide
Lateral tibial glide
Anteromedial tibial glide
Posterolateral tibial glide
Most common: anteromedial glide, medial glide and posterior glide

Treatment: Knee
Direct
Find resistance position of joint
Use muscle energy or springing to activate
Indirect
Find position of ease in all planes
Use breath as activating force

Dysfunction: Anterior Fibular Head


Treatment: Direct Articular (Supine)
Flex knee slightly
Internal rotation of tibia
Thumb over fibular head, opposite hand stabilizes lateral malleolus
Rapid extension of knee
Encourage posterior movement of fibula
Treatment: Direct Muscle Energy
Seated position
Thumb over fibular head directing posterior
Plantar flexion of ankle to resistance (encourages inversion, supination, internal
tibial rotation, posterior proximal fibula)
Isometric contraction dorsiflexion
Repeat

Dysfunction: Posterior Fibular Head


Treatment: Direct Articular
Flex knee
External rotation of tibia
MCP or thumb posterior to fibular head

Flex to resistance
Spring flexion of knee to encourage fibular head anterior
Treatment: Direct Muscle Energy
Seated position
Thumb and finger grasp fibular head
Dorsiflexion of ankle to resistance (encourages eversion, pronation, external tibial
rotation, anterior proximal fibula)
Isometric contraction plantarflexion
Repeat

Dysfunction: Interosseous
-Proximal or distal restriction identified by testing motion while palpating both
proximal fibular head and lateral malleolus
Treatment Indirect
-while holding proximal and distal fibula, balance ease within interosseus
membrane and soft tissue, use breath as activating force

Dysfunction: Ankle
-Anterior/posterior talus
Most common: anterior/plantar flexed talus
Treatment Direct
-Seated or supine-- dorsiflex foot until talus reaches edge of resistance, have patient
plantarflex against resistance
Treatment Indirect
Seated or supine plantarflex foot to ease, balance all motions possible, use breath as
activating force

Dysfunction: Subtalus + motion


Inversion/eversion of calcaneous
With associated posterolateral/anteromedial talus
Treatment Indirect
Seated or supine
Stabilize talus, find balanced ease in calcaneous, use breath and reposition as
spontaneous release

Dysfunction: Navicular/Cuboid/Cuneiform +motion


Plantar and medial rotationnavicular
Plantar and lateral rotationcuboid
Plantar cuneiform
Treatment Direct ("Hiss Whip")
Patient prone, thumb over plantar dysfunction, plantar flex to resistance, move foot
"to and fro," then rapid short thrust through resistance

Treatment Indirect
Balance ease in all planes, use breath, re-establish position in release

Dysfunction: Metatarsal + motion


Dorsal/plantar glide
Medial/lateral glide
Treatment Direct
Seated or supine. Stabilize metatarsal in question. Articulate MT through range of
motion.
Treatment Indirect
Seated or supine. Balance metatarsal into ease, use breath to facilitate release,
reposition accordingly

Upper Extremity:
Screen
Screening:
Two Tissue Textures
Two Motion Tests
At each joint
Evaluate Joint Position
Palpate minor motion of joints
Specific Joint will be compared bilaterally
Note: Unlike the axial spine, comparison of joints not above and below

Upper Extremity:
Areas to Scan
Shoulder Complex (4)
Sternoclavicular,
Acromioclavicular,
Glenohumeral,
Scapulothoracic
Elbow Complex (3)
Radial Head,
Ulnar-Humeral Joint
Interosseous Membrane
Wrist (3)
Carpals, Metacarpals, Phalanges

Upper Extremity:
Segmental Definition
-Determined during scan
-Joint named for position of ease or motion of ease
-Finding is relative to contralateral UE
Ex. Sternoclavicular Joint
Superior Clavicle at Sternoclavicular Joint
Superior Glide at Sternoclavicular Joint
Note: Segmental definition in upper extremity is similar to segmental definition in
the axial spine

Shoulder Complex
Normal motion at shoulder involves motion at multiple joints
Sternoclavicular
Acromioclavicular
Scapulothoracic
Glenohumeral
All 4 joints are assessed during upper extremity Scan

Sternoclavicular Joint - position and motion


Position:
Superior/Inferior
Anterior/Posterior
Superior/Inferior Glide
Anterior/Posterior Glide
Rotation
This is the only bony attachment of your arm to the body! Eek
Important for proper shoulder motion
Only bony articulation to attach UE!

testing sternoclavicular joint


To Palpate Proximal Clavicular Position:
Physician Behind Patient
Thumbs on superior portion of proximal clavicle bilaterally

Assess for superior/inferior position


Fingers placed on anterior surface of clavicle bilaterally
Assess anterior/posterior position
Note: can be performed seated or supine

Shoulder Complex:
Sternoclavicular Joint Motion To Test Superior/Inferior Glide:
Shrug Test
To Test Superior/Inferior Glide:
Palpate superior aspect of clavicle
Patient "shrugs" shoulders, then returns to neutral
Normal Motion
Proximal clavicle glides inferior with shoulder shrug
Proximal clavicle glides superior with return to neutral
Objective: Right Clavicle depressed at the SC joint with Inferior Glide
Assessment: Somatic Dysfunction of the Upper Extremity

Shoulder Complex:
Sternoclavicular Joint Motion To Test Anterior/Posterior Glide:
To Test Anterior/Posterior Glide:
Palpate anterior aspect of clavicle
Patient adducts and flexes shoulder to 90
Patient reaches forward, then returns to neutral
Normal Motion:
As patient reaches forward, clavicle glides posterior
Return to neutral, clavicle glides anterior

Ex:
Objective findings: Right clavicle anterior at SC joint with Anterior Glide
Assessment: Somatic Dysfunction of the Upper Extremity

Diagnosis Review & Treatment


-Sternoclavicular Jt
Direct Springing-Anterior & Superior Glide
-Doc places caudal hand on the table and cephalad hand on inferior portion of the
proximal clavicle
-Patient uses opposite hand to stabilize the arm
-Spring posteriorly and inferiorly-lateral
-Recheck

Acromioclavicular Joint position and motion


Position:
"Stair-step" - drop off from clavicle
Motion:
Abduction vs Adduction
Internal Rotation vs External Rotation
Follow clavicle distally to acromion
Place index finger on distal clavicle
Place middle finger on acromion

Palpate for "stair-step"


Compare bilaterally

motion testing for AC joint- To Test for Abduction vs Adduction:


To Test for Abduction vs Adduction:
Midde finger on distal clavicle
Index finger on acromion
Passively abduct arm while palpating AC joint
Palpate joint for restriction of motion and compare bilaterally

treatment of AC joint- adduction ease treatment- ME!


Muscle energy - Adduction ease
Palpate AC joint
Place the pt's AC joint into restriction, abduction
Have patient contract against your forearm 3-5s
Post isometric relaxation
Reposition joint into further restriction, repeat 3-5 times
Recheck

motion testing for AC joint=To Palpate Internal/External Rotation:


To Palpate Internal/External Rotation:
=Palpate AC joint same as previous slide
=Bring upper extremity to 90
=Grasp wrist and externally rotate, while palpating for restriction at AC joint
Wrap arm under patient's arm and grasp wrist, internally rotate while palpating for
restriction at AC joint
Example:
Objective: Right AC Joint positive stair-step, abducted and externally rotated
Assessment: Somatic Dysfunction of the Upper Extremity

treatment of AC joint- internal rotation ease treatment- ME!


Muscle energy - Internally rotated AC
-Place the pt's AC joint into restriction, external rotation
-Have patient contract against your forearm 3-5s
Post isometric relaxation
-Reposition joint into further restriction, repeat 3-5 times
Recheck

Scapulothoracic Joint- position and motion


Position
Protracted vs Retracted
Superior vs Inferior
Rotated Upward vs Rotated Downward
Motion:
SAME
Palpate landmarks and compare bilaterally

Shoulder Complex:
Scapulothoracic Joint Position Supine
Place palm of hand on each anterior aspect of each shoulder
Gently compress both shoulders and assess for resistance
Side of resistance = side of shoulder protraction

Shoulder Complex:
Scapulothoracic Joint Protraction: Muscular Involvement in
protraction:
Anterior Element: Pectoralis Minor
Lateral Element: Serratus Anterior

Scapulothoracic Joint Protraction- Assess Pectoralis Minor AND


TREATMENT
To Assess Pectoralis Minor:
-Contact insertion of pectoralis minor at ribs 3-5
-Abduct ipsalateral upper extremity while palpating insertion of pectoralis minor
-Compare ROM of pectoralis minor bilaterally
Muscle Energy- Hypertonic Pec Minor
Abduct extremity until restriction (~120-130) is palpated in the pec minor
Have the patient adduct for 3-5s
Reposition into restriction and repeat 3-5 times
Recheck
Note: Can be performed seated or supine

To Assess Serratus Anterior:


To Assess Serratus Anterior:
-Patient is lateral recumbant
-Contact lateral edge of scapula with thumb and thenar eminence of caudad hand
-Cephaland hand grasps superior portion of scapula
-Test motion medially and superiorly

Test motions for: Scapulothrocic Joint Motions


Same contact as for testing serratus
Test motions for:
Protraction vs retraction
Superior glide vs inferior glide
Upward rotation vs downward rotation
EX:
Objective Findings: Hypertonic rhomboids with upwardly rotated scapula
Assessment: Somatic Dysfunction of the Upper Extremity

Diagnosis Review & Treatment


-Scapulothoracic Jt
Stretching/ME- Hypertonic serratus anterior
-Move scapula superior and slightly medial into restriction
-Have pt pull scapula inferior and hold for 3-5s
-Post isometric contraction reposition and repeat 3-5 times
-Recheck
Stretching/ME hypertonic levator scapulae or trapezius or rhomboids
Place restricted muscle into restriction and have the patient move towards ease,
hold 3-5s
Post isometric contraction reposition and repeat 3-5 times
Recheck

Glenohumeral Joint Position: and Motion


Position:
Anterior vs Posterior
Motion:
Voluntary:
Flexion vs Extension
Abduction vs Adduction
Internal vs External Rotation
Involuntary Motion: Slide/Glide with voluntary motion directions

Shoulder Complex:
Glenohumeral Joint Position
Position
-Patient seated with physician behind
-Thumb on acromion
-Middle finger on anterior aspect of humeral head
-Compare distance between thumb and middle finger bilaterally
Motion
Thumb on acromion
Index and middle finger on anterior aspect of humeral head

Squeeze thumb and fingers together


Compare resistance to glide bilaterally
Ex:
Objective Findings: Right Anterior humeral head preferring anterior glide
Assessment: Somatic Dysfunction of Upper Extremity

Elbow Complex: components


Humeroradial Joint
Humeroulnar Joint
Interosseous Membrane

Humeroradial Joint Position: and motion


Position:
Radial Head Anterior/Posterior
Motion
Radial Head Rotation on Humerus
Anterior with Supination
Posterior with Pronation

how to test position and motion


POSITION
1. Thumb anterior to proximal radial head

2. Index and middle finger on posterior aspect of proximal radial head


3. Compare Bilaterally
MOTION
Monitor at proximal radial head (as in previous slide)
With other hand, grasp patient's hand in the handshake position
Monitor proximal radial head while supinating and pronating the hand
Example:
Objective Findings: Proximal Radial Head Anterior
Assessment: Somatic Dysfunction of the Upper Extremity

Remember! with motion! radial head - supination and pronation


MOTION
Monitor at proximal radial head (as in previous slide)
With other hand, grasp patient's hand in the handshake position
Monitor proximal radial head while supinating and pronating the hand
Radial Head Posterior with Pronation
Radial Head Anterior with Supination

Review & Treatment


-Radial Head POST and ANTERIOR

Muscle Energy - Anterior Radial Head


Pronate the forearm to place radial head into restriction while palpating the radial
head
Have your patient supinate the forearm against your force
Post isometric contraction reposition and repeat 3-5 times
Recheck
Muscle Energy - Posterior Radial Head
Supinate the forearm to place radial head into restriction while palpating the radial
head
Have your patient pronate the forearm against your force
Post isometric contraction reposition and repeat 3-5 times
Recheck

Elbow Complex:
Humeroulnar Joint- position and motion- carrying angle
Position:
Abduction vs Adduction
Motion:
Major Motion: Flexion vs Extension
Minor Motion: Abduction vs Adduction

Humeroulnar Joint Position and motion

POSITION
Grasp Elbow with both hands
Evaluate for increased/decreased carrying angle and compare bilaterally
MOTION
Using both hands, grasp the proximal ulna
Place patient's wrist between physician's elbow and trunk
Add translatory force medially and laterally (adduction/abduction)
Compare Bilaterally
Example:
Objective Findings: Right Elbow increased carrying angle with right humeroulnar
joint abducted
Assessment: Somatic Dysfunction of Upper Extremity

Diagnosis: Posterior radial head, posterior translation radius,


increased carrying angle
Stabilize the humerus - done by patient position.
Make contact with the proximal radius - cephalad hand; caudad hand - hand shake.
Use the motions available across the joint to enhance ease.
Ant/post translation, supination/pronation, compression/traction.
'Stacking' sequentially; Use respiratory ease; Eventual 'Smooth Torsion Arc'

Interosseus Membrane

Important for supination/pronation at forearm


Acts like a shock absorber in trauma
To Test for Interosseous Membrane:
Palpate for tension bands in forearm proximally -->distally
Ex: Objective Findings: Forearm Interosseous Membrane - Increased Tension 2/3
distally
Assessment: Somatic Dysfunction of the Upper Extremity

Diagnosis: Fascial restriction of interosseous membrane


1. Stabilize the humerus - done by patient position.
2. Make contact with the proximal radius - prox. hand; distal hand - hand shake.
3. Use the motions available across the joint to enhance ease.
--Supination/pronation, Med/lat & Ant/post translation, flexion/extension,
compression/traction.
4. 'Stacking' sequentially; Use respiratory ease; Eventual 'Smooth Torsion Arc'

Wrist:
Carpal Bones and Metacarpals: Position and Motion
Position:
Superior vs Inferior
Internally vs Externally Rotated
Motion:
Evaluate for superior vs inferior glide
Evaluate for internal vs external rotation
Evaluate each carpal bone and metacarpal and compare bilaterally
Example:
Objective: Right Lunate Ventral
Assessment: Somatic Dysfunction of the Upper Extremity

Carpal Bones and Metacarpals treatment


Stabilize the proximal bone with proximal hand.
Make contact with the distal bone with opposite hand
Use the motions available across the joint to enhance ease.
Med/lat & Ant/post translation, flexion/extension, compression/traction.
'Stacking' sequentially; Use respiratory ease; Eventual 'Smooth Torsion Arc'

Diagnosis Review & Treatment


-Metacarpals/Phalanges
Balanced ligamentous tension(BLT), aka ligamentous articular release (LAR) - Any
dysfunction
Stabilize proximal joint
Disengage: compression or traction
Exaggerate ease
Balance (follow) ease
Can also use the phase of respiratory ease to adjust motions
Recheck
Naming?

Diagnosis Review & Treatment


-Carpal bones
-Articulatory with traction -Any carpal dysfunction
-Place hands perpendicular to pts, one on each side of their wrist
-Squeeze palms together and use clockwise and couterclockwise motion, or figureof-eight carrying dysfunction gently through the barrier(s)
-Recheck

Upper Extremity Diagnostic Sequencing:


Beginner's Approach SUMMARY
Start proximally and work distally:
Cervical, Thoracic, ribs (Dx & Tx as previously taught)
SC
AC
Scapulothoracic joint
GH
Elbow
Wrist / Hand
Depending upon the chief complaint, this can be a good clinical approach.
For beginning students, it is a good idea to use this as a protocol to gather all the
relevant diagnostic information before deciding on where to start the treatment
You rarely find only one dysfunction when scanning the extremity.
How do you decide where to start when there are two or more dysfunctions in the
upper extremity?
Severity?
What if one location is too 'hot' to move?

Screen the upper extremities 2 Tissue Texture


2 Motion Tests
Scan using Position and Motion comparing bilaterally:
1. Sternoclavicular 4. Elbow

2. Acromioclavicular 5. Wrist
3. Glenohumeral 6. Hand
Segmentally Definition
Based on motions at each unique joint
Name in ease of position and motion

golfer's elbow. tennis elbow


Medial epicondylitis
lateral epicondylitis

innominate
Illium
Ischium
Pubis
WE HAVE 2

How do you screen the pelvis?


TISSUE TEXTURE
-Lateral Posterior Gluteal Tissue
-Sacral Tissue
MOTION
-Lateral Translation
-Pelvic Rotation

scan the pelvis?


Standing Flexion Test (ilio-sacral)
*ASIS Compression Test

Standing Flexion Test


(Scan of Pelvis)
Place thumbs under PSIS
Ask pt. to slowly bend at waist
Watch for thumb movement (superior/anterior)
Side that moves first is the positive side

Standing flexion test


Performed when scanning the pelvis
For pelvic dysfunction
Indicates __ILIO-SACRAL__________ component of dysfunction

Seated flexion test


Performed when scanning the sacrum
For sacral dysfunction
Indicates _SACR-OILIAC__________ component of dysfunction

What landmarks do you need to segmentally define the pelvis?


Anterior: iliac crests, ASIS, pubic tubercles, medial malleoli*
Posterior: PSIS, ischial tuberosities
check motion at sacrotuberous ligament (*)

Anterior Muscles Attachments- what attaches to each landmark?


ASIS
Sartorius
AIIS
Rectus femoris
ILIAC CREST
Tensor Fasciae latae
Gluteus Medius
PUBIC BONES
Adductor Longus
Pectineus
Adductor Brevis
Gracilis
PSIS
Gluteus maximus
ILIAC CREST
Transversus abdominus
External Oblique
Quadratus lumborum
ISCHIAL TUBEROSITY
Quadratus femoris
Adductor magnus
Biceps femoris
Semimembranosus

Making a Pelvic Diagnosis

The standing flexion test was the scan of the pelvis


Checking landmarks is how you segmentally define the pelvis
Record your findings at each landmark with respect to the side of the (+) standing
flexion test
Example: (+) standing flexion test on the R
If you find that the L iliac crest is higher than the R, record that the R iliac crest is
lower
Record what the side of (+) standing flexion test is doing!
This makes your diagnosis much easier at the end
Put all the pieces together to make a diagnosis
R Anterior Innominate, L Superior Pubic Shear, etc.

pelvic dysfunctions
Anterior innominate rotation
Posterior innominate rotation
Superior pubic shear
Inferior pubic shear
Superior innominate shear ("Upslip")
Inferior innominate shear ("Downslip")
Inflare
Outflare

What is the origin and insertion of the Rectus Femoris?


Anterior Inferior Iliac Spine/acetabular roof of the hip joint and
the tibial tuberosity via patellar ligament

What is the action of the posterior thigh muscles?


Hip Joint: Extends the hip,
stabilizes the pelvis
Knee Joint: Flexion and
internal rotation

Anterior Innominate Rotation


Findings (e.g. Left Anterior Innominate)
+ standing flexion on left side
L ASIS inferior
L PSIS superior
L medial malleolus may be inferior
L sacrotuberous ligament loose
Normal Physiologic Motion (iliosacral)
Tight quads

anterior rotation loosens what ligament


sacrotuberous

Anterior Innominate ME Treatment


Cephalad hand- L ASIS
Caudad hand- L ischial tuberosity
Flex knee and hip
Adduct LE to gap SI
Instruct pt to push knee into chest (gently!!)
Repeat 3xs, recheck
Obj: rotate innominate posteriorly using isometric counterforce

Posterior Innominate Rotation


PSIS down
ASIS up
Medial Malleolus Up
ALLL on one side

Posterior Rotation: ME Treatment


Pt is supine
Cephalad hand on opp ASIS, caudad hand just above knee
Pt is instructed to resist into caudad hand (or bring leg up); pt is bringing
innominate anterior
Repeat, recheck
Reminder: sacrotuberous ligament is tight on the side of posterior rotation!

Inflare/Outflare
Rare*
Diagnosed after the correction of any other pelvic dysfunction
Inflare Findings
Ex. Right
Standing Flexion Test positive on the right
Medial ASIS on right
Outflare Findings (at left)
Ex. Right
Standing Flexion Test positive on the right
Lateral ASIS on right
Anterior rotation can be associated with inflare. Posterior rotation can be associated
with outflare.

Innominate Inflare
Muscle Energy Treatment

Ex: Right
Doc: Flex R hip and knee, Abduct R hip to restrictive barrier while stabilizing L
ASIS
Pt moves knee toward midline while doc resists
Repeat, add further abduction, recheck

Innominate Outflare
Muscle Energy Treatment
Ex: Right
Doc: Flex R hip and knee, Adduct R hip
Doc's L hand monitors at R PSIS
Pt moves knee out while doc resists
May offer more adduction, repeat and recheck
*Opposite treatment as inflare*

Upslip, Right- HOW DOES THIS HAPPEN?


Step Off, Car accidents, Fall landing on buttocks
Landmark findings:
+ Right standing flexion
Iliac crest: R
ASIS: R
Pubic bone: R
Med Mal: R
PSIS: R
Ischial tuberosity: R
ST lig: lax on R

Inferior Shear (Down Slip)


Foot entrapment HOW DOES THIS HAPPEN?
Inferior Shear (Down Slip)
Foot entrapment
Landmark findings
Iliac crest: L
ASIS: L

Malleolus: L
PSIS: L
Ischial Tuberosity:L
Pubic bone: L

Pubic Shear- caused how?


Superior Shear
More common of the two
Typically caused by standing with weight unevenly distributed or by muscle
imbalance
Findings:
+ standing flexion test
Ipsilateral pubic bone superior
+/- Tension and tenderness of ipsilateral inguinal lig.

Inferior Shear
Typically caused by muscle imbalance
Findings:
+ standing flexion test
Ipsilateral pubic bone inferior
+/- Tension and tenderness of ipsilateral inguinal lig.

Standing flexion: what it means


Positive test: dysfunction is named for side that moves.
R moves first? "positive standing flexion on RIGHT"
ALL FURTHER DIAGNOSIS STEMS FROM SIDE OF DYSFUNCTION
Equal motion?- negative test
No dysfunction
Symmetrical dysfunction

Segmental definition: supine ASIS


ASIS Evaluation
Position
Thumbs immediately inferior to both bony prominences
S/I;A/P; I/E-closer or further away from belly button
ASIS Diagnosis
Position
Superior
Inferior

Scan: supine ASIS compression


Direct compressive pressure at ASIS...
...directed towards the Posterior Articulation of Innominate and Sacrum (i.e. The SI
joint)
...gives you information about the motion that joint is able to perform.
Springy vs Concrete
Compression test. Ease of motion in the A/P planes is noted with alternating,
rocking pressure

Segmental definition: supine pubic tubercles


Pubic tubercles Diagnosis
Position
Superior, or
Inferior?

Segmental definition: supine MM


Medial malleoli
Position
Compare relative position of the malleoli
Sup/inf?

Segmental definition: Prone


PSIS
Position: compare heights

Superior?
Inferior?

Segmental definition:Prone Ishceal tuberosities


Ischial tuberosities:
Position:
Cephalad
Caudad

SacroIliac articulation
Apply gentle, springing ANTERIOR, SUPERIOR presure at ILA.
Springy?
Concrete, unyielding?

Treatment by Muscle Energy


Position into restriction
Patient-activated movement towards ease
Pause to allow patient relaxation
Re-establish resistance by positioning
Repeat 3-5 times
Return to neutral

Pubic Shear Right PT Superior


More common of the two
Typically caused by standing with weight unevenly distributed or by muscle
imbalance
Findings:
+ standing flexion test
Ipsilateral pubic bone superior

+/- Tension and tenderness of ipsilateral inguinal lig.


Position of restriction: Bring right side inferior
Anterior rotation, pubic bone dips inferiorly
Patient actively seeks "ease"
Superior motion.

Pubic shear: Right PT Superio TREATMENTr


Doctor carries R leg into extension
Added ABduction, external rotation will gap the pubic bone to allow for inferior
movement
Doctor stabilizes at contralateral ASIS
Provides resistance at ipsilateral knee
Patient directed to "bring leg back on to the table"

Inferior Shear
Inferior Shear
Typically caused by muscle imbalance
Findings:
+ standing flexion test
Ipsilateral pubic bone inferior
+/- Tension and tenderness of ipsilateral inguinal lig.

Pubic inferior shear: L inferior


Pt supine
Doc flexes knee and hip of dysfunctional side.
ABduction gaps the pubic symphasis
Dr stabilizes IPSIlateral ASIS and ischial tuberosity to encourage posterior rotation,
superior pubic bone motion.
Patient tries to move leg toward end of table with doctor resisting.
Patient directed forces should localize at pubic bone.

Upslip, Right
Patient supine, doc at end of table.
Doc grasps RLE, just above ankle, braces L foot against thigh
inducing ABduction and INternal rotation to gap SI joint
Added traction encourages R innominate into restriction
Instruct patient to bring R hip superiorly towards R shoulder, or like they are
"pulling foot out of a tall boot"

Downslip, Left
Patient prone; hip and knee flexed, draped over side of table, with foot contacting
thigh of doctor
Doctor adds cephalad pressure through ischial tuberosity; pressure through knee
contact to "loose pack" the SI joint.
Patient attempts to straighten leg against doctor thigh.

H/P for Low Back Pain


History
Physical Exam
Differential Diagnosis
Lab & Imaging

Differential Diagnosis for LBP


Back strain
Acute disc herniation
Osteoarthritis
Infection
Malignancy
Spinal stenosis

BIOMECHANICS of Low back relationships


Bony
Ligamentous &
Muscular elements
Where you have joints you have the possibility for somatic dysfunction.

'Foundation' for trunk function


Sacroiliac & Pubic Joints are key components in the integrity of force distribution
from above and below.

Pubic Joint Function:


=Fibrocartilaginous joint
-Cartilaginous on each pubic bone at the joint
-Fibrous disc in between
=Motions Allowed are Small
Superior/Inferior
Anterior/Posterior
=Important for 'Shock Absorption'

Sacroiliac Joint Function:


Determined by:
-Joint shape and bony leverage
-Ligamentous and capsular integrity &/or strain
-Muscular pull across or around the joint

Sacroiliac Joint Function:


Each joint moves in relationship to the shape of the S-I joint

Sacroiliac Joint Function: HOW IS THIS A UNIVERSAL JOINT?


-3 functional axes are used to describe most common motions and associated
somatic dysfunction.
--They are not the only possible motions.
--This is, in reality, a 'universal joint'
What does that mean?
It can allow small motions in any direction.

important anterior ligaments

important posterior ligaments

Muscular Anatomy: Hip Restrictors:#1 - other important muscles too


Iliacus #1!
Hamstrings
Adductors
Piriformis
OTHER
Quadratus Lumborum
Psoas minor and major
Muscles of Axial Spine Thoracic & Lumbar

Attachments of all important muscles


Quadratus lumborum: from iliac crest to rib 12 & attaching to lumbar vertebrae
Psoas minor and major: from L-T junction to lesser trochanter
Iliacus: inside of ilium, capsule of S-I joint & edge of sacrum to lesser trochanter
Hamstrings: ishcial tuberosity across knee joint
Adductors: Pubic bone to femur
Piriformis: anterior edge of sacrum to greater trochanter

Lower Half of the Body


Diagnosis & Treatment Sequence
1. Hip restrictors
2. Pubic bone dysfunction

3. Superior innominate shear ("innom. upslip")


4. Lumbar dysfunction (esp. L5 or, less often L4)
5. Sacral dysfunction
6. Innominate dysfunctions (other than upslip)
7. Iliopsoas (including thoraco-lumbar junction, approx. T11-L2)
8. Other Lower Extremity Dysfunction

Iliacus importance on rotation of pelvis?


The arrows suggest the leverage of the iliacus muscle on the axis of motion for the
innominate.
The leverage of the hamstrings could be a significant, but in the upright position it is
more directly under the axis.

what can contribute to sacral dysfunction of the innominate?


Piriformis can contribute biomechanically to sacral dysfunction

Piriformis origin and insertion


Piriformis: Arises from the anterior surface of the sacrum;
If hypertonic or contractured, can create a relatively fixed pivot around which the
sacrum must move.

Adductors: significant attachment to what?


Significant attachment to pubic rami, thus hypertonic or contractured adductor
could contribute to dysfunction.

Pubic Dysfunction Influences S-I Dysfunction: pubic joint is important


too...
Pubic Joint's lack of mobility affects affects S-I function
Less shock absorption through the pelvis

3. Superior Innominate Shear


This is non-physiologic.
It is not gliding within the 'L' shape.
Puts a lot of stress on the ligaments
Traumatic in nature
No other S-I motion will readily occur when this is present.

4. Lumbar Dysfunction: name ligaments involved


Imagine L5 FRSR:
That is going to put tension on the sacral base via the annulus of the disc and via the
anterior and posterior longitudinal ligaments
On the innominate via the iliolumbar ligaments

5. Sacral Dysfunction: THE FOUNDATION OF THE REST OF THE


AXIAL SPINE
Subtle motions of this bone are required for the normal motions of walking,
bending forward, arching & more complex motions (sports, dance, work)
The sacrum is the foundation for the rest of the axial spine and its function.

6. Innominate Dysfunction: RESTRICTIONS


Physiologic motion restrictions:
Anterior/Posterior Rotation
Inflare/Outflare
Stress the S-I joint.
Upslip is not a physiologic dysfunction, therefore not listed here.

7. Iliopsoas (L-T Junction)- muscleS? bones?


-Iliopsoas (including thoraco-lumbar junction, approx. T11-L2 & ribs 11 -12)
-Usually compensatory, therefore one of the last areas to diagnose and treat

8. Other Lower Extremity Dysfunction


Knee, ankle, foot:
If these areas are unstable or strained, they affect the sacro-pelvis - the foundation
for the rest of the body.

If acute, directed to that area by the patient's complaint


-The other components of the 'Lower Half of the Body Diagnosis and Treatment
Sequence' may affect a lower extremity complaint.
-If chronic may be part of an underlying pattern contributing to the patient's chief
complaint.
so extremity affects pelvis- acute, treat knee- chronic- may be part of pattern

DYSFUNCTION TYPES
Myotonic Dysfunction: ROM?
articular dysfunction: end feel?
neuromuscular: tissue texture?
Myofascial: ROM?

PAIN!! PATHWAY!
-Nerve pathways are multilane freeways
-Pain fibers follow the course of the mixed spinal nerves
-Facilitation in the nervous system will amplify the experience of pain
-Local inflammation will prolong the pain fiber stimulation.

Lymphatic Drainage - Relevant to UTI


Lymphatics are the 'Overflow system' of the body.
Local drainage or lack thereof is only one component in the equation of fluid build
up.
Key Principle: Any blockage Proximal to the excess fluid will inhibit its drainage.

Is lymphatic drainage for the urinary bladder and the urogenital diaphragm
restricted?

visceral sympathetic cord levels


Biomechanical influences
SNS, PNS, Pain pathway influences
Fluid Drainage influences

visceral parasympathetic
Biomechanical influences
SNS, PNS, Pain pathway influences
Fluid Drainage influences

Key Components of the History


Onset
Location
Type of pain
Trauma history
Medical history
Psychosocial stressors
Red flags: weight loss

Physical Exam components


Comprehensive physical exam:
Observation
Gait & Stance
Mobility

Neurological Exam
SLR
Reflexes
Motor or Sensory Deficits
GI or GU, if indicated
Osteopathic examination

Diagnostics:
Lab tests and imaging

RED FLAGS! IN HISTORY


Cancer
Unexplained weight loss
Immunosuppression
Prolonged use of steroids
Intravenous drug use
Urinary tract infection
Fever
Pain that is increased or unrelieved by rest
Significant trauma related to age
Bladder or bowel incontinence
Urinary retention (with overflow incontinence)

RED FLAGS! IN PHYSICAL EXAM


Saddle anesthesia
Loss of anal sphincter tone
Major motor weakness in lower extremities
Fever
Vertebral tenderness
Limited spinal range of motion
Neurologic findings persisting beyond 1 mo.

why might you take an X-ray?


After 65 years of age, cancer, compression fractures, spinal stenosis, and aortic
aneurysms become more common.
Therefore, early X-ray is indicated in elderly patients.

Management
NSAIDS
ICE
Heat
Short term narcotic use
Activity assessment
Physical therapy
Manipulation

Board Review Chart

Important joints of hip and important ligaments


Joints:
Pubic Symphysis
Hip Joint-Acetabular-femoral
Sacro-iliac Joint
Ligaments
Iliofemoral- Strongest ligament in body.
With lower extremity extension - pulls innominate into anterior rotation - mid to
late stance phase
Prevents overextension of hip when standing
Pubofemoral
Anterior and inferior reinforcement
Ischiofemoral
Posterior reinforcement- prevents hyperextension

Hip Joint Funciton


Standing
Foundation for axial skeleton, ribcage and upper extremities
Transition from bipedal stance and motion to a central spinal column
Support for the abdominal & pelvic contents

Walking
Superior-Inferior Translatory Motion of the pelvis
Rotational Motion Left & Right
Sidebending / shift Left & Right

Hip Ranges of Motion


Flexion: 120-135
Extension: 30
Abduction: 45
Adduction: 20-30
Internal Rotation: 35
External Rotation: 45
VARIES IN REAL LIFE

Limiting structures (Think 'Tensegrity)


Flexion: hamstrings (rest of back of lower extremity)
Extension: ligamentous capsule, iliacus, iliopsoas (anterior fascias of the lower
extremity)
Abduction: adductor group (medial fascias of LE)
Adduction: tensor fascia lata, ITB, and abductors
Rotation: fibrous capsule (piriformis & other external rotators; internal rotators
Limitation must be assess in position of function to best understand its contribution
to a patient's complaint.

Diagnosis & Treatment principles for muscles


Diagnosis
Feel for nature of resistance at end-range
Abrupt
Gradual, elastic
Test strength against resistance
COMPARE SIDES
Treatment

Tightness: End-Range ME
isometric muscle energy stretch

Treating muscle groups? tight or weak first?


SUMMARY:
We are looking for limited range of motion (and the quality of that motion, esp. at
end-range).
If one muscle group is tight (contracted),
the motion that stretches it is decreased.
The muscle that opposes it may be weak (inhibited).
Take Home Message: Stretch tight muscles first, then strengthen weak muscles.

Postural and Phasic


Classifications for muscles: DIFFERENCES?
Tonic (or postural) muscles tend to become tighter with dysfunction.
Phasic muscles tend to become weaker with dysfunction.
These changes result from both simple reflex mechanisms and supraspinallymediated mechanisms.

Hip Flexors: major functions of iliacus


-Psoas
-Iliacus-Inside pelvic brim- Attach to lesser trochanter as iliopsoas tendon

Actions:
Flex thigh on pelvis
Raise trunk from recumbent position
Balance the trunk in sitting
Major player in creating anterior rotation of the innominate

these have poor mechanical advantage for influencing the sacro-pelvis


Rectus femoris (quadraceps)
Sartorius
crosses hip and knee
Adductor longus
Adductor brevis

Hip Extension Testing -Thomas Test


Pt. Supine - pelvis close to the end of the table
Both hips and knees are flexed
Contralateral LE is flexed knee to chest
Helps maintain lumbars flat on the table
Ipsilateral LE drops over the edge of the table
Normal range: Back of thigh strikes table

Hip Extension Restriction Treatment


(Hip Flexor Tightness)
Strength Testing:
Pt. Attempts to lift knee toward ceiling against operator resistance
Pt. Attempts to press foot & ankle medially against operator's leg.
Tightness - Treatment: Isometric contraction against operator resistance, Relax,
Reposition, Repeat, Retest.

'Iliacus Test' & Treatment


Pt. Supine - hip at edge of table
Cephalad Hand:
-Contact ASIS Force directed Posterior & Superior
-Caudad Hand:
Palmar contact just above the knee
Let gravity carry lower extremity to its end-point; assess
Add gentle pressure toward the floor to assess end-range resiliency
Normal Range:
Lift the pt's lower extremity back on to table after test
Tight Muscle/Restricted Range:
Direct Stretch with or without Muscle Energy

Iliacus Tender point


Location:
medial and inferior to the ASIS several cm
Treatment:

Cross ankle of involved LE over the other ankle


Flex, externally rotate
Frogleg Position
Rest ankles on physician's thigh - foot must be up on the table

Legg-calve-perthes disease. Which hip restrictor is most likely Tight/


restricted Anteriorly in legg-calve-perthes disease?
Avascular necrosis of the femoral head
WHICH ONE?
Illiacus

30 Degree Abduction Flexion Restriction


Pt. Supine:
Stand to the side of the patient so that the ASIS and the knee can easily be
contacted.
Bend the patient's knee until the arch of the foot is even with and nearly touching
the medial malleolus.
Use the cephalad hand to hold the ASIS in place - usually with a posterior, slightly
superior and medial force.
Place the caudad hand just above the knee and carry the lower extremity into
external rotation to its end-range
Compare sides, if the restriction is not obvious
Treat with Muscle Energy.
This addresses the anterior aspect of the capsular ligaments.

30 Degree Abduction Flexion Restriction


Home Exercise - Standing
Pt. Standing with foot of uninvolved side about 8-12 inches from the side of the door
frame.
Have them lift the heel of the invovled side up onto the medial malleolus of the
other lower extremity [This puts the lower extremity in about the same position
used supine on the table.]
The above should be done such that the knee comfortably contacts the door frame.
They next reach through the doorway with the hand on the involved side to grasp
the door frame
The other hand does not reach through, but contacts the door frame as well.
The two hands can now twist the whole body away from the knee, thus creating a
stretch in the same area of the anterior hip.

30 Degree Abduction Flexion Restriction


Home Exercise - Supine
Pt. Supine
The patient bends the knee until the arch of the foot is even with and nearly
touching the medial malleolus.
The patient brings the arch of the opposite foot up against the thigh very near the
knee.
The patient can let gravity and the weight of the opposite leg stretch this or actively

press with the opposite leg.


Many people are not flexible enough to do this variation effectively.

Hip Flexion/ Oblique Adduction


Think PIRIFORMIS & related fascias

Hip Flexion/ Oblique Adduction


Pt. Supine:
Flex the hip just past 90 degrees.
Then adduct, not toward the opposite ASIS, but toward the opposite chondral mass.
Compare sides.
The symptomatic side is usually tighter
Direct stretch or Muscle Energy

Hamstrings
I find the hamstrings on one side are often overstressed because the innominate is
rotated anterior on that side. This lengthens the hamstring on that side and makes it
more vulnerable to injury with activity, like soccer or dance or any other vigorous
activity with a lot of change of movement of the lower extremities.
First treat the cause. Often that is increased tone and tension in the ipsilateral
iliacus muscle. Then balance the pelvis including the anteriorly rotated innominate.

Hamstrings
Home Stretch
Patient Standing:
Puts heel up on a step or a stool or something else that does not feel like it
immediately puts a lot of tension on the hamstrings.
Find 'Pelvic Neutral' and maintain that with flexion around the hip joint.
To further help keep stress out of the low back, have the patient press the palms of
both hands against the proximal anterior thigh.
This creates a fulcrum to lift the axial spine, support it and focus the stretch into the
hamstrings.
The patient flexes until he/she engages resistance, not pain; continues to lift the
upper body via the hand contacts.
Hold 20-30 seconds. Repeat once or twice. Do every other day for the first week or
two. Then 5 out of 7 days per week after that. Compare with the uninvolved side at
least weekly. No need to get it any more flexible than that other side.

Adductors: NAME SOME Testing


End Range & Strength Against Resistance
All cross hip joint
Most arise somewhere on the pubic bone

Pectineus
Adductor longus
Adductor brevis
Adductor magnus
Gracilis
Descends beyond knee
Actions:
Femoral adduction
Aid in gait
Partial controllers of posture
Magnus and Longus also medially rotate thigh

Abductors NAME SOME Testing


End Range & Strength Against Resistance
Tensor Fascia Lata
Gluteus
maximus
medius
minimus
All attach somewhere on the ilium
All 3 have a role in extending the hip

Hip Extensors
Hamstrings
Biceps femoris
Semitendinosis
Semimembranosus
Gluteus maximusGluteus medius
Span hip and knee joints
All attach proximally somewhere on ischial tuberosity

Actions:
Flex knee / extend hip
Pull trunk upright against gravity
Forward reaching, swaying or bending evoke immediate, strong contraction in
hamstrings
Sometimes length can be short and limit flexion at hips with knees extended

Hip Flexion Testing and Restriction Treatment


: (Hip Extensor Tightness)
Tightness at End-Range:
Keep the knee bent a few degrees; that protects it.

External Rotators
Piriformis m.
Obturator internus m.
Obturator externus m.
Superior gemellus m.
Inferior gemellus m.
Quadratus femoris m.
Gluteus maximus
Gluteal medius
Sciatica, Hip Pain, Gluteal Pain

Neutral Internal Rotation


Testing & Treatment
(Tight External Rotators)
Pt. Prone - feet off end of table
Medial Hand: Palmar surface contacts sacrum
Lateral Hand: Palmar surface on the lateral gluteal musculature posterior to the
greater trochanter
Press anterior with lateral hand, carrying greater trochanter toward table
Medial hand stabilizes with counterbalancing pressure
ME or direct stretch work well
End-range is often decreased and lacking resilience when dysfunctional
Other approaches use a bent knee for leverage. OK if there is not knee problem.

Piriformis Tenderpoint
Location: middle of the gluteal region; in the body of the piriformis muscle
Treatment: approx. 135 degrees of flexion with abduction

Internal Rotators of the Hip


Gracilis
Adductor magnus
Adductor longus
Other actions
Gracilis both flexes leg and internally rotates
All 3 have a role in femoral adduction

Neutral External Rotation Treatment


(Tight Internal Rotators)
Cephalad Hand: Contacts the ASIS from its lateral side
Caudad Hand: Contacts the greater trochanter anteriorly
The two hands push away from each other to create an external rotation stretch at
the hip joint
Often a factor in the geriatric population with hip pain

Osteitis pubis
Inflamed pubic bone
Self-limited painful inflammation of the pubic symphysis
Symptoms: gradual onset with tenderness over symphysis pubis which radiates
along adductor and rectus abdominus muscles
Stress associated with athletics

Tensor fascia lata Tenderpoint


Location: 12 cm below greater trochanter along lateral surface
Treatment: ABDUCTION

Gluteus Medius and Minimus: Gluteus Medius Tenderpoint

Attached to external iliac surface


Attach distally to greater trochanter
Actions:
Abduct thigh
Internal rotation
Keep trunk upright when opposite foot is raised in walking and running
Active in Romberg's test
Point location: below iliac crest along upper portion of gluteus medius
Treatment: extension, internal rotation, slight abduction

What are the most commonly involved 'hip restrictors'?


Hip flexers - Iliacus
30 degree Abduction Flexion
Piriformis - Hip Flexed 90 degrees - Oblique Adduction
Adductors
Hamstrings
Abductors

Pubic Bone:
attachments
Superior- Rectus abdominus
Inferior- Adductor
Internal: puborectalis, pubococcygeus

SOAP notes:
Biomechanical:
MANY important muscular origins and insertions(from immersion!) from trunk
and lower extremity
Transition zone! Lumbo-sacral junction; iliopsoas as bridge btwn diaphragms (abd
and pelvic)
Visceral:
Distal GI issues: sigmoid colon, rectum
GU issues: Urinary incontinence, prostate inflam/hypertrophy, Baby-carrying,
cycles of menstruation
Vascular, Fluid:
common, int/ext iliac a; femoral a; lymphatic congestions
Neuro:

end of sympathetic chain


pelvic splanchnics, p'symp sacral plexus

pocket pelvis

Landmarks - static
Level of the PSIS
Gluteal folds: height, depth
Greater trochanters: height
Ischial tuberosities: height

Standing flexion: how to do it


Motion: standing flexion
Monitor: PSIS
Where?
Active patient motion: "reach for your toes"
Restriction on one side leads to premature locking at the SI joint.
This PSIS elevates SOONER, moves FIRST.
Positive test: dysfunction is named for side that moves.
Equal motion?- negative test
No dysfunction
Symmetrical dysfunction

ASIS segmental definition: name a motion test too


Thumbs immediately inferior to both bony prominences
A/P; S/I; closer or further away from belly button (I/E)

Motion (supplemental)
Compression test. Ease of motion in the A/P planes is noted with alternating,
rocking pressure

Pubic tubercles
Position
Find the belly button, glide base of hand inferiorly until you bump in to the pubic
bone.
Ensure finger pads rest on cephalad aspect of pubic bone.
Asymmetry is named for side of standing flexion dysfunction

Seg.Def. supine: MM
Compare relative position of the malleoli

Seg.Def. Prone PSIS. what ligament do you look at ?

Position: compare heights -S/I


Ischial tuberosities
Position: compare cephalad, caudad
Sacral Tuberous Ligament
Tight?
Lax ?
Sacral sulci
Position: compare depth- A/P

Pubic bone dysfunction


pubic bone superior or inferior to the other

Upslip
Are all landmarks superior or inferior to their contralateral partners

Stork test: how to do it


Pt standing
Monitor: one thumb at PSIS; one thumb at sacral crest (get a picture of where that
is)
Active patient motion: "raise knee (ipsilat to monitored psis) as high as is
comfortable"
Repeat both sides, noting the motion PSIS relative to sacral crest.
PSIS descends relative to sacral crest: negative test
PSIS moves Superior, or does not move relative to sacral crest: dysfunction=positive
test!

sacrum differentials? why look at sacrum?


Low Back Pain
Trauma
Whiplash
Falls
GI complaints
Parasympathetics to the lower GI tract
Postoperative Ileus
Irritable Bowel

GU complaints
Parasympathetics to bladder and prostate
Urinary incontinence
Urinary Retention
Gynecologic/Obstetrics
Parasympathetics to uterus
Dysmenorrhea
Changes during pregnancy and childbirth
Headaches
Cranio-sacral motion

Sacral Development
Developed from fusion of 5 vertebral segments, S1-S5
These are the 25th-29th of 33 vertebrae
Ossification begins in the 1st year of life
Complete ossification occurs between 25-30 years old

Sacral Anatomy: Anteriorly


Triangular shaped bone between innominates
Concave anterior surface
8 foramina (4 pairs) lie between the sacral vertebrae

Anterior sacral nerves exit through the foramina on anterior surface


Used as reference when naming torsions

Sacral Anatomy: Posteriorly


Convex posterior surface
Medial ridge created from spinous process rudiments
Central opening around S4-S5 forms sacral hiatus (arrow)
8 foramina allow passage of posterior sacral nerves

Sacral Articulations
Laterally
Innominate to form sacroiliac (SI) joints
Cephalad
5th Lumbar Vertebra via intervertebral disc
Caudad
Coccyx
3

base, sacral sulcus, ILA, apex

Name all important ligaments of sacrum TRUE


3 True: Sacroiliac
Posterior SI ligament
Note the orientation to
support weight bearing
Anterior SI ligament
Interosseous SI ligament
Unpictured

Name false ligaments of sacrum


3 False
Sacrotuberous Ligament
Sacrospinous Ligament
Divides greater and lesser
sciatic foramen
Ileolumbar Ligament

Sacral Anatomy
Muscular Attachments ANTERIOR AND POSTERIOR
Anterior attachments:
Iliacus
Coccygeus
Piriformis
PIC
Posterior attachments:
Gluteus Maximus
Erector spinae
Latissimus dorsi
GEL

Multifidus

sacral motion all axes


3 Transverse axes
Superior, Middle, and Inferior
2 Oblique axes
Right and Left
1 Longitudinal axis
1 Anterior-posterior axis

Transverse Axes
Superior: Respiratory and Craniosacral axis
Located at approximately S2, at the location of dural attachment
Middle: Postural axis
Bilateral Flexion & Extension occur around this axis
Inferior: Innominate rotation axis

Sacral Motion: Nomenclature


Sacral extension
Base moves posteriorly
In inhalation phase, sacral base moves posteriorly (AKA extends or counternutates)
Sacral flexion
Base moves anteriorly
Occurs during exhalation, sacral base moves anteriorly (AKA flexes or nutates)

Sacral Motion: Oblique Axes naming- when is dynamic motion


important
Dynamic Motion: physiologic
Occurs during ambulation
Weight bearing on Left leg will engage left axis and vice versa
Right and Left
Named for the side of the upper pole of the oblique axis!!

palpation landmarks for sacrum

palpation landmarks for sacrum


Palpation Nomenclature
Anterior and Deep
When the sacral base moves anterior the sulcus/sulci can be said to be anterior or
deep.

Posterior and Shallow


When the sacral base moves posterior the sulcus/sulci can be said to be posterior or
shallow.

screen scan and seg. def. for sacrum ALL VARIATIONS


Screen
Scan
Perform the Seated Flexion Test
Segmental Definition
Forward Torsion
Right on Right (R on R)
Left on Left (L on L)
Backward Torsion
Right on Left (R on L)
Left on Right (L on R)
Unilateral Sacral Flexion
Unilateral Sacral Extension
Bilateral Sacral Flexion
Bilateral Sacral Extension
R rotation on R forward torsion- L5 NSRRL

dynsfunction patterns. which one has L5? which one is due to sacral
shear?
Sacral Torsion
Rotation around an oblique axis with somatic dysfunction at L5
Unilateral Flexion/Extension
Flex/Ext around a transverse axis at only one SI joint
Due to sacral shearing
Bilateral Flexion/Extension
Flex/Ext around a middle transverse axis at both SI joints

L5 in torsions? RULES TO DIAGNOSING L5


In a torsion, L5 Sidebends into the axis, Rotates opposite sacrum
Rule #1: When L5 is sidebent, a sacral oblique axis is engaged on the same side as
the sidebending.
Rule #2: When L5 is rotated, the sacrum rotates the opposite way on an oblique
axis.
Rule #3: The seated flexion test is found on the opposite side of the oblique axis

L5 will always rotate in the opposite direction of the sacrum


L5 will always be sidebent towards the side of the sacral oblique axis (when normal
mechanics are operating; trauma can change that)

Left on Left (ROTATION ON ___ AXIS)


Right on Right
WHEN SAME- FORWARD
WHEN OPPOSITE- BACKWARDS

With Bilateral Sacral Flexion or Extension


seated flexion test is Falsely Negative and the Sacral Sulci appear equal in depth.**
Therefore, another diagnostic modality must be used
The Spring Test
SPHINX TEST

Spring Test
Classic Test:
Anterior pressure at the sacral sulci
Normal: should spring/have resiliency
Positive Test: Base is resistant, does not spring, feels like concrete
Meaning: Base is stuck in extension or posterior (it will not go into flexion)
ILA testing:
Positive test: ILAs are resistant to anterior pressure, do not spring

The Diagnostic Process - Scan


Seated Flexion Test
-Pt. Seated & feet flat on the floor
-Operator - thumbs on inferior slope of PSISs
-Pt. Forward bends with arms between the knees
One PSIS moves further than the other
More Cephalad or Ventral
This is the Side of the Sacroilial Dysfunction

Transverse Axes what happens at each one


Superior: Respiratory Axis
Motion relative to the pull of the dura occur around this axis
Craniosacral (Inherent) motion occurs here.
Located at S2
Middle: Postural Axis
Bilateral flexion and extension occur around this axis
Located at anterior convexity of upper and lower limbs of SI joint
Inferior: Innominate Rotation Axis
Located at the posteroinferior part of the lower limb of the SI joint

Screen Sacro-pelvis 2 motion, 2 tissue texture

Standing Flexion test


Standing flexion occurs at the hips and is translated from the innominate into the
sacrum via the middle transverse axis.

Therefore a positive standing flexion test is most likely due primarily to innominate
dysfunction.
Iliosacral

Seated Flexion test


What stabilizes the innominate?
Seated, the innominate is somewhat stabilized by the ischial tuberosities on the seat
and flexion occurs through the spine and translated from the sacrum into the
innominate via the middle transverse axis.
Therefore positive seated flexion test are most likely due primarily to sacral
dysfunction
Sacroilial

The Diagnostic Process - Scan


DEEP = ANTERIOR
SHALLOW = POSTERIOR
sacral base
PSIS:
Have your patient lie prone
Follow the iliac crests posterior to their limit - the bottom of the PSIS.
Move thumbs superiorly onto the main part of the PSIS, its posterior prominence.
SACRAL SULCUS
Move thumbs medially from the PSIS a thumb breadth - thumbs now over sulci.
Use your "awesome" palpatory skills to determine if one sulci is deeper or more
shallow than the other
Follow the spines of the sacrum inferior until your index finger drops into a 'hollow',
the sacral hiatus.
The posterior aspect of the ILA is just 2-3 cm lateral to and on either side of the
sacral hiatus.

Bilateral Sacral Extension


Bilateral Sacral Flexion
Improper lifting techniques
Fall in seated position

Patient can't backward bend or it is very painful.


Extremely common postpartum
Arched with a heavy load
Patient can't forward bend or it is very painful.

The Diagnostic Process: 'Sphinx Test'


Extend bacl flex sacral basePatient sulci depths are tested neutral prone first
Patient assumes the prone- prop position (sphinx)
Lumbars extend, sacrum flexes
Negative Test: Decreased sacral base asymmetry= sacral flexion ease
Positive Test: increased sacral base asymmetry = extension ease

+ spring test at Left ILA

L5 N SLRR
treatment- what muscles do you engage
Left on Left Torsion
Rotating the axial spine to the left (restriction) puts the sacrum into right rotation
(restriction: L rotation on L axis)
Dropping the legs off the table gaps the SI joint
Patient is positioned with + flexion test side up
With ME, the patient is activating the piriformis and engaging the ligaments in the
sacropelvic region

Left on Left Torsion TXT sequence


Patient Prone
Doctor on opposite side of axis
With the patient's hips and knees flexed to 90, roll the patient onto their hip such
that the axis is DOWN
Physician Monitors at Lumbosacral junction
Physician supports patients knees (with thigh) and lets ankles drop to floor
Continue to monitor lumbosacral jn
ME Treatment: patient pushes ankles to ceiling while physician restricts motion
Patient relaxes, take up the slack, then the ankles are allowed to drop further to
floor
Perform 3-5 times

Localize: may need to readjust flexion/extension after muscle contraction in order


to bring the forces back to the right sacral sulcus (sacral base)

Right on Left Torsion


Backward Torsion
+ spring test at R sulcus,
+ sphinx test (ease on extension although sacrum should flex)

Right on Left Torsion


-Patient in lateral recumbent position with axis DOWN
-Physician uses caudad hand to flex the knees until motion is felt at the lumbosacral
junction
-Physician pulls patient's arm closest to the table to rotate patient while monitoring
L5
-Patient should be on back
-Patient's bottom leg is extended until force is felt to localize to the right sacral base.
-Patient's top leg is dropped of the table in front of other leg.
-ME Treatment: patient pushes leg (ankle) toward the ceiling while physician
restricts motion
-Patient relaxes and the leg is allowed to drop further to floor; readjust other
elements, as needed.
-Perform 3-4 x's

What is happening during treatment?


With patient supine and the hips rotated with the axis down...
causing sacral motion to the left (motion that is restricted)L5 rotation occurs to
the right
Furthermore...
Extension of the left leg during treatment positioning causes further flexion of
sacrum (i.e. restrictive barrier of sacrum)
With ME, the patient is activating the piriformis and is also engaging the ligaments
around the sacral pelvic region to allow for sacral rotation
For backwards torsion...Lay patient on their "Back"
Remember, axis down on the table!

Left Unilateral Flexed Sacrum


+ Spring test on Left ILA

Sacral Movements and Respiration:


causes sacral extension (base goes backward)Inhalation
causes sacral flexion (base goes forward)Exhalation
Therefore...
We can use the phases of respiration to treat unilateral sacral lesions!

Treatment - Left Unilateral Flexed Sacrum- DONT FORGET TO USE


BREATH
Patient Prone
Physician on side of dysfunction
Knee is flexed to 90 degrees, and the leg is abducted and internally rotated.
This causes gapping at the SI joint
Physician monitors at SI Joint
Leg is stabilized by physicians chest
Caudad hand on the ipsilateral ILA, cephalad hand monitors SI joint
Avoid the coccyx with right hand!
Physician engages restrictive barrier by placing anterior, superior pressure at the
ILA
With the barrier engaged, patient assists by inhaling deeply to move sacral base
posteriorly and holds the breath in while physician continues with pressure at ILA.
After cycle, physician engages new barrier at ILA and another deep breath is held
Repeat 3-5 x's

What is happening during treatment?Left Unilateral Flexed Sacrum


With left unilateral flexed sacrum, the sacral sulcus is anterior and the ILA is
posterior.
The left leg is abducted and internally rotated to gap SI joint to allow for movement
of sacrum
Anterior, superior pressure at left ILA causes the left side of sacrum to extend (into
its restrictive barrier)
this causes increased sacral extension at the basePatient augments this by inhaling
Pressure at the left ILA during exhalation (between held inhalations) prevents the
sacrum from flexing

Low Velocity, Moderate Amplitude (LVMA) springing could be used to augment this
treatment.

Left Unilateral Extended Sacrum


(+) Sphinx Test

...
Patient prone
Physician on side of dysfunction
Physician places caudad hand on the inferior portion of left ischial tuberosity and
cephalad hand on left sacral sulcus
Ischial tuberosity is carried superiorly toward sacral base
Left sacral base is carried anterior and inferior to the restrictive barrier
Physician uses Low Velocity, Moderate Amplitude (LVMA) springing with both
hands into the restrictive barrier

Unilateral Extended Sacral Dysfunction WHAT IS GOING ON?


In a left unilateral extended sacrum, the left sulcus is posterior and the left ILA is
anterior
Pressure is held on the ischial tuberosity to keep the innominate from rotating and
moving inferior during the activating force
Springing at the left sacral base forces the left sacral sulcus into flexion (the motion
that is restricted)
Springing is continued until restricted barrier is passed (softening of resistance
occurs & motion is more like the other 'normal' side.

Bilateral Sacral Dysfunctions


Bilateral Flex Sacrum
Bilateral Extended Sacrum
Note: movement occurs around the middle transverse axis

Treatment - Bilaterally Flexed Sacrum


(-) spring test at sacral sulci
(+) spring test at sacral ILA

Treatment - Bilaterally Flexed Sacrum- LEVER


Patient seated
Physician places caudad hand on sacral apex and cephalad hand on thoracic region
Patient flexes trunk until resistance is felt at the sacrum by the physician
Patient actively extends trunk, while physician resists motion with cephalad hand
Patient holds extension for 3-5 sec then relaxes.
As patient relaxes, pressure is continued on the sacral apex, and the patient flexes

until a new barrier is reached


Repeat 3-5 x's

What is happening during treatment?


As the patient initially flexes the trunk, the sacrum becomes extended bilaterally
This takes the sacrum to the restricted barrier
As the patient tries to extend, this causes the sacrum to flex bilaterally
The physician uses a counterforce at the thoracic region, which causes isometric
contraction at the lumbosacral junction
As the patient relaxes, the trunk is able to flex further and increased sacral
extension is able to occur
HAVE THEM INHALE to modify

Bilateral Sacral Extension


(+) spring test at sacral base
(-) spring test at ILA's

Treatment - Bilaterally Extended Sacrum


Patient seated
Physician places caudad hand at sacral bases, and cephalad hand around patient's

chest
Patient is asked to extend trunk until restricted barrier felt at sacral base
Patient is instructed to flex trunk as physician restricts movement with cephalad
hand. During flexion, physician also uses caudad hand to restrict sacral extension
Patient holds contraction for 5 sec
As patient relaxes, the trunk is furthered extended until a new barrier is felt at the
sacral base
Repeated 3-5 x's.

What is happening during the treatment?


With a bilaterally extended sacrum, the sacrum prefers extension
As the patient extends the trunk, the sacrum is forced into flexion
This is the restricted barrier
Then, as the patient flexes, the sacrum starts to extend bilaterally
The physician uses a counterforce at the sacral base to prevent sacral extension.
As the patient relaxes, the patient can further extend the trunk and cause an
increase in sacral flexion

tricks
Dysfunction on an oblique axis:
1. Determine axis and place axis down
2. Determine forward/backward torsion (Pt on chest if forward, back if backward)
3. Position legs accordingly (Front 2 Back (1) )

OMM

Biomechanical
Fluid
Nervous system
Visceral

1. Relevant Biomechanical Concerns:


(torques, twists, strains, undue pressure anywhere in the musculoskeletal system
Delineate the local and regional biomechanical concerns that may impact the
patient problem.
Delineate the inter-regional biomechanical concerns that may impact the patient
problem.
Suggest possible Osteopathic manipulative interventions that may assist in
normalization of biomechanical influences associated with the patient problem

2. Relevant Concerns with Fluid Congestion (venous & lymphatic, in


particular):
Delineate the drainage pathway for the area, viscus, joint, or other component
relevant to the patient problem. Follow this to its terminal drainage
Describe and portray the 'choke points' that may impede drainage along this path.
Suggest Osteopathic manipulative interventions that may assist in drainage of the
fluid congestion associated with the patient problem

Tissue Level:
There are many mechanisms for accumulation of tissue fluid.
They are dependent upon an inability of the drainage system to cope with excess
fluid.

Lymphatic Drainage for the Body


Lymphatics are the 'Overflow system' of the body.
Local drainage or lack thereof is only one component in the equation of fluid build
up.
**Key Principle: Any blockage Proximal to the excess fluid will inhibit its drainage.

3. Relevant Nervous System Concerns:


-SNS
-PNS
- Pain
Delineate the innervation for the area, viscus, joint, or other component relevant to
the patient problem. Follow its course to the spinal cord.
Describe the anatomic factors/influences along the course of the innervation that
can change SNS function.
Suggest possible Osteopathic manipulative interventions that may assist in
normalization of the sympathetic influences associated with the patient problem.

Sympathetic Nervous System

"Hypersympathetic activity is a common factor in disease."


Multiple influences upon the internal organs
Sympathetic fibers have a vasomotor function, among other things.
These fibers stimulate contraction of blood vessels.
Notice the location of the chain ganglia
Rib somatic dysfunction may be seen to change the pressure upon these ganglia.

Parasympathetic Innervation
REST IS VAGUS- EXCEPT IN HEAD - parasymp.

Common Denominator?
Key concept: All of these stuctures course through the myo-fascial (muscles &
fascia) system.
This suggests that Biomechanical dysfunction (somatic dysfunction) within the
musculoskeletal system can play a role in optimal or suboptimal function of the four
factors just mentioned.

L3-L5 locations
L3
Most anterior in the lordotic curve
Freely movable- has the most rotation and SB of the LV
L4
Iliac crest
Limited motion
L5
Small SP

Lumbar Motion
Flexion/Extension
Primary motions
Increases from above downward
Side Bending
Small amount
Evenly distributed
Rotation
Minimal

Coupled motion
Neutral and Non-neutral

Local Motion Testing scan lumbar how


Prone
Assess paired transverse processes in neutral position
Rotation
Side bending
Flexion/extension
Recall: seated (Can be done as was done Muscle Energy Dx)
Assess forward bending
FRS dysfunction if TPs positions normalize
Neutral group: not much change; doesn't normalize in any position

HVLA works on
ARTICU-LAR Crepitant to good
- reduced ROM
abrupt end feel
Impulse, ME, ROM, MFR, FPR, FM

Lumbar Treatment Sequence


Treat Non-neutral (F/E) Type II First
Then treat Neutral (Group) Type I
Dysfunctions
Logic: Presence of Type II dysfunctions result in compensatory Type I dysfunctions

Lumbar Treatment
Prepare the area with soft tissue
The set up is the same for all techniquesPosition in resistance
Muscle Energy [You know how to Do This ]
Patient Seated
Patient applies counter force
HVLA-Impulse
Localization
Patient Relaxation
Apply thrust

Soft Tissue
Pt prone
lateral distraction on contralateral erector spinae
Pull up on ASIS as you push anterior and lateral on contralateral erector spinae

Contraindications for HVLA Lumbar-Red Flags


Acute fracture
Infection
Rheumatoid arthritis
Downs Syndrome-congenital weakness of transverse ligament of atlas
Acute herniated disc with radiculopathy
Patient tolerance
Physician fear
HVLA thrust directed straight down -while rotating pelvis anterior- into the barrier

Keys to the Lumbar Roll


the line of the hips is clearly forward
This difference is important for efficacy of the thrust.
Transverse process is down-against the table
Gently rotate pt upper body away from you to position the affected segment
perpendicular to the table
Rotate whole body toward you until the hip is in front of the axis of rotation.
Refine localization in the three planes, as needed
Patient relaxation
HVLA thrust is straight down into the table
Apply a stabilizing counter force against the shoulder as you rotate the pelvis toward
you down toward the table
Avoid over flexion as you position the pt [for Neutral or Flexed Dysfunction]

To induce extension
Straighten the lower leg
Hyper flex the upper extremity

To induce flexion
Keep lower knee slightly flexed
Draw upper extremity slightly inferior

HVLA: Most Common Errors


Not enough force
Poorly localized force-proper positioning
Thrust without commitment
ALWAYS PUT PATIENT ON SIDE OF DYSFUNCTION

Supine Lumbar HVLA

Patient's hands clasped behind neck.


Slide hand under flexed elbow on side of lumbar rotation, dorsum of hand onto
sternum.
With patient's knees flexed, walk around to opposite side of table inducing rotation
of the trunk
Stabilize pelvis with hand on ASlS.
Localize rotation into lumbar spine
Initiate thrust by stabilizing pelvis and continuing rotation of the trunk and lumbar
region toward operator and toward the floor.
Reassess.

innominate
Illium
Ischium
Pubis
WE HAVE 2

How do you screen the pelvis?


TISSUE TEXTURE
-Lateral Posterior Gluteal Tissue
-Sacral Tissue
MOTION
-Lateral Translation
-Pelvic Rotation

scan the pelvis?


Standing Flexion Test (ilio-sacral)
*ASIS Compression Test

Standing Flexion Test


(Scan of Pelvis)
Place thumbs under PSIS
Ask pt. to slowly bend at waist
Watch for thumb movement (superior/anterior)
Side that moves first is the positive side

Standing flexion test


Performed when scanning the pelvis
For pelvic dysfunction
Indicates __ILIO-SACRAL__________ component of dysfunction

Seated flexion test

Performed when scanning the sacrum


For sacral dysfunction
Indicates _SACR-OILIAC__________ component of dysfunction

What landmarks do you need to segmentally define the pelvis?


Anterior: iliac crests, ASIS, pubic tubercles, medial malleoli*
Posterior: PSIS, ischial tuberosities
check motion at sacrotuberous ligament (*)

Anterior Muscles Attachments- what attaches to each landmark?


ASIS
Sartorius
AIIS
Rectus femoris
ILIAC CREST
Tensor Fasciae latae
Gluteus Medius
PUBIC BONES
Adductor Longus
Pectineus
Adductor Brevis
Gracilis
PSIS
Gluteus maximus
ILIAC CREST
Transversus abdominus
External Oblique
Quadratus lumborum
ISCHIAL TUBEROSITY
Quadratus femoris
Adductor magnus
Biceps femoris
Semimembranosus

Making a Pelvic Diagnosis


The standing flexion test was the scan of the pelvis
Checking landmarks is how you segmentally define the pelvis
Record your findings at each landmark with respect to the side of the (+) standing
flexion test
Example: (+) standing flexion test on the R
If you find that the L iliac crest is higher than the R, record that the R iliac crest is
lower
Record what the side of (+) standing flexion test is doing!
This makes your diagnosis much easier at the end
Put all the pieces together to make a diagnosis
R Anterior Innominate, L Superior Pubic Shear, etc.

pelvic dysfunctions
Anterior innominate rotation
Posterior innominate rotation
Superior pubic shear
Inferior pubic shear
Superior innominate shear ("Upslip")
Inferior innominate shear ("Downslip")
Inflare
Outflare

What is the origin and insertion of the Rectus Femoris?


Anterior Inferior Iliac Spine/acetabular roof of the hip joint and
the tibial tuberosity via patellar ligament

What is the action of the posterior thigh muscles?


Hip Joint: Extends the hip,
stabilizes the pelvis
Knee Joint: Flexion and
internal rotation

Anterior Innominate Rotation


Findings (e.g. Left Anterior Innominate)
+ standing flexion on left side
L ASIS inferior
L PSIS superior
L medial malleolus may be inferior
L sacrotuberous ligament loose
Normal Physiologic Motion (iliosacral)
Tight quads

anterior rotation loosens what ligament


sacrotuberous

Anterior Innominate ME Treatment


Cephalad hand- L ASIS
Caudad hand- L ischial tuberosity
Flex knee and hip
Adduct LE to gap SI
Instruct pt to push knee into chest (gently!!)
Repeat 3xs, recheck
Obj: rotate innominate posteriorly using isometric counterforce

Posterior Innominate Rotation


PSIS down
ASIS up
Medial Malleolus Up
ALLL on one side

Posterior Rotation: ME Treatment


Pt is supine
Cephalad hand on opp ASIS, caudad hand just above knee
Pt is instructed to resist into caudad hand (or bring leg up); pt is bringing
innominate anterior
Repeat, recheck
Reminder: sacrotuberous ligament is tight on the side of posterior rotation!

Inflare/Outflare
Rare*
Diagnosed after the correction of any other pelvic dysfunction
Inflare Findings
Ex. Right
Standing Flexion Test positive on the right
Medial ASIS on right
Outflare Findings (at left)
Ex. Right
Standing Flexion Test positive on the right
Lateral ASIS on right
Anterior rotation can be associated with inflare. Posterior rotation can be associated
with outflare.

Innominate Inflare
Muscle Energy Treatment

Ex: Right
Doc: Flex R hip and knee, Abduct R hip to restrictive barrier while stabilizing L
ASIS
Pt moves knee toward midline while doc resists
Repeat, add further abduction, recheck

Innominate Outflare
Muscle Energy Treatment
Ex: Right
Doc: Flex R hip and knee, Adduct R hip
Doc's L hand monitors at R PSIS
Pt moves knee out while doc resists
May offer more adduction, repeat and recheck
*Opposite treatment as inflare*

Upslip, Right- HOW DOES THIS HAPPEN?


Step Off, Car accidents, Fall landing on buttocks
Landmark findings:
+ Right standing flexion
Iliac crest: R
ASIS: R
Pubic bone: R
Med Mal: R
PSIS: R
Ischial tuberosity: R
ST lig: lax on R

Inferior Shear (Down Slip)


Foot entrapment HOW DOES THIS HAPPEN?
Inferior Shear (Down Slip)
Foot entrapment
Landmark findings
Iliac crest: L
ASIS: L

Malleolus: L
PSIS: L
Ischial Tuberosity:L
Pubic bone: L

Pubic Shear- caused how?


Superior Shear
More common of the two
Typically caused by standing with weight unevenly distributed or by muscle
imbalance
Findings:
+ standing flexion test
Ipsilateral pubic bone superior
+/- Tension and tenderness of ipsilateral inguinal lig.

Inferior Shear
Typically caused by muscle imbalance
Findings:
+ standing flexion test
Ipsilateral pubic bone inferior
+/- Tension and tenderness of ipsilateral inguinal lig.

H/P for Low Back Pain


History
Physical Exam
Differential Diagnosis
Lab & Imaging

Differential Diagnosis for LBP


Back strain
Acute disc herniation
Osteoarthritis
Infection
Malignancy
Spinal stenosis

BIOMECHANICS of Low back relationships


Bony
Ligamentous &
Muscular elements
Where you have joints you have the possibility for somatic dysfunction.

'Foundation' for trunk function


Sacroiliac & Pubic Joints are key components in the integrity of force distribution
from above and below.

Pubic Joint Function:


=Fibrocartilaginous joint
-Cartilaginous on each pubic bone at the joint
-Fibrous disc in between
=Motions Allowed are Small
Superior/Inferior
Anterior/Posterior
=Important for 'Shock Absorption'

Sacroiliac Joint Function:


Determined by:
-Joint shape and bony leverage
-Ligamentous and capsular integrity &/or strain
-Muscular pull across or around the joint

Sacroiliac Joint Function:


Each joint moves in relationship to the shape of the S-I joint

Sacroiliac Joint Function: HOW IS THIS A UNIVERSAL JOINT?


-3 functional axes are used to describe most common motions and associated
somatic dysfunction.
--They are not the only possible motions.
--This is, in reality, a 'universal joint'
What does that mean?
It can allow small motions in any direction.

important anterior ligaments

important posterior ligaments

Muscular Anatomy: Hip Restrictors:#1 - other important muscles too


Iliacus #1!
Hamstrings
Adductors
Piriformis
OTHER
Quadratus Lumborum
Psoas minor and major
Muscles of Axial Spine Thoracic & Lumbar

Attachments of all important muscles

Quadratus lumborum: from iliac crest to rib 12 & attaching to lumbar vertebrae
Psoas minor and major: from L-T junction to lesser trochanter
Iliacus: inside of ilium, capsule of S-I joint & edge of sacrum to lesser trochanter
Hamstrings: ishcial tuberosity across knee joint
Adductors: Pubic bone to femur
Piriformis: anterior edge of sacrum to greater trochanter

Lower Half of the Body


Diagnosis & Treatment Sequence
1. Hip restrictors
2. Pubic bone dysfunction
3. Superior innominate shear ("innom. upslip")
4. Lumbar dysfunction (esp. L5 or, less often L4)
5. Sacral dysfunction
6. Innominate dysfunctions (other than upslip)
7. Iliopsoas (including thoraco-lumbar junction, approx. T11-L2)
8. Other Lower Extremity Dysfunction

Iliacus importance on rotation of pelvis?


The arrows suggest the leverage of the iliacus muscle on the axis of motion for the
innominate.
The leverage of the hamstrings could be a significant, but in the upright position it is
more directly under the axis.

what can contribute to sacral dysfunction of the innominate?

Piriformis can contribute biomechanically to sacral dysfunction

Piriformis origin and insertion


Piriformis: Arises from the anterior surface of the sacrum;
If hypertonic or contractured, can create a relatively fixed pivot around which the
sacrum must move.

Adductors: significant attachment to what?


Significant attachment to pubic rami, thus hypertonic or contractured adductor
could contribute to dysfunction.

Pubic Dysfunction Influences S-I Dysfunction: pubic joint is important


too...
Pubic Joint's lack of mobility affects affects S-I function
Less shock absorption through the pelvis

3. Superior Innominate Shear


This is non-physiologic.
It is not gliding within the 'L' shape.
Puts a lot of stress on the ligaments
Traumatic in nature
No other S-I motion will readily occur when this is present.

4. Lumbar Dysfunction: name ligaments involved


Imagine L5 FRSR:
That is going to put tension on the sacral base via the annulus of the disc and via the
anterior and posterior longitudinal ligaments
On the innominate via the iliolumbar ligaments

5. Sacral Dysfunction: THE FOUNDATION OF THE REST OF THE


AXIAL SPINE
Subtle motions of this bone are required for the normal motions of walking,
bending forward, arching & more complex motions (sports, dance, work)
The sacrum is the foundation for the rest of the axial spine and its function.

6. Innominate Dysfunction: RESTRICTIONS


Physiologic motion restrictions:
Anterior/Posterior Rotation
Inflare/Outflare
Stress the S-I joint.
Upslip is not a physiologic dysfunction, therefore not listed here.

7. Iliopsoas (L-T Junction)- muscleS? bones?


-Iliopsoas (including thoraco-lumbar junction, approx. T11-L2 & ribs 11 -12)
-Usually compensatory, therefore one of the last areas to diagnose and treat

8. Other Lower Extremity Dysfunction


Knee, ankle, foot:
If these areas are unstable or strained, they affect the sacro-pelvis - the foundation
for the rest of the body.

If acute, directed to that area by the patient's complaint


-The other components of the 'Lower Half of the Body Diagnosis and Treatment
Sequence' may affect a lower extremity complaint.
-If chronic may be part of an underlying pattern contributing to the patient's chief
complaint.
so extremity affects pelvis- acute, treat knee- chronic- may be part of pattern

DYSFUNCTION TYPES
Myotonic Dysfunction: ROM?
articular dysfunction: end feel?
neuromuscular: tissue texture?
Myofascial: ROM?

PAIN!! PATHWAY!
-Nerve pathways are multilane freeways
-Pain fibers follow the course of the mixed spinal nerves
-Facilitation in the nervous system will amplify the experience of pain
-Local inflammation will prolong the pain fiber stimulation.

Lymphatic Drainage - Relevant to UTI


Lymphatics are the 'Overflow system' of the body.
Local drainage or lack thereof is only one component in the equation of fluid build
up.
Key Principle: Any blockage Proximal to the excess fluid will inhibit its drainage.

Is lymphatic drainage for the urinary bladder and the urogenital diaphragm
restricted?

visceral sympathetic cord levels


Biomechanical influences
SNS, PNS, Pain pathway influences
Fluid Drainage influences

visceral parasympathetic
Biomechanical influences
SNS, PNS, Pain pathway influences
Fluid Drainage influences

Key Components of the History


Onset
Location
Type of pain
Trauma history
Medical history
Psychosocial stressors
Red flags: weight loss

Physical Exam components


Comprehensive physical exam:
Observation
Gait & Stance
Mobility

Neurological Exam
SLR
Reflexes
Motor or Sensory Deficits
GI or GU, if indicated
Osteopathic examination

Diagnostics:
Lab tests and imaging

RED FLAGS! IN HISTORY


Cancer
Unexplained weight loss
Immunosuppression
Prolonged use of steroids
Intravenous drug use
Urinary tract infection
Fever
Pain that is increased or unrelieved by rest
Significant trauma related to age
Bladder or bowel incontinence
Urinary retention (with overflow incontinence)

RED FLAGS! IN PHYSICAL EXAM


Saddle anesthesia
Loss of anal sphincter tone
Major motor weakness in lower extremities
Fever
Vertebral tenderness
Limited spinal range of motion
Neurologic findings persisting beyond 1 mo.

why might you take an X-ray?


After 65 years of age, cancer, compression fractures, spinal stenosis, and aortic
aneurysms become more common.
Therefore, early X-ray is indicated in elderly patients.

Management
NSAIDS
ICE
Heat
Short term narcotic use
Activity assessment
Physical therapy
Manipulation

Board Review Chart

Important joints of hip and important ligaments


Joints:
Pubic Symphysis
Hip Joint-Acetabular-femoral
Sacro-iliac Joint
Ligaments
Iliofemoral- Strongest ligament in body.
With lower extremity extension - pulls innominate into anterior rotation - mid to
late stance phase
Prevents overextension of hip when standing
Pubofemoral
Anterior and inferior reinforcement
Ischiofemoral
Posterior reinforcement- prevents hyperextension

Hip Joint Funciton


Standing
Foundation for axial skeleton, ribcage and upper extremities
Transition from bipedal stance and motion to a central spinal column
Support for the abdominal & pelvic contents

Walking
Superior-Inferior Translatory Motion of the pelvis
Rotational Motion Left & Right
Sidebending / shift Left & Right

Hip Ranges of Motion


Flexion: 120-135
Extension: 30
Abduction: 45
Adduction: 20-30
Internal Rotation: 35
External Rotation: 45
VARIES IN REAL LIFE

Limiting structures (Think 'Tensegrity)


Flexion: hamstrings (rest of back of lower extremity)
Extension: ligamentous capsule, iliacus, iliopsoas (anterior fascias of the lower
extremity)
Abduction: adductor group (medial fascias of LE)
Adduction: tensor fascia lata, ITB, and abductors
Rotation: fibrous capsule (piriformis & other external rotators; internal rotators
Limitation must be assess in position of function to best understand its contribution
to a patient's complaint.

Diagnosis & Treatment principles for muscles


Diagnosis
Feel for nature of resistance at end-range
Abrupt
Gradual, elastic
Test strength against resistance
COMPARE SIDES
Treatment

Tightness: End-Range ME
isometric muscle energy stretch

Treating muscle groups? tight or weak first?


SUMMARY:
We are looking for limited range of motion (and the quality of that motion, esp. at
end-range).
If one muscle group is tight (contracted),
the motion that stretches it is decreased.
The muscle that opposes it may be weak (inhibited).
Take Home Message: Stretch tight muscles first, then strengthen weak muscles.

Postural and Phasic


Classifications for muscles: DIFFERENCES?
Tonic (or postural) muscles tend to become tighter with dysfunction.
Phasic muscles tend to become weaker with dysfunction.
These changes result from both simple reflex mechanisms and supraspinallymediated mechanisms.

Hip Flexors: major functions of iliacus


-Psoas
-Iliacus-Inside pelvic brim- Attach to lesser trochanter as iliopsoas tendon

Actions:
Flex thigh on pelvis
Raise trunk from recumbent position
Balance the trunk in sitting
Major player in creating anterior rotation of the innominate

these have poor mechanical advantage for influencing the sacro-pelvis


Rectus femoris (quadraceps)
Sartorius
crosses hip and knee
Adductor longus
Adductor brevis

Hip Extension Testing -Thomas Test


Pt. Supine - pelvis close to the end of the table
Both hips and knees are flexed
Contralateral LE is flexed knee to chest
Helps maintain lumbars flat on the table
Ipsilateral LE drops over the edge of the table
Normal range: Back of thigh strikes table

Hip Extension Restriction Treatment


(Hip Flexor Tightness)
Strength Testing:
Pt. Attempts to lift knee toward ceiling against operator resistance
Pt. Attempts to press foot & ankle medially against operator's leg.
Tightness - Treatment: Isometric contraction against operator resistance, Relax,
Reposition, Repeat, Retest.

'Iliacus Test' & Treatment


Pt. Supine - hip at edge of table
Cephalad Hand:
-Contact ASIS Force directed Posterior & Superior
-Caudad Hand:
Palmar contact just above the knee
Let gravity carry lower extremity to its end-point; assess
Add gentle pressure toward the floor to assess end-range resiliency
Normal Range:
Lift the pt's lower extremity back on to table after test
Tight Muscle/Restricted Range:
Direct Stretch with or without Muscle Energy

Iliacus Tender point


Location:
medial and inferior to the ASIS several cm
Treatment:

Cross ankle of involved LE over the other ankle


Flex, externally rotate
Frogleg Position
Rest ankles on physician's thigh - foot must be up on the table

Legg-calve-perthes disease. Which hip restrictor is most likely Tight/


restricted Anteriorly in legg-calve-perthes disease?
Avascular necrosis of the femoral head
WHICH ONE?
Illiacus

30 Degree Abduction Flexion Restriction


Pt. Supine:
Stand to the side of the patient so that the ASIS and the knee can easily be
contacted.
Bend the patient's knee until the arch of the foot is even with and nearly touching
the medial malleolus.
Use the cephalad hand to hold the ASIS in place - usually with a posterior, slightly
superior and medial force.
Place the caudad hand just above the knee and carry the lower extremity into
external rotation to its end-range
Compare sides, if the restriction is not obvious
Treat with Muscle Energy.
This addresses the anterior aspect of the capsular ligaments.

30 Degree Abduction Flexion Restriction


Home Exercise - Standing
Pt. Standing with foot of uninvolved side about 8-12 inches from the side of the door
frame.
Have them lift the heel of the invovled side up onto the medial malleolus of the
other lower extremity [This puts the lower extremity in about the same position
used supine on the table.]
The above should be done such that the knee comfortably contacts the door frame.
They next reach through the doorway with the hand on the involved side to grasp
the door frame
The other hand does not reach through, but contacts the door frame as well.
The two hands can now twist the whole body away from the knee, thus creating a
stretch in the same area of the anterior hip.

30 Degree Abduction Flexion Restriction


Home Exercise - Supine
Pt. Supine
The patient bends the knee until the arch of the foot is even with and nearly
touching the medial malleolus.
The patient brings the arch of the opposite foot up against the thigh very near the
knee.
The patient can let gravity and the weight of the opposite leg stretch this or actively

press with the opposite leg.


Many people are not flexible enough to do this variation effectively.

Hip Flexion/ Oblique Adduction


Think PIRIFORMIS & related fascias

Hip Flexion/ Oblique Adduction


Pt. Supine:
Flex the hip just past 90 degrees.
Then adduct, not toward the opposite ASIS, but toward the opposite chondral mass.
Compare sides.
The symptomatic side is usually tighter
Direct stretch or Muscle Energy

Hamstrings
I find the hamstrings on one side are often overstressed because the innominate is
rotated anterior on that side. This lengthens the hamstring on that side and makes it
more vulnerable to injury with activity, like soccer or dance or any other vigorous
activity with a lot of change of movement of the lower extremities.
First treat the cause. Often that is increased tone and tension in the ipsilateral
iliacus muscle. Then balance the pelvis including the anteriorly rotated innominate.

Hamstrings
Home Stretch
Patient Standing:
Puts heel up on a step or a stool or something else that does not feel like it
immediately puts a lot of tension on the hamstrings.
Find 'Pelvic Neutral' and maintain that with flexion around the hip joint.
To further help keep stress out of the low back, have the patient press the palms of
both hands against the proximal anterior thigh.
This creates a fulcrum to lift the axial spine, support it and focus the stretch into the
hamstrings.
The patient flexes until he/she engages resistance, not pain; continues to lift the
upper body via the hand contacts.
Hold 20-30 seconds. Repeat once or twice. Do every other day for the first week or
two. Then 5 out of 7 days per week after that. Compare with the uninvolved side at
least weekly. No need to get it any more flexible than that other side.

Adductors: NAME SOME Testing


End Range & Strength Against Resistance
All cross hip joint
Most arise somewhere on the pubic bone

Pectineus
Adductor longus
Adductor brevis
Adductor magnus
Gracilis
Descends beyond knee
Actions:
Femoral adduction
Aid in gait
Partial controllers of posture
Magnus and Longus also medially rotate thigh

Abductors NAME SOME Testing


End Range & Strength Against Resistance
Tensor Fascia Lata
Gluteus
maximus
medius
minimus
All attach somewhere on the ilium
All 3 have a role in extending the hip

Hip Extensors
Hamstrings
Biceps femoris
Semitendinosis
Semimembranosus
Gluteus maximusGluteus medius
Span hip and knee joints
All attach proximally somewhere on ischial tuberosity

Actions:
Flex knee / extend hip
Pull trunk upright against gravity
Forward reaching, swaying or bending evoke immediate, strong contraction in
hamstrings
Sometimes length can be short and limit flexion at hips with knees extended

Hip Flexion Testing and Restriction Treatment


: (Hip Extensor Tightness)
Tightness at End-Range:
Keep the knee bent a few degrees; that protects it.

External Rotators
Piriformis m.
Obturator internus m.
Obturator externus m.
Superior gemellus m.
Inferior gemellus m.
Quadratus femoris m.
Gluteus maximus
Gluteal medius
Sciatica, Hip Pain, Gluteal Pain

Neutral Internal Rotation


Testing & Treatment
(Tight External Rotators)
Pt. Prone - feet off end of table
Medial Hand: Palmar surface contacts sacrum
Lateral Hand: Palmar surface on the lateral gluteal musculature posterior to the
greater trochanter
Press anterior with lateral hand, carrying greater trochanter toward table
Medial hand stabilizes with counterbalancing pressure
ME or direct stretch work well
End-range is often decreased and lacking resilience when dysfunctional
Other approaches use a bent knee for leverage. OK if there is not knee problem.

Piriformis Tenderpoint
Location: middle of the gluteal region; in the body of the piriformis muscle
Treatment: approx. 135 degrees of flexion with abduction

Internal Rotators of the Hip


Gracilis
Adductor magnus
Adductor longus
Other actions
Gracilis both flexes leg and internally rotates
All 3 have a role in femoral adduction

Neutral External Rotation Treatment


(Tight Internal Rotators)
Cephalad Hand: Contacts the ASIS from its lateral side
Caudad Hand: Contacts the greater trochanter anteriorly
The two hands push away from each other to create an external rotation stretch at
the hip joint
Often a factor in the geriatric population with hip pain

Osteitis pubis
Inflamed pubic bone
Self-limited painful inflammation of the pubic symphysis
Symptoms: gradual onset with tenderness over symphysis pubis which radiates
along adductor and rectus abdominus muscles
Stress associated with athletics

Tensor fascia lata Tenderpoint


Location: 12 cm below greater trochanter along lateral surface
Treatment: ABDUCTION

Gluteus Medius and Minimus: Gluteus Medius Tenderpoint

Attached to external iliac surface


Attach distally to greater trochanter
Actions:
Abduct thigh
Internal rotation
Keep trunk upright when opposite foot is raised in walking and running
Active in Romberg's test
Point location: below iliac crest along upper portion of gluteus medius
Treatment: extension, internal rotation, slight abduction

What are the most commonly involved 'hip restrictors'?


Hip flexers - Iliacus
30 degree Abduction Flexion
Piriformis - Hip Flexed 90 degrees - Oblique Adduction
Adductors
Hamstrings
Abductors

Pubic Bone:
attachments
Superior- Rectus abdominus
Inferior- Adductor
Internal: puborectalis, pubococcygeus

SOAP notes:
Biomechanical:
MANY important muscular origins and insertions(from immersion!) from trunk
and lower extremity
Transition zone! Lumbo-sacral junction; iliopsoas as bridge btwn diaphragms (abd
and pelvic)
Visceral:
Distal GI issues: sigmoid colon, rectum
GU issues: Urinary incontinence, prostate inflam/hypertrophy, Baby-carrying,
cycles of menstruation
Vascular, Fluid:
common, int/ext iliac a; femoral a; lymphatic congestions
Neuro:

end of sympathetic chain


pelvic splanchnics, p'symp sacral plexus

pocket pelvis

Landmarks - static
Level of the PSIS
Gluteal folds: height, depth
Greater trochanters: height
Ischial tuberosities: height

Standing flexion: how to do it


Motion: standing flexion
Monitor: PSIS
Where?
Active patient motion: "reach for your toes"
Restriction on one side leads to premature locking at the SI joint.
This PSIS elevates SOONER, moves FIRST.
Positive test: dysfunction is named for side that moves.
Equal motion?- negative test
No dysfunction
Symmetrical dysfunction

ASIS segmental definition: name a motion test too


Thumbs immediately inferior to both bony prominences
A/P; S/I; closer or further away from belly button (I/E)

Motion (supplemental)
Compression test. Ease of motion in the A/P planes is noted with alternating,
rocking pressure

Pubic tubercles
Position
Find the belly button, glide base of hand inferiorly until you bump in to the pubic
bone.
Ensure finger pads rest on cephalad aspect of pubic bone.
Asymmetry is named for side of standing flexion dysfunction

Seg.Def. supine: MM
Compare relative position of the malleoli

Seg.Def. Prone PSIS. what ligament do you look at ?

Position: compare heights -S/I


Ischial tuberosities
Position: compare cephalad, caudad
Sacral Tuberous Ligament
Tight?
Lax ?
Sacral sulci
Position: compare depth- A/P

Pubic bone dysfunction


pubic bone superior or inferior to the other

Upslip
Are all landmarks superior or inferior to their contralateral partners

Stork test: how to do it


Pt standing
Monitor: one thumb at PSIS; one thumb at sacral crest (get a picture of where that
is)
Active patient motion: "raise knee (ipsilat to monitored psis) as high as is
comfortable"
Repeat both sides, noting the motion PSIS relative to sacral crest.
PSIS descends relative to sacral crest: negative test
PSIS moves Superior, or does not move relative to sacral crest: dysfunction=positive
test!

sacrum differentials? why look at sacrum?


Low Back Pain
Trauma
Whiplash
Falls
GI complaints
Parasympathetics to the lower GI tract
Postoperative Ileus
Irritable Bowel

GU complaints
Parasympathetics to bladder and prostate
Urinary incontinence
Urinary Retention
Gynecologic/Obstetrics
Parasympathetics to uterus
Dysmenorrhea
Changes during pregnancy and childbirth
Headaches
Cranio-sacral motion

Sacral Development
Developed from fusion of 5 vertebral segments, S1-S5
These are the 25th-29th of 33 vertebrae
Ossification begins in the 1st year of life
Complete ossification occurs between 25-30 years old

Sacral Anatomy: Anteriorly


Triangular shaped bone between innominates
Concave anterior surface
8 foramina (4 pairs) lie between the sacral vertebrae

Anterior sacral nerves exit through the foramina on anterior surface


Used as reference when naming torsions

Sacral Anatomy: Posteriorly


Convex posterior surface
Medial ridge created from spinous process rudiments
Central opening around S4-S5 forms sacral hiatus (arrow)
8 foramina allow passage of posterior sacral nerves

Sacral Articulations
Laterally
Innominate to form sacroiliac (SI) joints
Cephalad
5th Lumbar Vertebra via intervertebral disc
Caudad
Coccyx
3

base, sacral sulcus, ILA, apex

Name all important ligaments of sacrum TRUE


3 True: Sacroiliac
Posterior SI ligament
Note the orientation to
support weight bearing
Anterior SI ligament
Interosseous SI ligament
Unpictured

Name false ligaments of sacrum


3 False
Sacrotuberous Ligament
Sacrospinous Ligament
Divides greater and lesser
sciatic foramen
Ileolumbar Ligament

Sacral Anatomy
Muscular Attachments ANTERIOR AND POSTERIOR
Anterior attachments:
Iliacus
Coccygeus
Piriformis
PIC
Posterior attachments:
Gluteus Maximus
Erector spinae
Latissimus dorsi
GEL

Multifidus

sacral motion all axes


3 Transverse axes
Superior, Middle, and Inferior
2 Oblique axes
Right and Left
1 Longitudinal axis
1 Anterior-posterior axis

Transverse Axes
Superior: Respiratory and Craniosacral axis
Located at approximately S2, at the location of dural attachment
Middle: Postural axis
Bilateral Flexion & Extension occur around this axis
Inferior: Innominate rotation axis

Sacral Motion: Nomenclature


Sacral extension
Base moves posteriorly
In inhalation phase, sacral base moves posteriorly (AKA extends or counternutates)
Sacral flexion
Base moves anteriorly
Occurs during exhalation, sacral base moves anteriorly (AKA flexes or nutates)

Sacral Motion: Oblique Axes naming- when is dynamic motion


important
Dynamic Motion: physiologic
Occurs during ambulation
Weight bearing on Left leg will engage left axis and vice versa
Right and Left
Named for the side of the upper pole of the oblique axis!!

palpation landmarks for sacrum

palpation landmarks for sacrum


Palpation Nomenclature
Anterior and Deep
When the sacral base moves anterior the sulcus/sulci can be said to be anterior or
deep.

Posterior and Shallow


When the sacral base moves posterior the sulcus/sulci can be said to be posterior or
shallow.

screen scan and seg. def. for sacrum ALL VARIATIONS


Screen
Scan
Perform the Seated Flexion Test
Segmental Definition
Forward Torsion
Right on Right (R on R)
Left on Left (L on L)
Backward Torsion
Right on Left (R on L)
Left on Right (L on R)
Unilateral Sacral Flexion
Unilateral Sacral Extension
Bilateral Sacral Flexion
Bilateral Sacral Extension
R rotation on R forward torsion- L5 NSRRL

dynsfunction patterns. which one has L5? which one is due to sacral
shear?
Sacral Torsion
Rotation around an oblique axis with somatic dysfunction at L5
Unilateral Flexion/Extension
Flex/Ext around a transverse axis at only one SI joint
Due to sacral shearing
Bilateral Flexion/Extension
Flex/Ext around a middle transverse axis at both SI joints

L5 in torsions? RULES TO DIAGNOSING L5


In a torsion, L5 Sidebends into the axis, Rotates opposite sacrum
Rule #1: When L5 is sidebent, a sacral oblique axis is engaged on the same side as
the sidebending.
Rule #2: When L5 is rotated, the sacrum rotates the opposite way on an oblique
axis.
Rule #3: The seated flexion test is found on the opposite side of the oblique axis

L5 will always rotate in the opposite direction of the sacrum


L5 will always be sidebent towards the side of the sacral oblique axis (when normal
mechanics are operating; trauma can change that)

Left on Left (ROTATION ON ___ AXIS)


Right on Right
WHEN SAME- FORWARD
WHEN OPPOSITE- BACKWARDS

With Bilateral Sacral Flexion or Extension


seated flexion test is Falsely Negative and the Sacral Sulci appear equal in depth.**
Therefore, another diagnostic modality must be used
The Spring Test
SPHINX TEST

Spring Test
Classic Test:
Anterior pressure at the sacral sulci
Normal: should spring/have resiliency
Positive Test: Base is resistant, does not spring, feels like concrete
Meaning: Base is stuck in extension or posterior (it will not go into flexion)
ILA testing:
Positive test: ILAs are resistant to anterior pressure, do not spring

The Diagnostic Process - Scan


Seated Flexion Test
-Pt. Seated & feet flat on the floor
-Operator - thumbs on inferior slope of PSISs
-Pt. Forward bends with arms between the knees
One PSIS moves further than the other
More Cephalad or Ventral
This is the Side of the Sacroilial Dysfunction

Transverse Axes what happens at each one


Superior: Respiratory Axis
Motion relative to the pull of the dura occur around this axis
Craniosacral (Inherent) motion occurs here.
Located at S2
Middle: Postural Axis
Bilateral flexion and extension occur around this axis
Located at anterior convexity of upper and lower limbs of SI joint
Inferior: Innominate Rotation Axis
Located at the posteroinferior part of the lower limb of the SI joint

Screen Sacro-pelvis 2 motion, 2 tissue texture

Standing Flexion test


Standing flexion occurs at the hips and is translated from the innominate into the
sacrum via the middle transverse axis.

Therefore a positive standing flexion test is most likely due primarily to innominate
dysfunction.
Iliosacral

Seated Flexion test


What stabilizes the innominate?
Seated, the innominate is somewhat stabilized by the ischial tuberosities on the seat
and flexion occurs through the spine and translated from the sacrum into the
innominate via the middle transverse axis.
Therefore positive seated flexion test are most likely due primarily to sacral
dysfunction
Sacroilial

The Diagnostic Process - Scan


DEEP = ANTERIOR
SHALLOW = POSTERIOR
sacral base
PSIS:
Have your patient lie prone
Follow the iliac crests posterior to their limit - the bottom of the PSIS.
Move thumbs superiorly onto the main part of the PSIS, its posterior prominence.
SACRAL SULCUS
Move thumbs medially from the PSIS a thumb breadth - thumbs now over sulci.
Use your "awesome" palpatory skills to determine if one sulci is deeper or more
shallow than the other
Follow the spines of the sacrum inferior until your index finger drops into a 'hollow',
the sacral hiatus.
The posterior aspect of the ILA is just 2-3 cm lateral to and on either side of the
sacral hiatus.

Bilateral Sacral Extension


Bilateral Sacral Flexion
Improper lifting techniques
Fall in seated position

Patient can't backward bend or it is very painful.


Extremely common postpartum
Arched with a heavy load
Patient can't forward bend or it is very painful.

The Diagnostic Process: 'Sphinx Test'


Extend bacl flex sacral basePatient sulci depths are tested neutral prone first
Patient assumes the prone- prop position (sphinx)
Lumbars extend, sacrum flexes
Negative Test: Decreased sacral base asymmetry= sacral flexion ease
Positive Test: increased sacral base asymmetry = extension ease

+ spring test at Left ILA

L5 N SLRR
treatment- what muscles do you engage
Left on Left Torsion
Rotating the axial spine to the left (restriction) puts the sacrum into right rotation
(restriction: L rotation on L axis)
Dropping the legs off the table gaps the SI joint
Patient is positioned with + flexion test side up
With ME, the patient is activating the piriformis and engaging the ligaments in the
sacropelvic region

Left on Left Torsion TXT sequence


Patient Prone
Doctor on opposite side of axis
With the patient's hips and knees flexed to 90, roll the patient onto their hip such
that the axis is DOWN
Physician Monitors at Lumbosacral junction
Physician supports patients knees (with thigh) and lets ankles drop to floor
Continue to monitor lumbosacral jn
ME Treatment: patient pushes ankles to ceiling while physician restricts motion
Patient relaxes, take up the slack, then the ankles are allowed to drop further to
floor
Perform 3-5 times

Localize: may need to readjust flexion/extension after muscle contraction in order


to bring the forces back to the right sacral sulcus (sacral base)

Right on Left Torsion


Backward Torsion
+ spring test at R sulcus,
+ sphinx test (ease on extension although sacrum should flex)

Right on Left Torsion


-Patient in lateral recumbent position with axis DOWN
-Physician uses caudad hand to flex the knees until motion is felt at the lumbosacral
junction
-Physician pulls patient's arm closest to the table to rotate patient while monitoring
L5
-Patient should be on back
-Patient's bottom leg is extended until force is felt to localize to the right sacral base.
-Patient's top leg is dropped of the table in front of other leg.
-ME Treatment: patient pushes leg (ankle) toward the ceiling while physician
restricts motion
-Patient relaxes and the leg is allowed to drop further to floor; readjust other
elements, as needed.
-Perform 3-4 x's

What is happening during treatment?


With patient supine and the hips rotated with the axis down...
causing sacral motion to the left (motion that is restricted)L5 rotation occurs to
the right
Furthermore...
Extension of the left leg during treatment positioning causes further flexion of
sacrum (i.e. restrictive barrier of sacrum)
With ME, the patient is activating the piriformis and is also engaging the ligaments
around the sacral pelvic region to allow for sacral rotation
For backwards torsion...Lay patient on their "Back"
Remember, axis down on the table!

Left Unilateral Flexed Sacrum


+ Spring test on Left ILA

Sacral Movements and Respiration:


causes sacral extension (base goes backward)Inhalation
causes sacral flexion (base goes forward)Exhalation
Therefore...
We can use the phases of respiration to treat unilateral sacral lesions!

Treatment - Left Unilateral Flexed Sacrum- DONT FORGET TO USE


BREATH
Patient Prone
Physician on side of dysfunction
Knee is flexed to 90 degrees, and the leg is abducted and internally rotated.
This causes gapping at the SI joint
Physician monitors at SI Joint
Leg is stabilized by physicians chest
Caudad hand on the ipsilateral ILA, cephalad hand monitors SI joint
Avoid the coccyx with right hand!
Physician engages restrictive barrier by placing anterior, superior pressure at the
ILA
With the barrier engaged, patient assists by inhaling deeply to move sacral base
posteriorly and holds the breath in while physician continues with pressure at ILA.
After cycle, physician engages new barrier at ILA and another deep breath is held
Repeat 3-5 x's

What is happening during treatment?Left Unilateral Flexed Sacrum


With left unilateral flexed sacrum, the sacral sulcus is anterior and the ILA is
posterior.
The left leg is abducted and internally rotated to gap SI joint to allow for movement
of sacrum
Anterior, superior pressure at left ILA causes the left side of sacrum to extend (into
its restrictive barrier)
this causes increased sacral extension at the basePatient augments this by inhaling
Pressure at the left ILA during exhalation (between held inhalations) prevents the
sacrum from flexing

Low Velocity, Moderate Amplitude (LVMA) springing could be used to augment this
treatment.

Left Unilateral Extended Sacrum


(+) Sphinx Test

...
Patient prone
Physician on side of dysfunction
Physician places caudad hand on the inferior portion of left ischial tuberosity and
cephalad hand on left sacral sulcus
Ischial tuberosity is carried superiorly toward sacral base
Left sacral base is carried anterior and inferior to the restrictive barrier
Physician uses Low Velocity, Moderate Amplitude (LVMA) springing with both
hands into the restrictive barrier

Unilateral Extended Sacral Dysfunction WHAT IS GOING ON?


In a left unilateral extended sacrum, the left sulcus is posterior and the left ILA is
anterior
Pressure is held on the ischial tuberosity to keep the innominate from rotating and
moving inferior during the activating force
Springing at the left sacral base forces the left sacral sulcus into flexion (the motion
that is restricted)
Springing is continued until restricted barrier is passed (softening of resistance
occurs & motion is more like the other 'normal' side.

Bilateral Sacral Dysfunctions


Bilateral Flex Sacrum
Bilateral Extended Sacrum
Note: movement occurs around the middle transverse axis

Treatment - Bilaterally Flexed Sacrum


(-) spring test at sacral sulci
(+) spring test at sacral ILA

Treatment - Bilaterally Flexed Sacrum- LEVER


Patient seated
Physician places caudad hand on sacral apex and cephalad hand on thoracic region
Patient flexes trunk until resistance is felt at the sacrum by the physician
Patient actively extends trunk, while physician resists motion with cephalad hand
Patient holds extension for 3-5 sec then relaxes.
As patient relaxes, pressure is continued on the sacral apex, and the patient flexes

until a new barrier is reached


Repeat 3-5 x's

What is happening during treatment?


As the patient initially flexes the trunk, the sacrum becomes extended bilaterally
This takes the sacrum to the restricted barrier
As the patient tries to extend, this causes the sacrum to flex bilaterally
The physician uses a counterforce at the thoracic region, which causes isometric
contraction at the lumbosacral junction
As the patient relaxes, the trunk is able to flex further and increased sacral
extension is able to occur
HAVE THEM INHALE to modify

Bilateral Sacral Extension


(+) spring test at sacral base
(-) spring test at ILA's

Treatment - Bilaterally Extended Sacrum


Patient seated
Physician places caudad hand at sacral bases, and cephalad hand around patient's

chest
Patient is asked to extend trunk until restricted barrier felt at sacral base
Patient is instructed to flex trunk as physician restricts movement with cephalad
hand. During flexion, physician also uses caudad hand to restrict sacral extension
Patient holds contraction for 5 sec
As patient relaxes, the trunk is furthered extended until a new barrier is felt at the
sacral base
Repeated 3-5 x's.

What is happening during the treatment?


With a bilaterally extended sacrum, the sacrum prefers extension
As the patient extends the trunk, the sacrum is forced into flexion
This is the restricted barrier
Then, as the patient flexes, the sacrum starts to extend bilaterally
The physician uses a counterforce at the sacral base to prevent sacral extension.
As the patient relaxes, the patient can further extend the trunk and cause an
increase in sacral flexion

tricks
Dysfunction on an oblique axis:
1. Determine axis and place axis down
2. Determine forward/backward torsion (Pt on chest if forward, back if backward)
3. Position legs accordingly (Front 2 Back (1) )

OMM

Biomechanical
Fluid
Nervous system
Visceral

1. Relevant Biomechanical Concerns:


(torques, twists, strains, undue pressure anywhere in the musculoskeletal system
Delineate the local and regional biomechanical concerns that may impact the
patient problem.
Delineate the inter-regional biomechanical concerns that may impact the patient
problem.
Suggest possible Osteopathic manipulative interventions that may assist in
normalization of biomechanical influences associated with the patient problem

2. Relevant Concerns with Fluid Congestion (venous & lymphatic, in


particular):
Delineate the drainage pathway for the area, viscus, joint, or other component
relevant to the patient problem. Follow this to its terminal drainage
Describe and portray the 'choke points' that may impede drainage along this path.
Suggest Osteopathic manipulative interventions that may assist in drainage of the
fluid congestion associated with the patient problem

Tissue Level:
There are many mechanisms for accumulation of tissue fluid.
They are dependent upon an inability of the drainage system to cope with excess
fluid.

Lymphatic Drainage for the Body


Lymphatics are the 'Overflow system' of the body.
Local drainage or lack thereof is only one component in the equation of fluid build
up.
**Key Principle: Any blockage Proximal to the excess fluid will inhibit its drainage.

3. Relevant Nervous System Concerns:


-SNS
-PNS
- Pain
Delineate the innervation for the area, viscus, joint, or other component relevant to
the patient problem. Follow its course to the spinal cord.
Describe the anatomic factors/influences along the course of the innervation that
can change SNS function.
Suggest possible Osteopathic manipulative interventions that may assist in
normalization of the sympathetic influences associated with the patient problem.

Sympathetic Nervous System

"Hypersympathetic activity is a common factor in disease."


Multiple influences upon the internal organs
Sympathetic fibers have a vasomotor function, among other things.
These fibers stimulate contraction of blood vessels.
Notice the location of the chain ganglia
Rib somatic dysfunction may be seen to change the pressure upon these ganglia.

Parasympathetic Innervation
REST IS VAGUS- EXCEPT IN HEAD - parasymp.

Common Denominator?
Key concept: All of these stuctures course through the myo-fascial (muscles &
fascia) system.
This suggests that Biomechanical dysfunction (somatic dysfunction) within the
musculoskeletal system can play a role in optimal or suboptimal function of the four
factors just mentioned.

Types of Somatic Dysfunction HVLA?


Myotonic- all
articular- IMPULSE
neuromuscular- all no impulse
myofascial
vascular

Rule of Threeesss
T1-3: TP at the same level as the tip of the SP
T4-6: TP vertebral level above the tip of the SP
T7-9: TP 1 full vertebral level above the tip of the SP
T10: TP 1 full vertebral level above the tip of the SP
T11: TP vertebral level above the tip of the SP
T12: TP at the same level as the tip of the SP

thoracic vertebra have an intimate relationship with...


ribs- diagnostic and therapeutic implications

if thoracic treatment doesnt work


rib dysfunciton may be primary- opposite can be true

limitations to thoracic movement name 4


ribs, bony structure, attaching ligaments, articular facets about 60 degrees from
horizontal

The direction of the plane of the facets in the thoracic spine is:
how about lumbar and cervical
Backward
Upward
Lateral
Lumbar & Cervical are Backward, Upward & Medial
BUM BUL BUM

The direction of the plane of the facets is consistent with the axis of
rotation for the vertebral body in the .... why is this important
thoracic
Therefore there is minimal shearing strain on the disc during neutral mechanics &
rotation is facilitated.

Movement- Thoracic Spine- greatest to least motion- discuss


limitations
Rotation- greatest inherent motion (decreased lower)
Limited by the rib cage
Lateral flexion (SB)- 2nd greatest motion (limited upper, freer lower) - upper has
tibs
Flexion- 2nd least motion (more in lower)
SPs & laminae separate
TPs separate
Inferior facets- move UP AND FORWARD (open)
Extension- least motion (more in lower)
Limited by approx. of laminae & SPs
TPs approx.
Inferior facets- move DOWN AND BACKWARDS (close)
Real Lamina Flex and Extend- greatest to least motion

Fryette's 3rd Principle: and motion of thoracic spine


more motion input in areas where the spine moves most freely (RLFE)
In the thoracic spine where there is little extension, it takes very little extension to
localize (especially if you have already localized in the other two planes - rotation &
sidebending.
You may need to roll further over the patient for upper thoracic somatic dysfunction
to localize the rotation component - because that is where the greatest thoracic
rotation ROM occurs.

posterior view of spine

treament HVLA 4 steps and WHAT IS THE MOST IMPORTANT THING


TO DO
diagnose, prepare, localize, thrust
- prepare patient and have them relax*****

FRYETT's PRINCIPLE *I and II


For a Type II non neutral dysfunction (F and E) sidebending is.... how
about type I neutral dysfunction?
Side bending will be to the same side as rotation
Side bending will be to the opposite side of rotation

where does the physian stand to check prone patient spinal vertebra
Doctor stands on Dominant Eye side of patient
Place thumb pads on transverse processes
Check flexion/extension, rotation & sidebending

if no change in flexion or extension. may get softer on flexion and


extension
A group of segments may be involved
If so, your scan has brought you to the most involved segment of the group
Neutral Mechanics
NSLRR or NSRRL
Tension/resistance eases in F/E, worse through Mid-ROM = NSR Dysfunction
Could represent structural scoliosis

preparing for HVLA- may correct or facilitate ease


soft tissue, springing, muscle energy

soft tissue method - what part of hand do you use


Apply transverse stretch across the paraspinal musculature in the thoracic region
Use base of thenar/hypothenar eminence to push tranversely along the erector
spinae group bilaterally allowing for release of the tissue

T4-12 Extension Dysfunction Prep for HVLA SEATED


Muscle Energy as prep:
Localize into resistance (flexion) at the level of the dysfunction
Use ME protocol

Non-neutral Extended RSleft


Supine ME Prep Technique SUPINE
Patient tries to pull backward toward the table

characteristics of thrust
High Velocity = Controlled FAST Movement
Low Amplitude = SHORT distance
Squeeze, squeeze, squeeze hard
VECTOR of Thrust

Thoracic HVLA: Extension Only T4-T12


Patient supine-doc stands at either side
Pt. arms crossed-opposite on top
Use caudad hand to support B/L TPs, Lift head into flexion
Localize to your TP contact
Thrust posterosuperior through patient's elbows
Creates a Flexion force at the dysfunctional segment
Recheck

alternative methods EXTENSION


Pt. supine, doc stand on opposite side of posteriorly rotated TP
Pt. cross arms

Rotate pt. into flexion with cephalad hand


Place fulcrum (thenar eminence) under posterior TP
Flex at and above dysfunctional unit
Sidebend toward you to dysfunctional level
Thrust posterosuperiorly into fulcrum at the stuck facet

Neutral HVLA SLRR


Pt. supine, doc stand on side opposite rotation (left)
Pt. cross arms
Rotate pt. into slight flexionwith cephalad hand
Fulcrum at posterior TP of dysfunctional segment
Spine neutral at level of dysfunction [this is where the resistance will feel most
localized]
Sidebend away [(right)-opposite dysfunction]
Thrust into fulcrum

T4-T12 Springing
Patient seated- doc standing in front
Patient crosses forearms
Doc passes arms under patient forearms and over shoulders to monitor segment
with fingertips
Patient drawn forward, flexing at hips
Thorax extended to restriction
Apply spring until release
Recheck

T7 Flexed RRSR
Supine ME technique
Force is localized through the elbows to the thenar eminence.
Sidebend, extend and rotate for localization using the head & neck motion input
Pt. uses isometric force against the physician.
Reposition for localization into resistance
Repeat, Recheck

prone HVLA FLEXED OR NEUTRAL


Flexed or neutral dysfunction only
Patient is prone and physician stands on the side of posterior transverse process.
Caudad hand, pisiform region inferior to transverse process, fingers pointing
cephalad
Cephalad hand, thenar eminence on opposite inferior transverse process fingers
pointing toward patient's feet
Pt. inhale/exhale to localize forces
Thrust is anterior and rotary engaging both flexion and rotation

T4-T12 FRSleft:
-Doc on opposite side of vertebral rotation
-Pt. cross arms-opposite (left) on top
-Roll pt. into flexion with cephalad hand
-Fulcrum (caudad/right) thenar eminence posterior to left transverse process
-Roll over fulcrum at stuck facet to extend at dysfunction while
-Flexion is maintained while head/neck is used to introduce sidebending (right)
down to the dysfunctional segment
-Thrust thru elbows straight down into fulcrum (medial to thenar eminence)
causing extension
Basically just turn into the segment

Alternate Pt. Arm Placement for T1-T3


-Accurately diagnose
-Doc stand at side of table opposite posteriorly rotated TP
-Patient clasps hands behind neck and approximates elbows
-Doc places thenar eminence on posterior TP
Doc grasps pt's elbows and rolls the pt slightly towards feet
-Patient is asked to inhale deeply and completely exhale
-At the end of exhalation doc exerts a rapid AP thrust through the patients arms to
the TP resting on the doc's thenar eminence.
Reassess

Local Motion Segmental Definition


ROTATION
With thumbs on TP, pushing gently on left TP induces rotation right
With thumbs on TP, pushing gently on right TP induces rotation left
Test for both motions until you have a feel for rotation ease/resistance
FLEXION and EXTENSION
Place both thumbs BELOW TP
Push anterior and cephalad to induce FLEXION
Place both thumbs ABOVE TP's and push anterior and caudad to induce
EXTENSION
Test both motions to deduce ease/resistance at that segment
Palpatory Approach to Sidebending:
Press superior on one transverse process while pressing inferior on the other
Below L TP and above R TP = R side bending
Below R TP and above L TP = L side bending
Generally corresponds to Fryette's Principles

Screen TART AND STAR- what generic questions might u ask?


-How are the TART/STAR criteria useful at this level of diagnosis?
-General Impression
-Is there a problem?
-What regions exhibit a problem?
Gait /A-P /Lateral
TART
-tissue texture abnormalities
-asymmetries
-restriction to motion
-tenderness

or STAR
-sensitivity changes instead of tenderness
tissue texture & motion tests

Red Reflex description of findings


T5 FRrSr: red reflex lingers after tissues above and below blanch out; right
paraspinal muscles at this level are tender.

SOAP NOTES - subjective


unique historical elements related to musculoskeletal system

objective
succinct musculoskeletal findings

assessment
1) medical diagnosis
2) regions of somatic dysfunction

Plan
1) diagnostics
2) OMT, medication
3) patient education etc.

where do you put specific dysfunction


in OBJECTIVE *just right somatic dysfunction in assessment

screen req (HOW ______ you encounter resistance)


2 tissue 2 motion- how soon do I encounter resistance

SCAN (__________ response to motion)


1 tissue 1 motion- immediate response to motion- looking for points of increased
resistance

goal of ME
To decrease Muscle Hypertonicity
To lengthen muscle fibers
To reduce the restrained movement
To strengthen weaker muscles
To allow for greater joint mobilization

Eight Essential Steps to ME- AEP MC RRR


Eight Essential Steps:
Accurate Structural Diagnosis
Engage restrictive barrier
Provide Unyielding Counterforce

Muscle Effort Appropriate for the Patient


Complete Relaxation
Repositioning
Repeat 3-6 for Three to Five Repetitions
Retest - What change has occurred?

Restrictive Barrier
A functional limit within the anatomical range of motion, which abnormally
diminishes the normal physiologic range' (Glossary)
A = anatomical
P = physiological
R = resistance

Unyielding Counterforce
-counterforce is toward resistance for the diagnosed segment
-The amount of force will determine how big of an area you engage for the
treatment (more force for a larger involved area)
-The resistance the operator applies determines how much force the patient
generates.

most effective position to treatment for muscle energy


sidebending has been determined by Dr. Mitchell and his team to be the most
effective.

Fryette's 3rd Priniciple:


Initiating motion at any vertebral segment in any one plane of motion will modify
the mobility of that segment in the other two planes of motion
-Positioning with sidebending will decrease the amount or repositioning needed
with rotation and flexion/extention.

complete relaxation of patient- what type of contraction do we have?


explain muscle energy?
MOST IMPORTANT PART. Will not work if the patient doesn't relax completely
after their muscle activation. The muscle will still be activated and so cannot be
"reset".
"... immediately after an isometric contraction, the neuromuscular apparatus is in a
refractory state during which passive stretching may be performed without
encountering strong myotactic reflex opposition. All the operator needs to do is

resist the contraction and then take up the slack in the muscles during the relaxed
refractory period."

reposition
Repositioning (re-engaging resistance)
Place patient into resistance at new localized restrictive barrier (F/E, Sb, R)

treatment of dysfunction with muscle energy


GOAL: Put the dysfunction where it doesn't like to go!!==Barrier
SEGEMENTAL DEFINTION==EASE of motions
Muscle energy== away from ease so you do the opposite motions of segemental
definition.

patient and physician bodily position for ME


Patient Seated:
Feet on the floor
Physician Standing:
Behind patient
To side of side bending resistance
(to provide physical support for the patient as they are sidebent into resistance)

hand position for physician


-Place operating forearm and hand on the shoulder of side bending resistance
-Create elbow pressure down on the patient's resistance-side shoulder and induce
sidebending into restriction
-Elbow pressure forward and backward will create either left or right rotations into
resistance.
-physican force is arrow

guiding a patient when force is applied


"Gently side bend to your left (or right)."
If the patient used too much force: "Now give me that much (or 2/3 or )."

If the patient used too little force: "Give me double that force please"

patient directions when counterforce applied


-Direct the patient to hold this isometric contraction for 3-5 seconds
-Against continuing physician resistance
-Then direct the patient to relax
-Reposition into resistance in side bending & rotation
-patient tries to sidebend the other direction

alternative way to apply force- bear hug


-The physician's axilla could be used on the shoulder instead.
-If you have the patient cross his arms, this looks like your diagnostic procedure.
It also increases patient stabilization.

C3-C7 articular facets


-Superior facets facing backward upward and medial
-This requires rotation and sidebending to the same side

deep cervical muscles


SHORT MUSCLES
Rectus capitis posterior major
Rectus capitis posterior minor
Obliquus capitis superior
Obliquus capitis inferior
Interspinalis
Intertransversii

C-spine functional anatomy


ONLY SPAN one to two segments
-Along with the junction of the zygapophyseal joint and the capsular structures
-Therefore, the motions introduced will be deliberately small and discrete towards
the restrictive barrier

supine cervical sidebending technique


-Use translation L & R to create side bending into resistance
--Left translation creates right sidebending and vice versa

-Add rotation into resistance


-Add flexion/extension into resistance

lumbar MEmore rotational forces are needed to localize down to the segment
More Side Bending, Rotation, and/or Flex/Ext might be required
Recall Fryette's Law III

alternative ME technique
lateral recumbent on left side with legs bent to induce flexion, pulled up to induce
Sr, and R the knees left to push more into restriction. SO.... "Pulling your feet to the
floor" would normalize the action towards ease.L4 ESlRr

flexed lumbar dysfunction

extended lumbar somatic dysfunction

2 caveats about ME
-ME of the lumbar spine is similar to that of the thoracic spine but we increase the
magnitude of motion applied in order to localize at the more distally located somatic
dysfunction.
-Soft tissue treatment to the area of the segment should be done after ME
treatment.

primary motion of rib cage


inhalation and exhalation- also exhibits the others-

primary muscle of respiration


diaphragm

SECONDARY muscles of respiration


Scalenes
Pectoralis Minor
Serratus Anterior
External Intercostal Muscles
Their attachments influence the pump vs bucket handle motions of the rib cage. Rib
dysfunction can be treated by utilizing these muscles because of their direct
attachment to the ribs. Because the ribs are all connected through intercostal
muscles, influencing one rib will also influence the neighboring ribs through these
connections.

Scalene -origin insertion function and motion


Engages pump and bucket handle of Ribs 1-2
Originate at the transverse process of C2-7 and insert on Ribs 1 and 2
Function: sidebend the neck ipsilaterally

pec minor origin insertion and function


Originate at ribs 3, 4, and 5 and insert on medial coracoid process.
Engages pump handle motion

Serratus Anterior origin insertion and function and motion


Originates on the lateral aspects of as many as ribs 1-9 (varies per source) and
attaches to the scapula from the superior angle, down along the medial border, and
to the inferior angle.
Function: Protracts the scapula, and assists with upward rotation
Engages bucket handle motion

Quadratus Lumborum insertion and function and motion


Originates at the iliac crest and inserts along the transverse processes of the lumbar
vertebrae and the 12th rib. You can use Quadratus Lumborum to treat the caliper
motion ribs: directly treating the 12th rib because of the direct attachment and
indirectly treating the 11th rib through intercostal connections.

overlapping muscles means overlapping motions

simplified motions
Rib 1: 50/50
Rib 2-6: Pump Handle
Ribs 7-10: Bucket Handle
Ribs 11-12: Caliper Motion

ribs 1-10 can have what kind of mechanics


bucket or pump AND inhalation or exhalation

rib caveat in screen and scan- motions of pump and bucket


Remember that the pump/bucket handle motion of the ribs causes the ribs to come
up (suprior) anteriorly and laterally. The rib heads are anchored at the
costovertebral joint. Therefore, the motions are opposite.
With inhalation, ribs will move superiorly along sternum and inferiorlyat the rib
angle!

With exhalation, ribs will move inferiorly along the sternum and superiorly at the
rib angle.

attention on motions of the ribs at the ...


end range of inhalation and exhalation

Exhalation restriction
rib will stop moving down before the "normal" rib on the other side

Restricted inhalation
rib will stop moving up before contralateral rib on attempted full inhalation

Treatment of 1st Rib: Pump Handle Exhalation ME


Position: Pt places hand on forehead: looking straight ahead
Counterforce: Doctor disengages rib head with anterior/lateral pressure, holds head
down
Contraction: Patient asked to raise head off the table

Treatment of 1st Rib: Bucket Handle Exhalation ME


Position: Pt places hand on forehead: looking 40 toward the lesion
Counterforce: Doctor disengages rib head with anterior/lateral pressure, hold head
down
Contraction: Patient asked to raise head off the table

Treatment of Ribs 2-10 : Pump Handle Exhalation ME


Position: Pt places hand on forehead: looking straight ahead
Counterforce: Doctor disengages rib head with anterior/lateral pressure, resists
horizontal adduction
Contraction: Patient asked to pull elbow down and across chest

Treatment of Ribs 2-10 : Bucket Handle Exhalation ME


Position: Pt places hand on forehead: looking straight ahead
Counterforce: Doctor disengages rib head with anterior/lateral pressure, resists
lateral adduction
Contraction: Patient asked to pull elbow down and along table

Treatment of Ribs:
Inhalation vs Exhalation
Exhalation ribs are "stuck down"
We are going to engage muscles to pull them up
Inhalation ribs are "stuck up"
We are going to apply force directly on ribs to push them back down

Treatment of 1st Rib: Inhalation


-Position: Pt places hand on forehead: looking straight ahead
-Counterforce: Doctor resists motion at 1st rib anteriorly
-Contraction: Patient asked to raise head off the table

Treatment of Ribs 2-10:Pump Handle Inhalation


Position: Pt places hand on forehead: looking straight ahead
Counterforce: Doctor holds anterior ribs inferior as muscles contract
Contraction: Patient asked to pull elbow into hand and across chest

Treatment of Ribs 2-10 :


Bucket Handle Inhalation
Position: Pt places hand on forehead: looking straight ahead
Counterforce: Doctor holds lateral ribs in place as muscles contract
Contraction: Patient asked to pull elbow down and along table

intercostals- where do they come in?


Intercostal muscles attach all ribs and can be utilized to help treat lower ribs pump
handle dysfunction

rib disengaging
Treatment augmentation by disengaging the rib head posteriorly is helpful when
possible

Concentric Isotonic
When the muscle tension causes the origin and insertion to approximate

Eccentric Isotonic

Isolytic contaction
Nonphysiological, contraction by the patient attempts to bring origin and insertion
in approximation but an external force applied by the operator occurs in the
opposite direction

Isometric:
Distance between the origin and insertion of the muscle is maintained at a constant
length

thoracic problem- imp ME muscles


Multifidi
Rotatores
Intertransversarii

MRI
These small tensile elements (multifidi, rotatores, Intertransversarii, etc) are the
focus of muscle energy treatment for the axial spine.
Intercostals for ribs

THEORY BEGIND MUSCLE ENERGY


Nociceptive stimuli results in excitatory input to the gamma motor neurons. When
gamma motor neurons are activated the spindle fibers are more sensitive to stretch,
and thus more likely/prone to feed forward excitatory input to the alpha-motor
neuron pool. Which will then likely result in even more nociceptive stimuli further
exacerbating the problem.

alpha-gamma coactivation theory


One performs a voluntary activation of skeletal muscle, that in addition to the
activation of the alpha motor neurons, that gamma motorneurons are also activated
(the theory of alpha-gamma coactivation). Thus, following the isometric contraction
(and subsequent activation of gamma motorneurons) there may be a period of
reduced sensitivity to the spindle fibers detecting a change in muscle length.

founder of muscle energy


mitchell
-You have two forces that are equal but opposite in direction. This interaction helps
by loosening up previously tightened joints.
He chose to get patient's own muscles involved in the treatment because the
patient's muscles are more biologically intimate with the joints as opposed to that
(i.e. the muscles) of the examiner.

Muscle Energy theory assumes


Abnormally shortened muscle(s)
--> part of the mechanism of somatic dysfunction
-->These shortened muscles prevent a full range of motion of its joint(s) in some
plane

Mechanism for abnormally short muscle


Neuroreflexive (predominant mechanism)
Fibrosis or gelosis (frozen muscle)
Active or Passive congestion (assoc. w/ myofascial trigger points)

ME is direct or indirect. shorten or lengthen muscle?


Attempts to lengthen the distance between the origin and insertion of certain
muscles
Direct

method of ME

Uses Muscular Activation to overcome motion restriction of tissues and joints


-Uses Muscle Spindles
-Causing Post-Isometric Relaxation

what is the activating force?


Patient provides
Breath counts as a Force

indications
Acute: patient unable to relax
local or general muscle tension is increased
Chronic: somatic dysfunction with fibrotic, shortened tissues
If Tx using Ease hasn't worked
Improved localization = less force needed

too much force=


opposite effects- post-treatment soreness is common and may happen

patient comes with stiff neck ,what do u do


Rule out infections/any trauma then precede to treat and advise the use of
medications.

Two Questions you must answer to decide upon the ME treatment:


for RIBS
Inhalation/Exhalation Dysfunction?
Primarily Bucket handle &/or Pump handle motion? [ribs 11 & 12: caliper motion]
-NOT Flexion/Extension, Sidebending, Rotation

treatment of exhalation or inhalation disfunction


exhalation dysfunction- move ribs into inhalation
inhalation dysfunction- move ribs into exhalation

DIAGNOSIS for muscle energy of ribs


Inhalation rib or Exhalation rib" with 'Bucket handle &/or Pump handle' elements
i.e. Right rib 4 inhalation dysfunction, pump handle primary

each side is ______ of the other (ribs)


independent

in scan u perform one motion scan to confirm (MUSCLE ENERGY)


sidebending OR rotation

dysfunction trick for anterior ribs PUMP HANDLE


Space above 'Wide' (below, 'narrow') = exhalation dysfunction (resists inhalation)
Space above 'Narrow' (below, 'wide' = inhalation dysfunction
COMPARE RESPONSE TO EXHALATION AND INHALATION
AE and NI

dysfunction trick for anterior ribs BUCKET HANDLE


Compare intercostal space above and below for width:
Space above 'Wide' (below, 'narrow') = exhalation dysfunction (resists inhalation)
Space above 'Narrow' (below, 'wide' = inhalation dysfunction
Confirm by Comparing its response to inhalation/exhalation with that of the one on
the other side or with the rib above and below.

Exhalation Dysfunction
Ribs 11-12
-Patient prone ARMS ABOVE HEAD, Doctor on opposite side of the table
-Legs pulled toward doctor to side bend, ipsilateral arm above head
-Doctor grasps ASIS and rotates pelvis posteriorly
-Doctor places hand laterally over involved rib
-Patient takes a deep breath, Doctor pushes laterally on the rib (gapping joint)
-Hold breath & localization 3-5 seconds
EVERYONE RELAX!!
-Reposition (increase traction on rib and increase posterior rotation of the pelvis)
-REPEAT (3-5 times)
-RECHECK

why patient arm above head in exhalation dysfunction 11-12 rib


Patient's arm position uses the latissimus dorsi to facilitate the movement of the rib
up (into restriction) while resisting the action of the quadratus lumborum

Inhalation Dysfunction
Ribs 11 & 12
-Patient prone ARMS AT SIDE, Doctor on opposite side of the table
-Legs pulled toward doctor to side bend, ipsilateral arm at side of body
-Doctor grasps ASIS and rotates pelvis posteriorly
-Doctor places hand proximally over involved rib (medial fulcrum)
-Patient takes a deep breath, exhales; Doctor pushes laterally on the rib (gapping
joint)
-Hold 3-5 seconds
-EVERYONE RELAX!!
-Reposition (traction on the lower ribs and posterior rotation of the innominate)
-REPEAT (3-5 times)
RECHECK

Inhalation Dysfunction
Rib 1-10 - Pump Handle:
-Patient Supine,
-Flex the spine to dysfunctional segment, supported for doctor
-Add sibebending toward the rib
-Doctor contacts superior aspect of rib
monitor anteriorly
-Respiration is activating force, patient takes a deep breath, doctor resists
inhalation and follows the rib more with exhalation
-Reposition (increase flexion and sidebending)
-REPEAT (3-5 times)
RECHECK
Engaging resistance is augmented by flexion & sidebending of the axial spine.

Inhalation Dysfunction
Rib 1-10 - Bucket Handle
Patient Supine,
Flex the spine to dysfunctional segment, supported for doctor
Add sibebending toward the rib
Doctor contacts superior aspect of rib
monitor anteriorly
Respiration is activating force, patient takes a deep breath, doctor resists inhalation
and follows the rib more with exhalation
Reposition (increase flexion and sidebending)
REPEAT (3-5 times)
RECHECK
Engaging resistance is augmented by flexion & sidebending of the axial spine

Exhalation Dysfunction
Rib 1-10 - Pump Handle:
Patient Supine,
Doctor contacts inferior aspect of rib; finger pads contact via interspace below the
rib (chondral)
Monitors and supports inhalation phase.
Extend spine, if possible
Respiration is activating force, patient takes a deep breath, doctor follows the rib
with inhalation & resists exhalation.
Reposition (increase extension?)
REPEAT (3-5 times)
RECHECK
Engaging resistance is augmented by extension of the axial spine - use the Head of
the table

Exhalation Dysfunction
Rib 1-10 - Bucket Handle
-Patient Supine,
-Doctor contacts inferior aspect of rib; finger pads contact via interspace below the
rib (mid-axillary line)
-Monitors and supports inhalation phase.
-Sidebending patient away from the dysfunctional rib

-Respiration is activating force, patient takes a deep breath, doctor follows the rib
with inhalation and resists exhalation
-Reposition (increase sidebending)
-REPEAT (3-5 times)
-RECHECK
Engaging resistance is augmented by sidebending of the axial spine away.

Alternative: Respiration as Activating Force - Pump Handle


Treatment- exhalation and inhalation
Posterior Hand: disengages the posterior articulation (as done for Exhal. Dys.
Previously)
Anterior Hand: Thumb pad contacts anterior rib
Exhalation Dysfunction:
below the costochondral jct. (narrow part)
Inhalation Dysfunction:
above costochondral jct. (narrow part)

Alternative: Respiration as Activating Force - Bucket Handle


Treatment
Posterior Hand: disengages the posterior articulation (as done for Exhal. Dys.
Previously)
Lateral Hand: Thumb pad contacts anterior rib
Exhalation Dysfunction:
below the costochondral jct.
Inhalation Dysfunction:
above costochondral jct.

Anterior subluxed rib diagnosis

Rib Anterior Subluxation Treatment - Right 5th Rib


Pt's. right hand holds left shoulder
Control motion of the thorax via the left hand contact with the pt's right elbow &
abdominal contact with the pt's back
Right thumb creates the fulcrum
Via contact with the shaft of the rib, Medial to the rib angle
Pt is instructed to pull the right elbow laterally or caudally
Isometric, relax, repeat, retest
Additional Fulcrum:
Pt's left fist on anterior rib to create A-P force during contraction
Fulcrum: Thumb creates & maintains posterolateral 'pull' on rib shaft

goal with engaging anterior subluxed rib

Rib Anterior or Posterior Subluxation Treatment - Right 5th Rib


-SUPINE
Same principles and contacts:
Here, pt's. body weight helps hold finger pad contact against the medial aspect of
the rib angle. [ or lateral for posterior subluxation]

Muscle Energy Technique Rib Posterior Subluxation Treatment - Right 5th Rib
Difference with Tx for Anterior Subluxation:
Posterior Thumb pressure is MEDIAL
Isometric force: pt. tries to adduct elbow and physician resists

what is the mobile unit?


Bony structure with articular surfaces for segmental movement
Adnexal tissues (neuromuscular & connective)
Control system

the body is not static

In the standing position, the body is not "static"


The spine = complex organized arrangement of mobile parts
The local segment responds to motion and the overall need for erect positioning
Response to gravity
-The spine is a complex organized arrangement of mobile parts that cam be
positioned
-Within this motor function, the examiner is looking at the local response to gross
motor demand (e.g. standing) is an essential concept for the procedural principle
being applied to investigate motor function.

Fundamental Unit of Somatic Dysfunction


- the mobile unit
-3 segment group
-Central vertebra/rib = primary focus
-Segments above and below respond by compensating in a predictable way

central segment versus those above and below


The central segment can not respond to a movement request in a coordinated
fashion with the segments immediately above and below
Those segments compensate for this as much as possible, but the motion is still
altered

somatic dysfunction encompasses


Skeletal, Arthrodial & Myofascial Structures
Related Vascular, Lymphatic & Neural Elements

Direct techniques and examples


are ones in which the restricted joint or tissue is initially taken into the direction of
resistance to motion"
Muscle energy, HVLA

Indirect techniques and examples


are those that initially position the joint or tissue away from a barrier to motion and
toward the relative ease or freedom of motion"
Strain-counter-strain, FPR, Functional Methods

what relationship is important in FUNCTIONAL METHODS


Neuro-reflexive relationship************* Functional Methods uses the Neuroreflexive response to motion input as the basis for treatment - ribs, axial spine, etc.

functional technique describe defining characteristic


3 Rotary Motion Asymmetries
3 Translation Motion Asymmetries
Anterior/posterior rotation
Left/right translation
Cephalad/Caudad translation
Respiration Asymmetry
COMPLETE MOTOR ASYMMETRY (imp vocab word)All motions are involved when there is somatic dysfunction present
*note above and below!! inhlation and exhalation

The Functional Methods Treatment ProtocolStand on the side of side bending ease
Start by engaging sidebending toward ease
Follow with rotation toward ease
Follow rotation with flexion/extension toward ease
Follow with anterior/posterior translation toward ease
Follow anterior/posterior translation with left/right translation toward ease.
Follow with cephalad/caudad translation toward ease
Each motion is refined during the respiratory ease

Functional Technique- Thoracic/Lumbar Positioning


Patient is seated with arms crossed
At the end of the treatment table
Doc on the side of sidebending ease
Example - SB ease to left
Doc stands on left side of patient
The left upper extremity reaches under the opposite armpit of the patient

Functional TechniqueCervical Positioning


Doc seated at the head of the table
Use finger pads of your middle finger to palpate segment and use your palms to
input motion

Functional TechniqueRib Cage Positioning


Side of sidebending ease if using trunk motions
And/or
Upper extremity input is most often also utilized to treat a rib dysfunction

Functional TechniqueUpper Extremity & Rib Function


Upper Extremity Motion Input
Assess for asymmetric response (ease/bind) at the rib angle. Motions:
Abduction/Adduction
External/Internal Rotation
Cephalad/Caudad Compression

how fast do you put the motions in *


SLOWLY

your are looking for what kind of response*


IMMEDIATE-

one hand is always*


listening

mistakes with functional methods


Do not carry any motions too far because you could actually increase tension
There is such a thing as positioning "past" ease
This can be used to refine your sense of optimal ease as you learn.
Not necessary after you become more comfortable with the feel of 'ease'

term to describe the addition of motions, when do you refine the


motions
The order and the process of placing all six motions into its ease has been termed
'Stacking'
Each motion is refined during the respiratory phase of ease
-Ride the phase of respiratory resistance maintaining as much ease as possible*

How many breaths are necessary?**


Until you feel no increase in tension as you return the patient towards neutral (good
approach for beginners) OR
You feel the central segment and the adjacent mirror image segments feel the same
OR

Until you feel a 'release'

Our goalSimultaneous input of all 6 motions riding ease through respiration to resolution of
the dysfunction
=they make us slow down to optomize efficacy of treatment

COMPLETE MOTOR ASYMMETRY


All motions are involved when there is somatic dysfunction present

documentation for functional methods


T6- flexed rotated sidebent right translated posterior left and superior
and inhalaiton
T 6: FRSr, Tran PLS, Inh

way to remember motions


S
R
E
P
R
S
respiration

how do you engage translational motion


USE YOUR BODY CONTACT TO INTRODUCE THESE THREE MOTIONS

difference between axial spine and rib adjustments FUNCTIONAL


METHODS
AXIAL:
Shoulder/Trunk input ONLY
Contact transverse processes
RIB:
Shoulder/Trunk AND upper extremity input
Contact rib angles

Regional Interactions: axial and ribs and upper extremity


upper extremity and axial spine affect ribs and converse

resistance to inhalation testing technique


Resistance to inhalation is a more immediate palpable cue. This phase (inhalation)
is the easiest to monitor
Resistance to exhalation can be confirmed by the characteristic mirror image
resistance to inhalation for the segments above and below.

where is listening hand in treatment?


'listening hand' may be on the ipsilateral or contralateral rib angle depending upon
the sidebending ease. MUST BE ON RIB ANGLE

Upper Extremity Influence Upon Rib Function


Upper Extremity Motion Input:
Assess for asymmetric response (ease/bind) at the rib angle. Motions:
Abduction/Adduction
External/Internal Rotation
Cephalad/Caudad Compression
ALSO A COMPLETE MOTOR ASYMMETRY - FOR UPPER EXTREMITY

treatment?
Patient: Lateral Recumbent
Physician: Standing in front of and facing the patient

founder of funcitonal methods


Dr. Johnston

Cross-Column Mirror Image Findings


When there is a rudimentary rib dysfunction - a rib preferring exhalation - mirror
image findings are found with the associated vertebra

rib that prefers inhalation does what?


RIB INHALATION DYSFUNCTION - all the vowels
I
E
A
Example:
Rib 4 inhalation ease (exhalation resistance)
Pattern of Ease:
Adduction
Internal Rotation
Cuadad Traction

rib preferring exhalation does what?

Example:
Rib 4 exhalation ease (inhalation resistance)
Pattern of Ease:
Abduction
External Rotation
Cephalad Compression

hand placement for functional methods with arm movement


...

Always take into account


chief complaint => leading to a diagnosis and relevant OMM case considerations:
BIOMECHANICAL
Fluid (Venous-Lymphatic)
Nervous System
Sympathetic
Parasympathetic
Pain

components to fixing a discunction

2 main regional considerations

Biomechanics and leverage

Case Considerations above the diaphragm CORD LEVELS


==Musculoskeletal Anatomy
Cranial and Cervical
Thoracic and Ribs
Upper Extremities
==Autonomic Innervation
T1-4 Head and Neck
T1-5 Heart, T1-6 Lungs
T5-9 Upper GI
T10-11 Colon to the Splenic Flexure, Renal, Gonadal
T12-L2 Lower Colon
T3-T8 Upper Extremity
==Lymphatics

case considerations below the diaphragm CORD LEVELS


==Musculoskeletal Anatomy
Lumbar
Sacropelvic
Lower Extremity
==Autonomic Innervation
T12-L2: Pelvic Organs, Sigmoid colon, lower extremity
(T10-L2: Kidneys/Adrenals, Gonads)

==Parasympathetic: Vagus to level of descending colon, Pelvic splanchnic plexus for


pelvic organs
Lymphatic

3 diff clinical approaches


Centrifugal (Focal)
Start with area of complaint and scan outward from there
Screen adjacent regions until no dysfunction is identified
Centripetal (General)
General screen and scan with greater focused attention on area of complaint and
other positive screen areas
Key Lesion (Hybrid)
General screen and scan but focus treatment on "region of greatest restriction" and
sequentially progress from there

3 things which feed into dysfunction


Oftentimes, it all comes together in the somatic dysfunction diagnoses: disturbances
in the Nervous system, Fluid Dynamics & Biomechanical considerations.
Somato-visceral
Viscero-somatic
Somato-somatic

Korr:
A segment in view is a segment in trouble.

UPPER HALF DIAGNOSIS SEQUENCE****


1 Anterior Ribs
2 Thoracolumbar Junction
3 Cervicothoracic Junction
4 Thoracics / Other Ribs

5 Scapulothoracic Articulation
6 Craniocervical Junction / Cranium
7 Cervicals
8 Rest of Upper Extremity

Anterior Ribs
Treatment of anterior ribs and anterior fascia has a direct effect on posterior
costovetebral dysfunction and vertebral dysfunction
Anterior soft tissue relationships impact posterior relationships
Treatment of Sutherland's "Front of the Back"
Therefore, evaluation for anterior rib 'focal points of dysfunction' = 'fulcrums' is a
good place to start

anterior fulcrum and stress on anterior chest?


Kyphotic posture: Converging to a fulcrum in the anterior rib cage
Rib Dysfunctions- e.g. rib 6 exhalation dysfunction, pump handle primary - can lock
in the fulcrum/focal point of dysfunction
Normal Posture: no convergence to a fulcrum
Trauma or repetitive stress could create an anterior fulcrum, however

lumbar lordosis problems?


Lumbar Lordosis: Patients tend to flatten thoracics in response
Fulcrum becomes higher
Rib 3 inhalation dysfunction could reinforce this postural challenge

what if you find upper rib problems?


If you find rib problems, especially in the top 6 ribs, these ribs will be important
factors in upper body complaints

Pump Handle Relationships of Interest, in particular


Monitor Pump handle Motion at the costochondral junction will be our primary
focus initially
Scan the ribs, esp. 1-6
Segmentally define the pump handle aspect
Wider gap above or below?
Response to respirations:
Inhalation
Exhalation

ways to treat anterior ribs


-Counterstrain
Intercostal tender points
-Muscle Energy
Pump handle ribs
-Anterior Rib Fulcrum [new technique]
-Functional Methods
Anterior rib release
Same contact as muscle energy, but treating in the directions of ease

diagnosing anterior ribs


Diganosis:
Listening: Index fingers contact the anterior rib cage sequentially, at the level of
each rib
Motion: Use forearm contact bilaterally to introduce rotation right and left
alternately
Feel for a sense of increased localized tension during rotation under one of the
palpating fingers
Localized increased tension = a rib fulcrum
Start at rib one and work down
assess responses to
Rotation
Flexion/Extension
Sidebending
Translations
Using the forearm contact and the Osteopathic belly, carry the patient into the
directions of resistance
The localization of tension should be confined to the diagnosed anterior rib fulcrum.
Use the Muscle Energy treatment sequence

Thoracolumbar Junction
WHAT DO WE FIND HERE? how would u treat it?
Biomechanical transition zone
Postural implications on upper transition zones and segments
Relationship of 12th rib and/or diaphragm can have dramatic impact on remainder
of costal cage (and secondarily the cervical region)
Diaphragm
Supine/seated release [new technique]
Muscle Energy/ Direct Stretch
11th & 12th ribs [new technique]
T11-L2
Functional Methods/FPR
11th & 12th ribs
T11-L2

muscle energy thoracolumbar junciton- diagnosis- DIRECT STRETCH


TECHNIQUE
Diagnosis:
Patient seated:
Contact the lateral inferior border of the 12th (or 11th) rib
Press gently superior and laterally; compare with opposite side for ease/resistance
Determine the tight side.
Treatment:
Same position and contact
Press gently superior and lateral on the under side of the rib
Carry the patient into flexion & sidebending until the 'feather edge' of resistance is
encountered
Continue as a direct stretch, or
Have patient try to side bend back to neutral (muscle energy

Cervico-Thoracic Junction
[Superior Thoracic Aperture]
A cadaveric depiction shows this area slightly higher than in a living patient.
Why? The ribs go into exhalation at death.
Superior Thoracic Aperture & Terminal Lymphatic Drainage
First rib/clavicle/vetebral unit are the superior thoracic aperture with associated
visceral structures
Dramatic transition zone biomechanically
Soft tissue relationships connecting the vulnerable visceral neck to the sturdier
thorax
Thoracic duct - Left and Right
Therefore, Diagnose & Treat:
C7-T1 with associated 1st rib
Can extend out to C6 and T2 with 2nd rib

3. Cervico-Thoracic Junction
[Superior Thoracic Aperture] treatment
Treatment
1st Rib:
Indirect: Counterstrain, FPR, FM
Direct: Muscle Energy
Superior Thoracic Aperture
Supine release [new technique]- STEERING WHEEL
Anterior cervical fascia releaseseated [new technique]- choke hold

SUPERIOR THORACIC APERTURE- steering wheel- SUPINE RELEASE


Supine Diagnosis & Treatment:
Thumbs go posteriorly: contact costo-transverse junction bilaterally (this allows

control of T1 and the first ribs)


Fingers wrap around the neck: the finger pads come as close to the junction of the
first rib and manubrium as possible (this means the finger is looping over the
clavicle.)
Gently try to rotate the superior thoracic aperture left & right
to the 'feather edge of resistance'
Which direction is easier? (like screening)
Gently side bend left & right
Which direction is easier?
Treatment: carry the sup. thor. Aperture in both directions of ease

Anterior cervical fascia releaseseated


[new technique]
CHOKE
Patient Seated
Place thumb pads between the two heads of the sternocleidomastoid muscle on each
side
Patient slumps - back and neck
Let the thumbs follow the tissue inferior (not much posterior)
Hold that position with the thumbs as the patient inhales, holds breath and
straightens up

4. Thoracic/Other Ribs
Upper thoracics provide sympathetic innervation to viscera above the diaphragm
Heart, Lungs, Head and Neck
Upper thoracic regions provides important biomechanical relationships to cervical
spine, thoracic inlet, ribs and associated scapulothoracic joint
A Non-Neutral in the T4-6 area is not uncommon.
Correlate with Sympathetic innervations and associated viscera.
autonomic nerves and intercostals *

4. Thoracic/Other Ribs: Treatment


Counterstrain/Functional Methods/Muscle Energy (HVLA - will learn later Winter
quarter; LAR in year 2)
You already know how to treat somatic dysfunction in this region.

5. Scapulo-Thoracic Articulation
One of the important articulations of the shoulder
Restriction is usually due to costal cage dysfunction combined with muscular
imbalance
Has attachments to thoracic and cervical spine, as well as rib cage
Therefore significant contributor to dysfunction in those regions, as well.

May secondarily change biomechanics of the glenohumeral joint


Counterstrain:
Ribs
Pectoralis minor
Trapezius
Levator Scapulae
Serratus Anterior
Rhomboid
Muscle Energy (&/or direct stretch):
Ribs & same muscles

Pectoralis Minor
Assessment & Treatment
Operator contacts the insertion of the pectoralis minor at the 3rd-5th ribs anteriorly
The ipsilateral elbow is abducted sufficiently to bring the vector forces to a
localization at ribs 3-5 anteriorly
Abdominal contact with the patient's back brings stability during the treatment
Have the patient pull the elbow forward with only sufficient force to engage the
pectoralis minor at its insertion.
Isometric force, Relax, Repeat, Retest

Posterior Scapular Muscle Diagnosis & Treatment:

Caudad Hand: Thumb & Web cup the inferior angle of the scapula
Cephalad Hand: Contacts the lateral shoulder and the superior border of the
scapula
With these contacts you can engage stretch for any of the muscles listed.
Is it tight?
It may be obvious or you can compare with the opposite upper extremity
Treatment:
Lengthen the muscle to stretch it
Direct stretch or
Muscle energy
Trapezius
Levator Scauplae
Rhomboid
Serratus Anterior

6. Cranial-Cervical Junction/Cranium
Occiput/Atlas/Axis/Suboccipital tissues
Final common pathway for all forces originating inferiorly
Jugular foramen
Vagus nerve
Greater occipital nerve
Emerges between C1 & C2

6. Treatment:
Suboccipital Release [new technique]
Indirect:
FM, FPR, Counterstrain:
C1
C2
Direct: Muscle Energy

SUBOCCIPITAL REGION
RELEASE OF RESTRICTION(S) BETWEEN OCCIPUT AND SUPERIOR SURFACE
OF C1

DIAGNOSIS:
Place finger pads against the tissues overlying the OA/AA.
Determine resistance vs. compliance (ease)
TREATMENT:
RELEASE TENSION OF SUBOCCIPITAL MUSCULATURE
REDUCE RESPIRATORY EFFORT
MAY BE ACCOMPLISHED BY VARIOUS APPLICATIONS OF FORCE

7. Cervicals
8. Rest of Upper Extremity
...

Thoracic anatomy
Thoracic vertebra
slightly larger vertebral bodies than the cervicals,
and increase in size as they become more weight bearing
Angle of the articular facets
about 60 degrees from the horizontal plane

Articulations of each thoracic vertebra


-With vertebra above and below
-With the vertebral disc of adjecent vertebral segments
-Articulates with adjacent ribs

muscles that attach to thoracic vertebrae


longus colli, serratus posterior, erector spinae and transversospinalis
SHORT: levator costarum, intertransversarii, and interspinalis muscles

Upper extremity muscles


rhomboids, latissimus dorsi and trapezius

thoracic spine prior to rib cage, TXT SEQUENCE


FM, ME, Impulse (direct) --> HVLA

Sequencing technique**
1) diagnose accurately
2) patient must be comfortable
3) position to restrictive barrier
4) use release-enhancing maneuvers *BREATHING, springing, jaw clenching
5) deliver thrust once relaxed
6) reasses

important motions 1,2,3,4

1) rotation- decreased lower


2) lateral flexion- limited upper freer lower
extension
3) 2nd least: flexion, more in lower
TPs separatefacets up and forward (open)
4) extension: least: more in lower, limited by approx- more in lowerTP's are close to one another
facets down and back (closed)

Rule of 3's
1-3 SP= TP
4-6 SP =1/2 below TP
7-9 SP= 1 below TP
10=7-9
11=4-6
12=1-3

Fryette's principles
I. neutral SB before R OPPOSITES
II. non-neutral R before SB- same side
III. motion in one plane affects motion in other planes
THORACIC AND LUMBAR ONLY

most posterior on L side=


ease side

if no change in flexion or extension=


remains neutral
more than one segment may be involved

testing while prone


push down on either side = rotation
lift transverse process up and down= flexion extension
above on one side and below on the other= sidebending

techniques to prepare for HVLA


Soft tissue techniques,
Springing
Muscle energy techniques

thrust is 2 features
HIGH VELOCITY
LOW AMPLITUDE- short distance-

put hand under what and patient supine


UNDER transverse process

treatment
Patient Supine-arms crossed
2. Doctor - Stand on the opp side posterior TP
3. Holding pt's head, place thenar eminence
under the TP of the dysfunctional segment
4. Flex pt's head until feeling motion at that
segment
5. Position pt's elbows for your comfort
6. Pt takes breath in and out
7. Thrust will be in exhalation through pt's
elbows into your thenar eminence

flexed or neutral dysfunction


-Flexed or neutral dysfunction only
-Patient is prone and physician stands on the side of posterior transverse process.
-Caudad hand, pisiform region inferior to transverse process, fingers pointing
cephalad
-Cephalad hand, thenar eminence on opposite inferior transverse process fingers
pointing toward patient's feet
-Pt. inhale/exhale to localize forces
-Thrust is anterior and rotatory engaging both flexion and rotation

Screen TART AND STAR- what generic questions might u ask?


-How are the TART/STAR criteria useful at this level of diagnosis?
-General Impression
-Is there a problem?
-What regions exhibit a problem?
Gait /A-P /Lateral
TART
-tissue texture abnormalities
-asymmetries
-restriction to motion
-tenderness
or STAR
-sensitivity changes instead of tenderness
tissue texture & motion tests

Red Reflex description of findings


T5 FRrSr: red reflex lingers after tissues above and below blanch out; right
paraspinal muscles at this level are tender.

SOAP NOTES - subjective


unique historical elements related to musculoskeletal system

objective
succinct musculoskeletal findings

assessment
1) medical diagnosis
2) regions of somatic dysfunction

Plan
1) diagnostics
2) OMT, medication
3) patient education etc.

where do you put specific dysfunction


in OBJECTIVE *just right somatic dysfunction in assessment

screen req (HOW ______ you encounter resistance)


2 tissue 2 motion- how soon do I encounter resistance

SCAN (__________ response to motion)


1 tissue 1 motion- immediate response to motion- looking for points of increased
resistance

goal of ME
To decrease Muscle Hypertonicity
To lengthen muscle fibers
To reduce the restrained movement
To strengthen weaker muscles
To allow for greater joint mobilization

Eight Essential Steps to ME- AEP MC RRR


Eight Essential Steps:
Accurate Structural Diagnosis
Engage restrictive barrier
Provide Unyielding Counterforce
Muscle Effort Appropriate for the Patient
Complete Relaxation
Repositioning
Repeat 3-6 for Three to Five Repetitions
Retest - What change has occurred?

Restrictive Barrier
A functional limit within the anatomical range of motion, which abnormally
diminishes the normal physiologic range' (Glossary)
A = anatomical
P = physiological
R = resistance

Unyielding Counterforce
-counterforce is toward resistance for the diagnosed segment
-The amount of force will determine how big of an area you engage for the
treatment (more force for a larger involved area)
-The resistance the operator applies determines how much force the patient
generates.

most effective position to treatment for muscle energy


sidebending has been determined by Dr. Mitchell and his team to be the most
effective.

Fryette's 3rd Priniciple:


Initiating motion at any vertebral segment in any one plane of motion will modify
the mobility of that segment in the other two planes of motion
-Positioning with sidebending will decrease the amount or repositioning needed
with rotation and flexion/extention.

complete relaxation of patient- what type of contraction do we have?


explain muscle energy?
MOST IMPORTANT PART. Will not work if the patient doesn't relax completely
after their muscle activation. The muscle will still be activated and so cannot be
"reset".
"... immediately after an isometric contraction, the neuromuscular apparatus is in a
refractory state during which passive stretching may be performed without
encountering strong myotactic reflex opposition. All the operator needs to do is
resist the contraction and then take up the slack in the muscles during the relaxed
refractory period."

reposition

Repositioning (re-engaging resistance)


Place patient into resistance at new localized restrictive barrier (F/E, Sb, R)

treatment of dysfunction with muscle energy


GOAL: Put the dysfunction where it doesn't like to go!!==Barrier
SEGEMENTAL DEFINTION==EASE of motions
Muscle energy== away from ease so you do the opposite motions of segemental
definition.

patient and physician bodily position for ME


Patient Seated:
Feet on the floor
Physician Standing:
Behind patient
To side of side bending resistance
(to provide physical support for the patient as they are sidebent into resistance)

hand position for physician


-Place operating forearm and hand on the shoulder of side bending resistance
-Create elbow pressure down on the patient's resistance-side shoulder and induce
sidebending into restriction
-Elbow pressure forward and backward will create either left or right rotations into
resistance.
-physican force is arrow

guiding a patient when force is applied


"Gently side bend to your left (or right)."
If the patient used too much force: "Now give me that much (or 2/3 or )."
If the patient used too little force: "Give me double that force please"

patient directions when counterforce applied


-Direct the patient to hold this isometric contraction for 3-5 seconds
-Against continuing physician resistance
-Then direct the patient to relax
-Reposition into resistance in side bending & rotation
-patient tries to sidebend the other direction

alternative way to apply force- bear hug


-The physician's axilla could be used on the shoulder instead.
-If you have the patient cross his arms, this looks like your diagnostic procedure.
It also increases patient stabilization.

C3-C7 articular facets


-Superior facets facing backward upward and medial
-This requires rotation and sidebending to the same side

deep cervical muscles


SHORT MUSCLES
Rectus capitis posterior major
Rectus capitis posterior minor
Obliquus capitis superior
Obliquus capitis inferior
Interspinalis
Intertransversii

C-spine functional anatomy


ONLY SPAN one to two segments
-Along with the junction of the zygapophyseal joint and the capsular structures
-Therefore, the motions introduced will be deliberately small and discrete towards
the restrictive barrier

supine cervical sidebending technique


-Use translation L & R to create side bending into resistance
--Left translation creates right sidebending and vice versa

-Add rotation into resistance


-Add flexion/extension into resistance

lumbar MEmore rotational forces are needed to localize down to the segment
More Side Bending, Rotation, and/or Flex/Ext might be required
Recall Fryette's Law III

alternative ME technique
lateral recumbent on left side withL4 ESlRr legs bent to induce flexion, pulled up
to induce Sr, and R the knees left to push more into restriction. SO.... "Pulling your
feet to the floor" would normalize the action towards ease.

flexed lumbar dysfunction

extended lumbar somatic dysfunction

2 caveats about ME
-ME of the lumbar spine is similar to that of the thoracic spine but we increase the
magnitude of motion applied in order to localize at the more distally located somatic
dysfunction.
-Soft tissue treatment to the area of the segment should be done after ME
treatment.

primary motion of rib cage


inhalation and exhalation- also exhibits the others-

primary muscle of respiration


diaphragm

SECONDARY muscles of respiration


Scalenes
Pectoralis Minor
Serratus Anterior
External Intercostal Muscles
Their attachments influence the pump vs bucket handle motions of the rib cage. Rib
dysfunction can be treated by utilizing these muscles because of their direct
attachment to the ribs. Because the ribs are all connected through intercostal
muscles, influencing one rib will also influence the neighboring ribs through these
connections.

Scalene -origin insertion function and motion


Engages pump and bucket handle of Ribs 1-2
Originate at the transverse process of C2-7 and insert on Ribs 1 and 2
Function: sidebend the neck ipsilaterally

pec minor origin insertion and function


Originate at ribs 3, 4, and 5 and insert on medial coracoid process.
Engages pump handle motion

Serratus Anterior origin insertion and function and motion


Originates on the lateral aspects of as many as ribs 1-9 (varies per source) and
attaches to the scapula from the superior angle, down along the medial border, and
to the inferior angle.
Function: Protracts the scapula, and assists with upward rotation
Engages bucket handle motion

Quadratus Lumborum insertion and function and motion


Originates at the iliac crest and inserts along the transverse processes of the lumbar
vertebrae and the 12th rib. You can use Quadratus Lumborum to treat the caliper
motion ribs: directly treating the 12th rib because of the direct attachment and
indirectly treating the 11th rib through intercostal connections.

overlapping muscles means overlapping motions

simplified motions
Rib 1: 50/50
Rib 2-6: Pump Handle
Ribs 7-10: Bucket Handle
Ribs 11-12: Caliper Motion

ribs 1-10 can have what kind of mechanics


bucket or pump AND inhalation or exhalation

rib caveat in screen and scan- motions of pump and bucket


Remember that the pump/bucket handle motion of the ribs causes the ribs to come
up (suprior) anteriorly and laterally. The rib heads are anchored at the
costovertebral joint. Therefore, the motions are opposite.
With inhalation, ribs will move superiorly along sternum and inferiorlyat the rib
angle!

With exhalation, ribs will move inferiorly along the sternum and superiorly at the
rib angle.

attention on motions of the ribs at the ...


end range of inhalation and exhalation

Exhalation restriction
rib will stop moving down before the "normal" rib on the other side

Restricted inhalation
rib will stop moving up before contralateral rib on attempted full inhalation

Treatment of 1st Rib: Pump Handle Exhalation ME


Position: Pt places hand on forehead: looking straight ahead
Counterforce: Doctor disengages rib head with anterior/lateral pressure, holds head
down
Contraction: Patient asked to raise head off the table

Treatment of 1st Rib: Bucket Handle Exhalation ME


Position: Pt places hand on forehead: looking 40 toward the lesion
Counterforce: Doctor disengages rib head with anterior/lateral pressure, hold head
down
Contraction: Patient asked to raise head off the table

Treatment of Ribs 2-10 : Pump Handle Exhalation ME


Position: Pt places hand on forehead: looking straight ahead
Counterforce: Doctor disengages rib head with anterior/lateral pressure, resists
horizontal adduction
Contraction: Patient asked to pull elbow down and across chest

Treatment of Ribs 2-10 : Bucket Handle Exhalation ME


Position: Pt places hand on forehead: looking straight ahead
Counterforce: Doctor disengages rib head with anterior/lateral pressure, resists
lateral adduction
Contraction: Patient asked to pull elbow down and along table

Treatment of Ribs:
Inhalation vs Exhalation
Exhalation ribs are "stuck down"
We are going to engage muscles to pull them up
Inhalation ribs are "stuck up"
We are going to apply force directly on ribs to push them back down

Treatment of 1st Rib: Inhalation


-Position: Pt places hand on forehead: looking straight ahead
-Counterforce: Doctor resists motion at 1st rib anteriorly
-Contraction: Patient asked to raise head off the table

Treatment of Ribs 2-10:Pump Handle Inhalation


Position: Pt places hand on forehead: looking straight ahead
Counterforce: Doctor holds anterior ribs inferior as muscles contract
Contraction: Patient asked to pull elbow into hand and across chest

Treatment of Ribs 2-10 :


Bucket Handle Inhalation
Position: Pt places hand on forehead: looking straight ahead
Counterforce: Doctor holds lateral ribs in place as muscles contract
Contraction: Patient asked to pull elbow down and along table

intercostals- where do they come in?


Intercostal muscles attach all ribs and can be utilized to help treat lower ribs pump
handle dysfunction

rib disengaging
Treatment augmentation by disengaging the rib head posteriorly is helpful when
possible

Concentric Isotonic
When the muscle tension causes the origin and insertion to approximate

Eccentric Isotonic

Isolytic contaction
Nonphysiological, contraction by the patient attempts to bring origin and insertion
in approximation but an external force applied by the operator occurs in the
opposite direction

Isometric:
Distance between the origin and insertion of the muscle is maintained at a constant
length

thoracic problem- imp ME muscles


Multifidi
Rotatores
Intertransversarii

MRI
These small tensile elements (multifidi, rotatores, Intertransversarii, etc) are the
focus of muscle energy treatment for the axial spine.
Intercostals for ribs

THEORY BEGIND MUSCLE ENERGY


Nociceptive stimuli results in excitatory input to the gamma motor neurons. When
gamma motor neurons are activated the spindle fibers are more sensitive to stretch,
and thus more likely/prone to feed forward excitatory input to the alpha-motor
neuron pool. Which will then likely result in even more nociceptive stimuli further
exacerbating the problem.

alpha-gamma coactivation theory


One performs a voluntary activation of skeletal muscle, that in addition to the
activation of the alpha motor neurons, that gamma motorneurons are also activated
(the theory of alpha-gamma coactivation). Thus, following the isometric contraction
(and subsequent activation of gamma motorneurons) there may be a period of
reduced sensitivity to the spindle fibers detecting a change in muscle length.

founder of muscle energy


mitchell
-You have two forces that are equal but opposite in direction. This interaction helps
by loosening up previously tightened joints.
He chose to get patient's own muscles involved in the treatment because the
patient's muscles are more biologically intimate with the joints as opposed to that
(i.e. the muscles) of the examiner.

Muscle Energy theory assumes


Abnormally shortened muscle(s)
--> part of the mechanism of somatic dysfunction
-->These shortened muscles prevent a full range of motion of its joint(s) in some
plane

Mechanism for abnormally short muscle


Neuroreflexive (predominant mechanism)
Fibrosis or gelosis (frozen muscle)
Active or Passive congestion (assoc. w/ myofascial trigger points)

ME is direct or indirect. shorten or lengthen muscle?


Attempts to lengthen the distance between the origin and insertion of certain
muscles
Direct

method of ME

Uses Muscular Activation to overcome motion restriction of tissues and joints


-Uses Muscle Spindles
-Causing Post-Isometric Relaxation

what is the activating force?


Patient provides
Breath counts as a Force

indications
Acute: patient unable to relax
local or general muscle tension is increased
Chronic: somatic dysfunction with fibrotic, shortened tissues
If Tx using Ease hasn't worked
Improved localization = less force needed

too much force=


opposite effects- post-treatment soreness is common and may happen

patient comes with stiff neck ,what do u do


Rule out infections/any trauma then precede to treat and advise the use of
medications.

Two Questions you must answer to decide upon the ME treatment:


for RIBS
Inhalation/Exhalation Dysfunction?
Primarily Bucket handle &/or Pump handle motion? [ribs 11 & 12: caliper motion]
-NOT Flexion/Extension, Sidebending, Rotation

treatment of exhalation or inhalation disfunction


exhalation dysfunction- move ribs into inhalation
inhalation dysfunction- move ribs into exhalation

DIAGNOSIS for muscle energy of ribs


Inhalation rib or Exhalation rib" with 'Bucket handle &/or Pump handle' elements
i.e. Right rib 4 inhalation dysfunction, pump handle primary

each side is ______ of the other (ribs)


independent

in scan u perform one motion scan to confirm (MUSCLE ENERGY)


sidebending OR rotation

dysfunction trick for anterior ribs PUMP HANDLE


Space above 'Wide' (below, 'narrow') = exhalation dysfunction (resists inhalation)
Space above 'Narrow' (below, 'wide' = inhalation dysfunction
COMPARE RESPONSE TO EXHALATION AND INHALATION
AE and NI

dysfunction trick for anterior ribs BUCKET HANDLE


Compare intercostal space above and below for width:
Space above 'Wide' (below, 'narrow') = exhalation dysfunction (resists inhalation)
Space above 'Narrow' (below, 'wide' = inhalation dysfunction
Confirm by Comparing its response to inhalation/exhalation with that of the one on
the other side or with the rib above and below.

Exhalation Dysfunction
Ribs 11-12
-Patient prone ARMS ABOVE HEAD, Doctor on opposite side of the table
-Legs pulled toward doctor to side bend, ipsilateral arm above head
-Doctor grasps ASIS and rotates pelvis posteriorly
-Doctor places hand laterally over involved rib
-Patient takes a deep breath, Doctor pushes laterally on the rib (gapping joint)
-Hold breath & localization 3-5 seconds
EVERYONE RELAX!!
-Reposition (increase traction on rib and increase posterior rotation of the pelvis)
-REPEAT (3-5 times)
-RECHECK

why patient arm above head in exhalation dysfunction 11-12 rib


Patient's arm position uses the latissimus dorsi to facilitate the movement of the rib
up (into restriction) while resisting the action of the quadratus lumborum

Inhalation Dysfunction
Ribs 11 & 12
-Patient prone ARMS AT SIDE, Doctor on opposite side of the table
-Legs pulled toward doctor to side bend, ipsilateral arm at side of body
-Doctor grasps ASIS and rotates pelvis posteriorly
-Doctor places hand proximally over involved rib (medial fulcrum)
-Patient takes a deep breath, exhales; Doctor pushes laterally on the rib (gapping
joint)
-Hold 3-5 seconds
-EVERYONE RELAX!!
-Reposition (traction on the lower ribs and posterior rotation of the innominate)
-REPEAT (3-5 times)
RECHECK

Inhalation Dysfunction
Rib 1-10 - Pump Handle:
-Patient Supine,
-Flex the spine to dysfunctional segment, supported for doctor
-Add sibebending toward the rib
-Doctor contacts superior aspect of rib
monitor anteriorly
-Respiration is activating force, patient takes a deep breath, doctor resists
inhalation and follows the rib more with exhalation
-Reposition (increase flexion and sidebending)
-REPEAT (3-5 times)
RECHECK
Engaging resistance is augmented by flexion & sidebending of the axial spine.

Inhalation Dysfunction
Rib 1-10 - Bucket Handle
Patient Supine,
Flex the spine to dysfunctional segment, supported for doctor
Add sibebending toward the rib
Doctor contacts superior aspect of rib
monitor anteriorly
Respiration is activating force, patient takes a deep breath, doctor resists inhalation
and follows the rib more with exhalation
Reposition (increase flexion and sidebending)
REPEAT (3-5 times)
RECHECK
Engaging resistance is augmented by flexion & sidebending of the axial spine

Exhalation Dysfunction
Rib 1-10 - Pump Handle:
Patient Supine,
Doctor contacts inferior aspect of rib; finger pads contact via interspace below the
rib (chondral)
Monitors and supports inhalation phase.
Extend spine, if possible
Respiration is activating force, patient takes a deep breath, doctor follows the rib
with inhalation & resists exhalation.
Reposition (increase extension?)
REPEAT (3-5 times)
RECHECK
Engaging resistance is augmented by extension of the axial spine - use the Head of
the table

Exhalation Dysfunction
Rib 1-10 - Bucket Handle
-Patient Supine,
-Doctor contacts inferior aspect of rib; finger pads contact via interspace below the
rib (mid-axillary line)
-Monitors and supports inhalation phase.
-Sidebending patient away from the dysfunctional rib

-Respiration is activating force, patient takes a deep breath, doctor follows the rib
with inhalation and resists exhalation
-Reposition (increase sidebending)
-REPEAT (3-5 times)
-RECHECK
Engaging resistance is augmented by sidebending of the axial spine away.

Alternative: Respiration as Activating Force - Pump Handle


Treatment- exhalation and inhalation
Posterior Hand: disengages the posterior articulation (as done for Exhal. Dys.
Previously)
Anterior Hand: Thumb pad contacts anterior rib
Exhalation Dysfunction:
below the costochondral jct. (narrow part)
Inhalation Dysfunction:
above costochondral jct. (narrow part)

Alternative: Respiration as Activating Force - Bucket Handle


Treatment
Posterior Hand: disengages the posterior articulation (as done for Exhal. Dys.
Previously)
Lateral Hand: Thumb pad contacts anterior rib
Exhalation Dysfunction:
below the costochondral jct.
Inhalation Dysfunction:
above costochondral jct.

Anterior subluxed rib diagnosis

Rib Anterior Subluxation Treatment - Right 5th Rib


Pt's. right hand holds left shoulder
Control motion of the thorax via the left hand contact with the pt's right elbow &
abdominal contact with the pt's back
Right thumb creates the fulcrum
Via contact with the shaft of the rib, Medial to the rib angle
Pt is instructed to pull the right elbow laterally or caudally
Isometric, relax, repeat, retest
Additional Fulcrum:
Pt's left fist on anterior rib to create A-P force during contraction
Fulcrum: Thumb creates & maintains posterolateral 'pull' on rib shaft

goal with engaging anterior subluxed rib

Rib Anterior or Posterior Subluxation Treatment - Right 5th Rib


-SUPINE
Same principles and contacts:
Here, pt's. body weight helps hold finger pad contact against the medial aspect of
the rib angle. [ or lateral for posterior subluxation]

Muscle Energy Technique Rib Posterior Subluxation Treatment - Right 5th Rib
Difference with Tx for Anterior Subluxation:
Posterior Thumb pressure is MEDIAL
Isometric force: pt. tries to adduct elbow and physician resists

what is the mobile unit?


Bony structure with articular surfaces for segmental movement
Adnexal tissues (neuromuscular & connective)
Control system

the body is not static

In the standing position, the body is not "static"


The spine = complex organized arrangement of mobile parts
The local segment responds to motion and the overall need for erect positioning
Response to gravity
-The spine is a complex organized arrangement of mobile parts that cam be
positioned
-Within this motor function, the examiner is looking at the local response to gross
motor demand (e.g. standing) is an essential concept for the procedural principle
being applied to investigate motor function.

Fundamental Unit of Somatic Dysfunction


- the mobile unit
-3 segment group
-Central vertebra/rib = primary focus
-Segments above and below respond by compensating in a predictable way

central segment versus those above and below


The central segment can not respond to a movement request in a coordinated
fashion with the segments immediately above and below
Those segments compensate for this as much as possible, but the motion is still
altered

somatic dysfunction encompasses


Skeletal, Arthrodial & Myofascial Structures
Related Vascular, Lymphatic & Neural Elements

Direct techniques and examples


are ones in which the restricted joint or tissue is initially taken into the direction of
resistance to motion"
Muscle energy, HVLA

Indirect techniques and examples


are those that initially position the joint or tissue away from a barrier to motion and
toward the relative ease or freedom of motion"
Strain-counter-strain, FPR, Functional Methods

what relationship is important in FUNCTIONAL METHODS


Neuro-reflexive relationship************* Functional Methods uses the Neuroreflexive response to motion input as the basis for treatment - ribs, axial spine, etc.

functional technique describe defining characteristic


3 Rotary Motion Asymmetries
3 Translation Motion Asymmetries
Anterior/posterior rotation
Left/right translation
Cephalad/Caudad translation
Respiration Asymmetry
COMPLETE MOTOR ASYMMETRY (imp vocab word)All motions are involved when there is somatic dysfunction present
*note above and below!! inhlation and exhalation

The Functional Methods Treatment ProtocolStand on the side of side bending ease
Start by engaging sidebending toward ease
Follow with rotation toward ease
Follow rotation with flexion/extension toward ease
Follow with anterior/posterior translation toward ease
Follow anterior/posterior translation with left/right translation toward ease.
Follow with cephalad/caudad translation toward ease
Each motion is refined during the respiratory ease

Functional Technique- Thoracic/Lumbar Positioning


Patient is seated with arms crossed
At the end of the treatment table
Doc on the side of sidebending ease
Example - SB ease to left
Doc stands on left side of patient
The left upper extremity reaches under the opposite armpit of the patient

Functional TechniqueCervical Positioning


Doc seated at the head of the table
Use finger pads of your middle finger to palpate segment and use your palms to
input motion

Functional TechniqueRib Cage Positioning


Side of sidebending ease if using trunk motions
And/or
Upper extremity input is most often also utilized to treat a rib dysfunction

Functional TechniqueUpper Extremity & Rib Function


Upper Extremity Motion Input
Assess for asymmetric response (ease/bind) at the rib angle. Motions:
Abduction/Adduction
External/Internal Rotation
Cephalad/Caudad Compression

how fast do you put the motions in *


SLOWLY

your are looking for what kind of response*


IMMEDIATE-

one hand is always*


listening

mistakes with functional methods


Do not carry any motions too far because you could actually increase tension
There is such a thing as positioning "past" ease
This can be used to refine your sense of optimal ease as you learn.
Not necessary after you become more comfortable with the feel of 'ease'

term to describe the addition of motions, when do you refine the


motions
The order and the process of placing all six motions into its ease has been termed
'Stacking'
Each motion is refined during the respiratory phase of ease
-Ride the phase of respiratory resistance maintaining as much ease as possible*

How many breaths are necessary?**


Until you feel no increase in tension as you return the patient towards neutral (good
approach for beginners) OR
You feel the central segment and the adjacent mirror image segments feel the same
OR

Until you feel a 'release'

Our goalSimultaneous input of all 6 motions riding ease through respiration to resolution of
the dysfunction
=they make us slow down to optomize efficacy of treatment

COMPLETE MOTOR ASYMMETRY


All motions are involved when there is somatic dysfunction present

documentation for functional methods


T6- flexed rotated sidebent right translated posterior left and superior
and inhalaiton
T 6: FRSr, Tran PLS, Inh

way to remember motions


S
R
E
P
R
S
respiration

how do you engage translational motion


USE YOUR BODY CONTACT TO INTRODUCE THESE THREE MOTIONS

difference between axial spine and rib adjustments FUNCTIONAL


METHODS
AXIAL:
Shoulder/Trunk input ONLY
Contact transverse processes
RIB:
Shoulder/Trunk AND upper extremity input
Contact rib angles

Regional Interactions: axial and ribs and upper extremity


upper extremity and axial spine affect ribs and converse

resistance to inhalation testing technique


Resistance to inhalation is a more immediate palpable cue. This phase (inhalation)
is the easiest to monitor
Resistance to exhalation can be confirmed by the characteristic mirror image
resistance to inhalation for the segments above and below.

where is listening hand in treatment?


'listening hand' may be on the ipsilateral or contralateral rib angle depending upon
the sidebending ease. MUST BE ON RIB ANGLE

Upper Extremity Influence Upon Rib Function


Upper Extremity Motion Input:
Assess for asymmetric response (ease/bind) at the rib angle. Motions:
Abduction/Adduction
External/Internal Rotation
Cephalad/Caudad Compression
ALSO A COMPLETE MOTOR ASYMMETRY - FOR UPPER EXTREMITY

treatment?
Patient: Lateral Recumbent
Physician: Standing in front of and facing the patient

founder of funcitonal methods


Dr. Johnston

Cross-Column Mirror Image Findings


When there is a rudimentary rib dysfunction - a rib preferring exhalation - mirror
image findings are found with the associated vertebra

rib that prefers inhalation does what?


RIB INHALATION DYSFUNCTION - all the vowels
I
E
A
Example:
Rib 4 inhalation ease (exhalation resistance)
Pattern of Ease:
Adduction
Internal Rotation
Cuadad Traction

rib preferring exhalation does what?

Example:
Rib 4 exhalation ease (inhalation resistance)
Pattern of Ease:
Abduction
External Rotation
Cephalad Compression

hand placement for functional methods with arm movement


...

Always take into account


chief complaint => leading to a diagnosis and relevant OMM case considerations:
BIOMECHANICAL
Fluid (Venous-Lymphatic)
Nervous System
Sympathetic
Parasympathetic
Pain

components to fixing a discunction

2 main regional considerations

Biomechanics and leverage

Case Considerations above the diaphragm CORD LEVELS


==Musculoskeletal Anatomy
Cranial and Cervical
Thoracic and Ribs
Upper Extremities
==Autonomic Innervation
T1-4 Head and Neck
T1-5 Heart, T1-6 Lungs
T5-9 Upper GI
T10-11 Colon to the Splenic Flexure, Renal, Gonadal
T12-L2 Lower Colon
T3-T8 Upper Extremity
==Lymphatics

case considerations below the diaphragm CORD LEVELS


==Musculoskeletal Anatomy
Lumbar
Sacropelvic
Lower Extremity
==Autonomic Innervation
T12-L2: Pelvic Organs, Sigmoid colon, lower extremity
(T10-L2: Kidneys/Adrenals, Gonads)

==Parasympathetic: Vagus to level of descending colon, Pelvic splanchnic plexus for


pelvic organs
Lymphatic

3 diff clinical approaches


Centrifugal (Focal)
Start with area of complaint and scan outward from there
Screen adjacent regions until no dysfunction is identified
Centripetal (General)
General screen and scan with greater focused attention on area of complaint and
other positive screen areas
Key Lesion (Hybrid)
General screen and scan but focus treatment on "region of greatest restriction" and
sequentially progress from there

3 things which feed into dysfunction


Oftentimes, it all comes together in the somatic dysfunction diagnoses: disturbances
in the Nervous system, Fluid Dynamics & Biomechanical considerations.
Somato-visceral
Viscero-somatic
Somato-somatic

Korr:
A segment in view is a segment in trouble.

UPPER HALF DIAGNOSIS SEQUENCE****


1 Anterior Ribs
2 Thoracolumbar Junction
3 Cervicothoracic Junction
4 Thoracics / Other Ribs

5 Scapulothoracic Articulation
6 Craniocervical Junction / Cranium
7 Cervicals
8 Rest of Upper Extremity

Anterior Ribs
Treatment of anterior ribs and anterior fascia has a direct effect on posterior
costovetebral dysfunction and vertebral dysfunction
Anterior soft tissue relationships impact posterior relationships
Treatment of Sutherland's "Front of the Back"
Therefore, evaluation for anterior rib 'focal points of dysfunction' = 'fulcrums' is a
good place to start

anterior fulcrum and stress on anterior chest?


Kyphotic posture: Converging to a fulcrum in the anterior rib cage
Rib Dysfunctions- e.g. rib 6 exhalation dysfunction, pump handle primary - can lock
in the fulcrum/focal point of dysfunction
Normal Posture: no convergence to a fulcrum
Trauma or repetitive stress could create an anterior fulcrum, however

lumbar lordosis problems?


Lumbar Lordosis: Patients tend to flatten thoracics in response
Fulcrum becomes higher
Rib 3 inhalation dysfunction could reinforce this postural challenge

what if you find upper rib problems?


If you find rib problems, especially in the top 6 ribs, these ribs will be important
factors in upper body complaints

Pump Handle Relationships of Interest, in particular


Monitor Pump handle Motion at the costochondral junction will be our primary
focus initially
Scan the ribs, esp. 1-6
Segmentally define the pump handle aspect
Wider gap above or below?
Response to respirations:
Inhalation
Exhalation

ways to treat anterior ribs


-Counterstrain
Intercostal tender points
-Muscle Energy
Pump handle ribs
-Anterior Rib Fulcrum [new technique]
-Functional Methods
Anterior rib release
Same contact as muscle energy, but treating in the directions of ease

diagnosing anterior ribs


Diganosis:
Listening: Index fingers contact the anterior rib cage sequentially, at the level of
each rib
Motion: Use forearm contact bilaterally to introduce rotation right and left
alternately
Feel for a sense of increased localized tension during rotation under one of the
palpating fingers
Localized increased tension = a rib fulcrum
Start at rib one and work down
assess responses to
Rotation
Flexion/Extension
Sidebending
Translations
Using the forearm contact and the Osteopathic belly, carry the patient into the
directions of resistance
The localization of tension should be confined to the diagnosed anterior rib fulcrum.
Use the Muscle Energy treatment sequence

Thoracolumbar Junction
WHAT DO WE FIND HERE? how would u treat it?
Biomechanical transition zone
Postural implications on upper transition zones and segments
Relationship of 12th rib and/or diaphragm can have dramatic impact on remainder
of costal cage (and secondarily the cervical region)
Diaphragm
Supine/seated release [new technique]
Muscle Energy/ Direct Stretch
11th & 12th ribs [new technique]
T11-L2
Functional Methods/FPR
11th & 12th ribs
T11-L2

muscle energy thoracolumbar junciton- diagnosis- DIRECT STRETCH


TECHNIQUE
Diagnosis:
Patient seated:
Contact the lateral inferior border of the 12th (or 11th) rib
Press gently superior and laterally; compare with opposite side for ease/resistance
Determine the tight side.
Treatment:
Same position and contact
Press gently superior and lateral on the under side of the rib
Carry the patient into flexion & sidebending until the 'feather edge' of resistance is
encountered
Continue as a direct stretch, or
Have patient try to side bend back to neutral (muscle energy

Cervico-Thoracic Junction
[Superior Thoracic Aperture]
A cadaveric depiction shows this area slightly higher than in a living patient.
Why? The ribs go into exhalation at death.
Superior Thoracic Aperture & Terminal Lymphatic Drainage
First rib/clavicle/vetebral unit are the superior thoracic aperture with associated
visceral structures
Dramatic transition zone biomechanically
Soft tissue relationships connecting the vulnerable visceral neck to the sturdier
thorax
Thoracic duct - Left and Right
Therefore, Diagnose & Treat:
C7-T1 with associated 1st rib
Can extend out to C6 and T2 with 2nd rib

3. Cervico-Thoracic Junction
[Superior Thoracic Aperture] treatment
Treatment
1st Rib:
Indirect: Counterstrain, FPR, FM
Direct: Muscle Energy
Superior Thoracic Aperture
Supine release [new technique]- STEERING WHEEL
Anterior cervical fascia releaseseated [new technique]- choke hold

SUPERIOR THORACIC APERTURE- steering wheel- SUPINE RELEASE


Supine Diagnosis & Treatment:
Thumbs go posteriorly: contact costo-transverse junction bilaterally (this allows

control of T1 and the first ribs)


Fingers wrap around the neck: the finger pads come as close to the junction of the
first rib and manubrium as possible (this means the finger is looping over the
clavicle.)
Gently try to rotate the superior thoracic aperture left & right
to the 'feather edge of resistance'
Which direction is easier? (like screening)
Gently side bend left & right
Which direction is easier?
Treatment: carry the sup. thor. Aperture in both directions of ease

Anterior cervical fascia releaseseated


[new technique]
CHOKE
Patient Seated
Place thumb pads between the two heads of the sternocleidomastoid muscle on each
side
Patient slumps - back and neck
Let the thumbs follow the tissue inferior (not much posterior)
Hold that position with the thumbs as the patient inhales, holds breath and
straightens up

4. Thoracic/Other Ribs
Upper thoracics provide sympathetic innervation to viscera above the diaphragm
Heart, Lungs, Head and Neck
Upper thoracic regions provides important biomechanical relationships to cervical
spine, thoracic inlet, ribs and associated scapulothoracic joint
A Non-Neutral in the T4-6 area is not uncommon.
Correlate with Sympathetic innervations and associated viscera.
autonomic nerves and intercostals *

4. Thoracic/Other Ribs: Treatment


Counterstrain/Functional Methods/Muscle Energy (HVLA - will learn later Winter
quarter; LAR in year 2)
You already know how to treat somatic dysfunction in this region.

5. Scapulo-Thoracic Articulation
One of the important articulations of the shoulder
Restriction is usually due to costal cage dysfunction combined with muscular
imbalance
Has attachments to thoracic and cervical spine, as well as rib cage
Therefore significant contributor to dysfunction in those regions, as well.

May secondarily change biomechanics of the glenohumeral joint


Counterstrain:
Ribs
Pectoralis minor
Trapezius
Levator Scapulae
Serratus Anterior
Rhomboid
Muscle Energy (&/or direct stretch):
Ribs & same muscles

Pectoralis Minor
Assessment & Treatment
Operator contacts the insertion of the pectoralis minor at the 3rd-5th ribs anteriorly
The ipsilateral elbow is abducted sufficiently to bring the vector forces to a
localization at ribs 3-5 anteriorly
Abdominal contact with the patient's back brings stability during the treatment
Have the patient pull the elbow forward with only sufficient force to engage the
pectoralis minor at its insertion.
Isometric force, Relax, Repeat, Retest

Posterior Scapular Muscle Diagnosis & Treatment:

Caudad Hand: Thumb & Web cup the inferior angle of the scapula
Cephalad Hand: Contacts the lateral shoulder and the superior border of the
scapula
With these contacts you can engage stretch for any of the muscles listed.
Is it tight?
It may be obvious or you can compare with the opposite upper extremity
Treatment:
Lengthen the muscle to stretch it
Direct stretch or
Muscle energy
Trapezius
Levator Scauplae
Rhomboid
Serratus Anterior

6. Cranial-Cervical Junction/Cranium
Occiput/Atlas/Axis/Suboccipital tissues
Final common pathway for all forces originating inferiorly
Jugular foramen
Vagus nerve
Greater occipital nerve
Emerges between C1 & C2

6. Treatment:
Suboccipital Release [new technique]
Indirect:
FM, FPR, Counterstrain:
C1
C2
Direct: Muscle Energy

SUBOCCIPITAL REGION
RELEASE OF RESTRICTION(S) BETWEEN OCCIPUT AND SUPERIOR SURFACE
OF C1

DIAGNOSIS:
Place finger pads against the tissues overlying the OA/AA.
Determine resistance vs. compliance (ease)
TREATMENT:
RELEASE TENSION OF SUBOCCIPITAL MUSCULATURE
REDUCE RESPIRATORY EFFORT
MAY BE ACCOMPLISHED BY VARIOUS APPLICATIONS OF FORCE

7. Cervicals
8. Rest of Upper Extremity
...

Rib Landmarks
Under the trapezius- first rib
Sternal angle- anterior second rib
Inferior angle of the scapula -seventh rib
Spine of the scapula- third rib

Characteristics of FPR! passive? active? indirect? direct? soft tissue?


dysfunction?
-Passive
-Indirect
-Facilitating force compression/torsion
-May be used to treat soft tissue or or specific joint dysfunction
-Decreased gain on the muscle spindle gamma loop the spindle becomes unloaded
ceasing the firing of Ia fiber discharges to the motor neuron controlling extrafusal
muscle fiber and cause relaxation.

REASONING BEHIND FPR


-Decreased gain on the muscle spindle gamma loop the spindle becomes unloaded
ceasing the firing of Ia fiber discharges to the motor neuron controlling extrafusal
muscle fiber and cause relaxation.

pump handle? bucket handle? calipers? how to assess each?


Upper ribs ( 1-6 pump handle ) - ASSESS ANTERIOR AND POSTERIOR = increase
in anterior and posterior diameter
Lower ribs (7-10 bucket handle - ASSESS LATERAL SIDE (increase in transverse
diameter)
11-12 caliper motion)- slight increase in transverse and AP diameter -------IF
MIXED middle ribs- can be ant/post/transverse

Segmental Definition
-Patient cross arms in the front
-Find the rib angle
-Place your arm over the patient's shoulder and introduce side bending to the right
and then to the left

-Stand on the side of side bending ease and introduce rotation and note which way
is ease
-Ask the patient to drop his or her shoulders and then straighten up
-Ask the patient to take a deep breath in and exhale. Under your finger is there
decreased tension with inhalation or exhalation. Does the rib angle move smoothly
with inhalation or exhalation. (up with inhalation and down with exhalation

Anterior ribs
Find the same rib anteriorly. Contact is at the costalchondral junction.
Find sidebending,rotation, and flexion or extension ease
Now have the patient take a deep breath in and exhale.
If the patient favors exhalation the rib moves down easily and the tissue becomes
less tense under your finger

lateral ribs
Now follow the same rib laterally
Stop in the mid-axillary area.
Sidebend, rotate and flex/extend finding the area to ease.
Now have the patient take a deep breath in and exhale do the move lateral and
superior with inhalation?

3 motions and name all 3 parts of spine you can test


Sidebend, rotate and flex/extend
anterior posterior and lateral

FPR procedure! NAME ALL 6 steps


Find dysfunction
Flatten the curve
Compress to dysfunction
Add rotation/side bending ease
Hold 3-5 seconds
Recheck

how to define inhalation and exhalation


ask patient to sit up straight and drop shoulders

FPR is different from SCS HOW!?


FPR- Looking for tissue tension/ activating force
CSC- looking for tender point/no force
BOTH FIND EASE, BOTH ARE INDIRECT METHODS

When is activating force added : before or after putting the segment


into ease
The sequence can be varied depending on the skill and experience of the physician

FPR rib
Monitor dysfunction with one hand,
Reaches across cervicothoracic junction (CTJ). Forearm is over shoulder.

Tell patient to sit up straight. 'Push the chest forward.'


Compress downward and introduce side bending/rotation as needed.
Hold for 3-5 secs, release, re-evaluate.
CAN ALSO DO FACEDOWN

anterior versus posterior rib adjustment


anterior rib (exhalation- push on front) posterior (inhalation)- in both treatments,
stand behind patient

what important thing do you do while you monitor and before you
position a patient
apply the downward force-

position of arm for rib adjustment


arm over cervicothoracic junction, and elbow over acromion

FPR-Lateral Rib
Patient seated
Physician standing behind patient
Physician axilla at the CT junction
Monitor dysfunctional rib
Grasp flexed elbow and abducted arm 45 degree
Ask patient to straighten up
Compress to the dysfunction
Hold 3-5 seconds
Recheck

1st Rib
Patient supine
Physician on the side of the dysfunction
Contact the first rib with the hand closest the patient
With the other hand abduct the patient arm with the elbow flexed
Compress into the dysfunction
Internally rotate and adduct the arm
Release and recheck

1st Rib
1. Stand on side of dysfunction.
2. Place caudad hand over 1st rib - monitor.
3. Bend patient's elbow,
4. Flex/abduct arm to maximum ease under monitoring fingers.
5. Compress at elbow with force directed toward rib
6.Internally rotate arm against physicians' caudad forearm
7.Hold 3 seconds
8.Abduct followed by circumduction back to anatomical position-maintain
compression
9.Release and re-evaluate

What are two defining characteristics of FPR?


-Place the patient in the position "EASE" for the segment
- Activating force cuts down treatment time to 3-5 seconds.

Name some characteristics of FPR.


-Indirect technique
-Accurate DIAGNOSIS is essential
-Placement of the spine in a NEUTRAL position in order to disengage\idle facets
(flatten the axial spine)
-Superficial Muscles: ABNORMAL MUSCLE TENSION
-Deep Muscles/ Segment: based on segmental definition
-ACTIVATING FORCE
(compression, torsion, or traction)

Compare and Contrast FPR and Strain Counterstrain.


Screen and Scan to locate a segment
**Find the tender point associated with the segment (anterior/posterior)
-Place patient into a position of comfort
-**Hold for 90 seconds
-Reposition without patient help
-Reassess
FPR
-Screen, Scan and accurately Segmentally
**Define a segment
-Flatten lordosis/kyphosis of the spine
-Add activating force

-Place segment/musculature into a position of ease


-**Hold for 3-5 seconds
-**Reposition without patient help (important to make sure the pain is gone)
-Reassess

Characteristics of good FPR


-Easily applied
-Non-traumatic
-Effective and efficient
-Relief of tenderness and restores function
-Can be repeated
-Other methods of treatment can be applied immediately afterward

Right Suboccipital Muscle Hypertonicity


Pt supine
Right hand:
-Doc supports head
-Hand on general tissue to be treated
Left Hand:
-Gently flatten A-P curve (slight flexion)
-Add activating force of gentle axial compression
Maintain compression and move head in positions of ease
Hold for 3-5 seconds
Release compression while returning to neutral
Recheck

Cervical Segmental Somatic Dysfunction/Deep Muscles

C4 ER(R)S(R)
-Gently support cervical region with index finger on C4
-Flatten the cervical lordosis
-Add compressive force to C4, directed through head and neck.
-Extend neck through level of C4
-Slightly sidebend and rotate to right to maximal ease
-Hold 3-5 seconds
-Slowly return to neutral
-Recheck

Treatment of Thoracic Superficial Muscles


-Pt seated
-Monitor hypertonic area with listening hand
-Doc places left forearm on pt's left shoulder and behind neck
-Pt instructed to "sit up straight" to flatten kyphosis
-Add compressive through to listening hand
-Sidebend and rotate the pt toward the listening hand
-Hold 3-5 seconds
-Return to neutral slowly
-Recheck

Thoracic Segmental Somatic Dysfunction/ Deep Muscles:


T7 ER(L)S(L)
-Pt seated

-Finger on posterior transverse process


-Forearm on pt's left shoulder and behind neck
-Instruct pt to sit up straight to flatten kyphosis
-Add compressive force down to T7
-Extend through level of T7
-Sidebend and rotate left to maximal ease
-Hold 3-5 seconds
-Slowly return to neutral
-Recheck

Alternate Method for Upper Thorax


T3 ER(L)S(L)
-Prone position flattens thoracic kyphosis
-Stand opposite the dysfunction
-Place your finger on posterior TP of T3
-Grasp pt's shoulder with caudad hand
-Pull shoulder toward pt's feet to induce sidebending
-Maintaining force pull pt's shoulder backward off the -table creating left rotation
-Hold for 3-5 sec
-Recheck

Mechanics of the lumbar spine


The lumbar spine (L5) and the sacrum move in opposite directions.

Mechanics of External Rotation of the Hip


External rotation of the right hip
Results in rotation of the spine in the opposite direction
(which is left in this example)

Mechanics of Internal Rotation of the Hip


-Internal rotation of the right leg (pelvis rotation left)
-Results in rotation of the lumbar spine to move towards the leg that is internally
rotated
(Which is the right side in this example)

internal versus external rotation for treatment L3 ER(L)S(L)


-(extend ipsilateral leg if internal, contralateral if it is external rotation)
- side-bend to the left both times
- internal- rotates pelvis right (ipsilateral)
-external- rotate pelvis right (contralateral leg)

Lumbar Soft Tissue TreatmentHypertonic Left Low Back Muscles


Pt prone
Pillow under abdomen to flatten the curve
Doc at left side of table monitoring hypertonic area (left paraspinal region)
Doc places left knee on table next to pt's ilium - this is your fulcrum
Induces sidebending using knee as a fulcrum

Lumbar Soft Tissue TreatmentHypertonic Left Low Back Muscles


Place pt's right ankle over left
Reach around and grasp pt's right thigh from the lateral surface.
Extend posteriorly
Externally rotate contralateral leg/ hip (causes the spine to rotate left)
The torque is the Activating Force
Hold for 3-5 seconds and release slowly toward neutral
Recheck

Treating the Lumbar Spine Using a Long Lever


L3 FR(L)S(L)
-Pt prone
-Pillow under abdomen to flatten lordosis
-Doc seated at left side of table facing pt's head with right lateral thigh beside table
-Monitor the left transverse process with finger of left hand
-Drop pt's flexed left knee and thigh off table, over doc's right thigh
-Grasp pt's knee with right hand and flex hip
-Adduct the knee toward the table
-Some compression may be added through the pt's knee
-Hold for about 3-5 seconds and slowly return to neutral
-Recheck

Treating the Lumbar Spine Using a Long Lever


L3 E!!R(L)S(L)
Pt prone with pillow under abdomen
Second pillow between pt's thigh and table
Doc standing on left side
Monitor left transverse process of L3 with left index finger
Grasp lower ankle
Abduct leg (creating left lumbar sidebending)
Internally rotate leg until you feel it at your monitoring finger (this causes lumbar
rotation toward left)
Press left leg down towards floor (results in extension of low back)
Hold for 3-5 seconds and slowly return to neutral
Recheck

why do a focused screen?


Complete, Full-Body Screen
-Useful as a learning tool
-Useful at the first visit
-May be time consuming!
Problem-Focused Screen
-Focus on areas relative to a problem
-Asthma
-Ankle Sprain

focused screen- Determining Areas to Screen


1Anatomy
2Visceral Relationships
SNS
PNS
Lymphatics
3"Follow the Tissues"
4Follow the patient('s history)

visceral relationships
Visceral Relationships
Sympathetic Nervous System (aka SNS)
Controls "Fight or Flight" responses
Lymphatics!

Parasympathetic Nervous System (aka PNS)


Mediates the "Rest and Digest" functions

Follow the Tissues


Palpation of fascial strains
Feel restriction, movement, or flow throughout body
It takes time to develop skills!
Remember the scarf activity we did at the end of immersion?

ankle sprain example


Is there a problem?
Tear or partial tear of ankle ligaments
What's the problem?
Pain
Swelling
Bruising
Inability to bear weight
Decreased ROM
Where is the problem?
Fluid movement - swelling, choke points, inactivity
Biomechanical - pain, decreased movement, ligamentous strain or integrity change,
membranous and muscular tension, bony involvement, changes in ambulation and
posture
Psychosocial - impact on life, work, well-being

ankle sprain can affect all above


lumbars sacrum and pelvis

asthma screen
Asthma
Is there a problem?
Hyperreactivity over a trigger causing spasm of the small airways
What's the problem?
Cough, can't breathe (SOB), chest heaviness, wheezing, fatigue
Mucus!
Anxiety, panic
Postural- anxiety, splinting
Visceral - coughing, decreased air movement, cavity pressure changes
Fluid movement - thoracic inlet, abdominal diaphragm, pelvic diaphragm
Biomechanical - tight muscles, ribs/thoracics stuck due to breath patterns, pain
Heightened Sympathetics
Depressed Parasympathetics
Psychosocial - fear, medication side effects

asthma involves
cervicals thoracic ribs and lumbars

osteopathic diagnosis and treatment


Correct Medical DiagnosisFirst
=Patient history, physical exam, labs/imaging
Correct Structural/Osteopathic =Diagnosis (Screen, Scan, Segmental
Definition_Second
ManagementThird
-Education
-OMM
-Medications
-Home Exercises/Stretches
Follow-upFourth

Segmental definition
"What is the Problem?"
Diagnostic Detail of Dysfunction
(then management)

Segmental Definition
What is the nature of the problem at the segment?
Per Foundations: Final Stage of Somatic Examination in which the nature of the
problem is detailed at the segmental level
Rotation - do the twist
Sidebending - rock to the music
Flexion/Extension - take a bow
BRING IT DOWN TO ONE VERTEBRA

diff between active and passive motion in seg definition


Active Motion
Motion accomplished by the Patient
Not as consistent
Used during some screening motions
Passive Motion
Motion generated by the Physician
More consistent
Used predominately during screen and scan
Both should reveal the SAME osteopathic diagnosis
Aside from what they are COMMONLY used for, think of why they MAY or
SHOULD be used...

what is the key to an effective scanning technique

Only if you have a motor hand and a listening hand


Only if you put motions in EVENLY!

process of diagnosis
-Palpate Transverse Process of Dysfunctional Segment to Determine Position of
Ease
-Use minimal amounts of force
-Use the position of ease to name the dysfunction

naming dysfunction important identifications


Identify Vertebral Level
Identify Rotation Direction
Identify Direction of Side-Bending Ease
Identify If Vertebrae is Flexed or Extended
ERS or NSR describe position (subscript L and R)

OMM SOAP NOTE goal


Capture of patient presentation/history
Documentation of structural exam
Assessment
Plan of Action
Implementation of plan
Billing/Coding
Bookmarks/Post-its (for yourself and fellow docs!)

Subjective:
Chief complaint
History

Objective:
Physical exam
Neurological
Musculoskeltal
Other pertinent systems
Structural/Osteopathic
A specific somatic dysfunction, listed by nomenclature or description of
TART/STAR findings is an objective finding WHICH is no different than describing
heart or lung sounds
Detailed Structural Findings from Observation and Segmental Definition
O: Vitals: BP 120/80, P 80, R 14, T 37C
MSK: strength 4/5, engagement of adductors causes pain
Neuro: reflexes +2/4, achilles and patellar, sensation intact to touch, temperature
and pain
GU: no hernia, swelling, masses or discharge
Struc: DYSFUNCTIONS EVERYWHERE! (lower extremity and pelvis, that is)
Ex: posterior fibular head, tight adductors, tight psoas, L on R sacral torsion

Assessment:
THE diagnosis(es)
Medical diagnoses or symptoms are listed before the diagnosis of somatic
dysfunction
Somatic dysfunctions are listed by region (body area) not the name of the specific
finding.

Plan:
List OMM performed as part of plan:
-Region treated
-Treatment type used
-Patient response to treatment
---Subjective
---Objective
-Discussions
---Stretching
---Exercises
---Education
1. RICE, ibuprofen as needed, stretches demonstrated, decrease activity, follow-up
2. Strain Counterstrain (SCS) of anterior and posterior pelvis points - patient
verbalized relief of pain
3. Soft tissue (inhibition/kneading) of LE - warming, decreased tissue tension noted
by operator

Rib 1a: the pose of despair


-leg same side as tender point
-patient drapes arm over that side
-side bend head to that side with flexion-tune tender point
-pay put legs on the table and flexed to induce more effect

Rib 1b:
The Lazy Student's Question
In supine position, upper extremity is flexed above the head, elbow slightly bent.
Forearm is placed on operator's palpating arm, fine tuning with traction and further
flexion of the arm until tender point is relieved.

Anterior C1
-find behind ascending ramus of mandible half way between mastoid process and
inferior angle of the jaw
-treat: rotate head 90 degrees away, apply side-bending away to reduce sensitivityapply caudal force on contralateral parietal aspect of cranium- do not apply
extension and do not maintin position if discomfort increases

Trapezius
-Tender point is either anterior or posterior along superior edge of trapezius muscle.
-abduct arm, suspend from fingers and flex elbow.
-Fine tuning anterior or posterior depending on point location

levator scapulae
inferior attachement of muscle at levator scapulae, pull arm back to rotate scapula
medially. push scapula up to head
-head should be on pillow to assist in side-bending

Levator Scapulae 2.0


-extend the arm slightly
-head rotated to tender side,
-inferior traction while holding the wrist
-extremity in internal rotation

Iliacus
medial from ASIS by 7 cm
-flexion of hips
-lateral rotation of hips
-knees abducted

Piriformis
b/w PSIS and coccyx (midway)
-flex same side hip 135 degrees and laterally rotate (especially if tenderness is
lateral)

gluteus medius
upper outer portion of gluteus medius
-extend hip, abduction and lateral rotation

FIRST RIB COMPLEX


First Rib
Anterior C1
Trapezius
Levator Scapulae FLAT!

LOW BACK COMPLEX


Iliacus
Gluteus Medius
Piriformis
Sacrum
(PIGS)

Sacrum
find tenderpoint and press as far from tenderpoint as possible

Somatic Dysfunction
Impaired or altered function of related components of the somatic (body
framework) system: skeletal, arthrodial, and myofascial structures, and related
vascular, lymphatic, and neural elements.

Principles of Treatment
-Find tender point
-Position the patient for maximum comfort
-Maintain the position for 90 sec
-Slowly return the patient to a neutral position
-Recheck the tender point

WHAT TO WORRY ABOUT when doing counterstrain


!!!!!!Without identification of a tender point you cannot do counterstrain!!!!!
-MUST Clearly and discretely identify the tender point
-Pick the most significant tender point in the region
-Maintain light contact with the tender point
-throughout treatment to facilitate recheck

every patient is different!


-The final position of comfort generally follows a pre-established pattern
-Fine tuning the position is a !!!highly individualized procedure!!!
-Occassional points are "mavericks" and requires positioning opposite to what you
might expect
-(may be opposite!)
Quantify the comfort 10-->3 or less, $1.00-->0.30 or less, however you choose

position of comfort of operator


just as important as patient )must have one to have the other)= some aspects of the
counterstrain position may be uncomfortable = MUST COMMUNICATE THIS TO
PATIENT

90 second rule?
- 90 sec (by the clock) is essential at this point in your training EVEN IF YOU FEEL
RELAXATION
-Start the clock only when the patient has perceptibly relaxed (seated positions,
particularly with ribs may require some additional time)!!!!!!!!!!!! RIBS MAY TAKE
LONGER
- CHECK THROUGHOUT TREATMENT WITH LIGHT TOUCH OF FINGERS "the
beeping point"

return to neutral
-The patient must remain passive during the return to neutral
-TELL THEM TO NOT MOVE
-make sure if patient tenses, STOP TREATMENT you tell them to chill before you
finish!
- note first couple degrees are important and try to not excite proprioceptive
mechanisms

reassessment
-CHECK SAME POINT YOU TREATED
- maintain contact during return to neutral
-reduce tenderness to about 70%, if not check for more points, or may need to
repeat again
-Warn patients about treatment reaction

Screen checks for what


REGION (2 resistance to pressure, 2 motion)

Scan (confirms what?)


motion scan and uses deep pressure

LATERAL POSTERIOR T5-T12


abuduct arm, turn head ipsilateral, side-bend trunk away from tender side

IMAGE

(Anterior) Depressed Ribs 1&2


flex neck, rotate towards bad side, and side-bend towards

IMAGE

posterior elevated rib 1!


neck extend and side-bent away and rotate to tender point

IMAGe

Rib 1a:
The Pose of Despair
Operator puts leg on table ipsilateral to tender point, patient drapes ispsilateral arm
over leg and sidebends head to that side with flexion. Operator uses head to fine
tune tender point.
Patient's legs may be brought up on table and flexed to opposite side for further
effect.

Rib 1b:
The Lazy Student's Question
In supine position, upper extremity is flexed above the head, elbow slightly bent.
Forearm is placed on operator's palpating arm, fine tuning with traction and further
flexion of the arm until tender point is relieved.

(Posterior) Elevated Rib 2-6


side-bend away from tender point, to elevate rib, CONTRALATERAL arm behind
back, ipsilateral arm is abducted and supported by physician thigh

(Posterior) Elevated Rib 2-6

RULES
-Examine the patient thoroughly
-One dysfunction can have more than one tenderpoint
-Test with equal pressure
-Use less pressure during treatment
-Trust your palpation
-90 sec rule and give patient time to adjust

RULES AFTER TREATMENT


-Do not engage in any activity that is different from your normal activity for 24
hours
-Drink plenty of water
-Do not test previously restricted motions for few days
30% of patients may experience treatment reaction from 1-48 hours after treatment,
may last 1-5 days
Flu-like symptoms

tender points
-Located in tendinous attachments, bellies of muscle, other myofacial tissues
-Typically discrete, small tense, edematous
-Non-radiating
-Identify most-significant tender point in a group

-Palpate with finger pad


-Use your structural exam to guide you
-Significant tender points result from the patient attempting to obtain a comfortable
posture to alleviate the functional distress
-Tender points tend to be at the apex or the focal point of a the concavity.
-Forward bent=anterior
-Backward bent=posterior
-Sidebent right=tender points to left
-Sidebent left=tender point to right of spine
-Tender points are frequently 180 degrees around the body from the point of
presenting pain

tender points
-Advantage of treatment, further injury or pain is avoided
-Tends to correlate with the position of injury
-Anterior Points>>Flexed
-Posterior Points>>Extended
-Rotation and sidebending increased as move farther away from spine
-Patient must be relaxed
-Position is similar between patients but unique for each patient
-Slowly move patient toward position of maximum comfort, stop and check
-Test tender point while in position
-Light contact during treatment
-If greater than 3 out of 10, keep going until less than 3.
If worse, backtrack until more comfortable again
As you get close to position, fine tune with small movements until maximum
comfort is achieved
May feel therapeutic pulse, sympathetic vasodilation?

90 seconds
Once position of comfort is reached hold for 90 seconds
90 secs does not begin until patient is relaxed
May need to remind patient
Physician comfort is essential
Feel for release of myofascial tissue

first degrees of motion significance?


The first few degrees of motion are most critical
Need to remind patient to remain passive and not help
If they start to help, stop moving, ask them to relax again, wait for relaxation, and
begin moving

re-check?
Once back in neutral position, recheck tender point, should be 30% or less of
original, ideal is 0%
If not better, may not have been optimally positioned
May be another primary tender point

Recheck original structural findings


You are treating dysfunction, not tender point

Posterior Rib 7-10


In the seated position, add extension with rotation and sidebending to the same side
as the tender point.

What is the mobile unit?


The relationships between the segment of interest and the segments above and
below (3 segments)
The Mobile Unit is not comprised of just bony elements. It includes the connective
tissue and neuromuscular elements such as: Proprioceptive and Nociceptive
Sensory feedback that provide input to the brain.

Types of contacts and where


Thumb & index finger
One finger contact
Paravertebral muscles that are superficial to transverse processes (or articular
pillars)

where do you find dysfunction

where you find a bump OR sudden decrease, It is not acting in concert with tissue
around it. It is OUT OF STEP. We check above and below segment and look for
asymmetry.
So the bad vertebra has actions OPPOSITE on the vertebra above and below it.

Local Deep Pressure Test


apply pressure along back superficial to transverse processes.
Look for EASE and RESISTANCE

Red Dots for this lab


You place the DOTS on the MIDLINE. Just because muscles are contracted on one
side doesnt mean that side has the dysfunction/

Local Deep Pressure for Cervical! and Lumbar!


Cervical
You may need to gently support the forehead when using local deep pressure
scanning with a SEATED patient.
Lumbar
Sitting on a stool or kneeling will allow you to use this approach to scanning for the
SEATED or STANDING patient

With one segmental dysfunction finding you need to verify with


ONE MOTION TEST--> Passive motion imput

We will focus on these 2 types of passive motion testing for the spine
sidebending and rotation --> LOOK FOR THE IMMEDIATE RESPONSE!!! Any
further and it is a diff test.

Listening and Motor Hand


one hand feels (segment of interest) and one hand moves

Rotation test of thoracic vertebra


stand behind and to the side. Put hand on spot (listening hand) and rotate left and
right (with other hand on elbow)

motions can be used for these regions too


Cervical
Thoracic
Lumbar
Sacrum
Rib Cage

What if the Motion scan does not yield mirror image asymmetry?
where above and below are not opposite to middle vertebra. You expand the tile to
the motion tested location. Then you confirm with another motion test.

Tissue Shear Test


May use after segment is located
Indicates acuity or chronicity of segment
Assess subcutaneous issue and superficial musculature. Put fingers on transverse
processes and move section up and down.

Chronic dysfunction after shear test


Decreased shear
Increased fibrotic qualities
Chronic contractile and hypoxic state

Acute Dysfunction after shear test


Increased pliability of interstitial connections
Increased fluidity in subcutaneous tissues

Percussion Scan Technique


Use the thumb and index (or middle) finger to contact the paravertebral muscles
that overlie transverse processes bilaterally
Controlled, soft percussion is used to allow rebound on contact.
Use a loose wrist, as if breaking an egg.

Percussion Test
Both tissue texture and motion scan
NOT cervical spine
Compare above vs. below
LOOSE WRIST
A single test that can detect signs of both increased tissue tension and limited
mobility in the same procedure
Can be used in the lumbar & sacral regions, as well
Not used for the rib cage or cervicals

Rules for screening, how many tests per region


how many of each to = somatic dysfunction
Rules for Scanning, how many tests per region and how many for
dysfunction
Screen
2 Tissue Textures and 2 Motions for each of 8 regions
At least 1 tissue and 1 motion asymmetry = Somatic Dysfunction in that region
Scan
1 Tissue Texture and 2 Motions for each of 8 regions
At least 1 Tissue and 1 Motion to identify a segment (2 motions if there is a tie)

name 5 ways to address soft tissue issues


kneading
traction
inhibition

petrisage
effleurage

kneading
rhythmic lateral stretching, force applide perpendicular to muscle axis

traction
longitudinal stretch

inhibition
deep pressure

"joint play" deals with


-articulation of joint architecture
tension and laxity of ligaments and tendons

when to use these soft tissue techniques?


prep for other techniques
reduce adhesions, hypertonicity
applicable to any part of body

dysfunction
2 texture, 2 motion

START
sensitivity
tissue texture
asymmetry
range of motion deficit
tenderness

tissue response
-relax muscles
-inc temp response
-pulsation under finger tips

kneading type
DEEP friction- short, deep thumb strokes
-petrissage- wringing, rate, rhythm and type of pressure- like kneading dough
-skin rolling- may be painful- stimulating petrisage

traction
drawing structures apart

inhibition

steady deep pressure applied, broad or precise.


TRIGGER points- nodules, tender points that do not radiate!
-suboccipital release- prone (may do traction and kneading before-hand)

lymphatic drainage
all the techniques, local , acute, to draw fluid DISTAL TO PROXIMAL towards
thoracic duct!

choke points
open from central to peripheral so appropriate drainage can take place

lymphatic congestion
breathing in with compression to allow for fluid drainage- release quickly

effleurage
helps fluids move along lymphatic channels

Tapotment
start at hypothenar ridge and down, rapid massage to increase tone and arterial
profusion

rules
strokes to heart, broad strokes, encourage deep breathing

How does it all fit together in patient care?


-Correct Diagnosis
Use history and physical exam
-Correct Structural Diagnosis
Screen, Scan, Segmental Diagnosis
-Management
OMM/Medications/Home Exercises
-Follow-up

3 components of diagnosis process


screen scan segmental definition (is there one, where is it, what is it?)

screening process
Structural impression of posturalObservation alignment
Assess superficial muscle tone in 8 regions (Resistance to Pressure)Tissue Texture
Assess gross region motion testing in 8 regions (Resistance to Motion)Motion

testing 8 regions
Tissue Texture and Motion Symmetry allow examiner to assess regional responses
independently

goal of med records


Overall Goals of Record:
Initial Record of Somatic Dysfunction
Evaluation of Response to OMT (short-term/long-term)
Longitudinal Record of Musculoskeletal Health

goal of med records during visit


During Visit:
Identifies somatic dysfunction in a region by use of altered tissue texture and
motion asymmetry
Allows for immediate visual recall of observation

observation
Observation
Gait
Posture

tissue texture
standing

motion testing
Supine
Standing
Seated

3 steps to evaluating a patient


Screen, Scan, Segmental definition
is there a problem?
where is the problem?
what are it's characteristics?

scanning
TART
-tissue texture abnormalities
-asymmetries
-restriction to motion
-tenderness
or STAR
-sensitivity changes instead of tenderness

places to feel for resistance to pressure


upper and lower 3 regions, lateral limbs, anterior ribs

exceptions to passive motion


jaw, costal cartilage, and lumbar spine

define passive motion

minimal force to initiate motion

You should pay attention to which direction resistance is encountered


________________
sooner

...

lower ribs inspiration (LOWER)


supine feel near margins and feel for resistance

S-ar putea să vă placă și